SlideShare une entreprise Scribd logo
1  sur  46
Télécharger pour lire hors ligne
Professor: Fabrício Maia
Matemática
5
○ ○ ○ ○ ○ ○ ○ ○ ○ ○ ○ ○ ○ ○ ○ ○ ○ ○ ○ ○ ○ ○ ○ ○ ○ ○ ○ ○ ○ ○ ○ ○ ○ ○ ○ ○ ○ ○ ○ ○ ○ ○ ○ ○ ○ ○ ○ ○ ○ ○ ○ ○ ○ ○ ○ ○ ○ ○ ○ ○ ○ ○
140 questões resolvidas
“A força não provém da capacidade física e sim de uma vontade indomável”
(Mahatma Gandhi)
01 Os valores de b para os quais a parábola y = x2
+ bx tem um único ponto em comum com a reta y = x – 1 são:
A) – 1 e 3 D) 0 e – 1
B) – 1 e 2 E) 0 e 2
C) – 3 e – 1
Solução:
Temos:
⎧ = +
⎨
= −⎩
2
y x bx
y x 1
Comparando:
+ = −
+ − + =
2
2
x bx x 1
x (b 1)x 1 0
Como as equações têm um único ponto comum, então:
Δ =
− − ⋅ ⋅ =
− =
2
2
0
(b 1) 4 1 1 0
(b 1) 4
Daí: − = → = − = − → = −b 1 2 b 3 ou b 1 2 b 1
Resposta: A
02 Se f(x) = 4x + 1
e g(x) = 4x
, a solução da inequação f(x) > g (2 – x) é:
A) x > 0 D) x > 1,5
B) x > 0,5 E) x > 2
C) x > 1
Solução:
Temos:
f(x) > g(2 –x)
4x + 1
> 42 – x
(base > 1)
Daí: x + 1 > 2 –x
> → >
1
2x 1 x
2
Resposta: B
03 log 50 + log 40 + log 20 + log 2,5 é igual a:
A) 1 D) 10
B) 3 E) 1.000
C) 5
Solução:
Lembre: ⋅
+ =b c b c
a a alog log log
Temos: log 50 + log 40 + log 20 + log 2,5 = log 100000 = log 105
= 5
Soma = 5
Resposta: C
“A força não provém da capacidade física e sim de uma vontade indomável”
(Mahatma Gandhi)
01 Os valores de b para os quais a parábola y = x2
+ bx tem um único ponto em comum com a reta y = x – 1 são:
A) – 1 e 3 D) 0 e – 1
B) – 1 e 2 E) 0 e 2
C) – 3 e – 1
Solução:
Temos:
⎧ = +
⎨
= −⎩
2
y x bx
y x 1
Comparando:
+ = −
+ − + =
2
2
x bx x 1
x (b 1)x 1 0
Como as equações têm um único ponto comum, então:
Δ =
− − ⋅ ⋅ =
− =
2
2
0
(b 1) 4 1 1 0
(b 1) 4
Daí: − = → = − = − → = −b 1 2 b 3 ou b 1 2 b 1
Resposta: A
02 Se f(x) = 4x + 1
e g(x) = 4x
, a solução da inequação f(x) > g (2 – x) é:
A) x > 0 D) x > 1,5
B) x > 0,5 E) x > 2
C) x > 1
Solução:
Temos:
f(x) > g(2 –x)
4x + 1
> 42 – x
(base > 1)
Daí: x + 1 > 2 –x
> → >
1
2x 1 x
2
Resposta: B
03 log 50 + log 40 + log 20 + log 2,5 é igual a:
A) 1 D) 10
B) 3 E) 1.000
C) 5
Solução:
Lembre: ⋅
+ =b c b c
a a alog log log
Temos: log 50 + log 40 + log 20 + log 2,5 = log 100000 = log 105
= 5
Soma = 5
Resposta: C
Professor: Fabrício MaiaMatemática
6
04 Utilizando a tabela abaixo, conclui-se que 5
371.293 é igual a:
A) 11
B) 13
C) 14
D) 15
E) 17
Solução:
Tomando: n = 5
371.293
Daí: log n = log 5
371.293 → log n =
1
5log (371. 293)
log n =
1
log371.293
5
⋅ (veja tabela) → = ⋅
1
log n 5,55
5
→ log n = 1,11 (veja tabela)
logo: n = 13
Resposta: B
05 O número de pontos de interseção dos gráficos de y = 3 logx e de y = log 9x, sendo x > 0, é:
A) 0 B) 1 C) 2 D) 3 E) 4
Solução:
Temos: { = ⋅
=
=
> < ≠
x
a
Sabemos :
y 3 logx
f(x) log
y log 9x
(x 0 e 0 a 1)
Comparando:
⋅ =
=3
3 logx log9x
logx log9x
Daí: =
− =
− =
= − = → = = −
3
3
2
2
x 9x
x 9x 0
x(x 9) 0
x 0(n.s) ou x 9 0 x 3 ou x 3(n.s)
Resposta: B
06 A equação
+ +⎛ ⎞ ⎛ ⎞
+⎜ ⎟ ⎜ ⎟
⎝ ⎠ ⎝ ⎠
=
+⎛ ⎞
⎜ ⎟
⎝ ⎠
k 1 k 1
2 3
1
k 2
5
A) não admite soluções.
B) admite uma solução entre 1 e 5.
C) admite uma solução entre 5 e 12.
D) admite uma solução entre 12 e 20.
E) admite uma solução maior que 20.
Solução:
Lembre:
n n n 1
p p 1 p 1
+⎛ ⎞ ⎛ ⎞ ⎛ ⎞
+ =⎜ ⎟ ⎜ ⎟ ⎜ ⎟+ +⎝ ⎠ ⎝ ⎠ ⎝ ⎠
Daí:
+ + +⎛ ⎞ ⎛ ⎞ ⎛ ⎞
+ =⎜ ⎟ ⎜ ⎟ ⎜ ⎟
⎝ ⎠ ⎝ ⎠ ⎝ ⎠
k 1 k 1 k 2
2 3 3
9
11
13
15
17
...
371.293
log NN
0,95
1,04
1,11
1,18
1,23
...
5,55
Professor: Fabrício Maia
Matemática
7
Substituindo:
+⎛ ⎞
⎜ ⎟
⎝ ⎠
=
+⎛ ⎞
⎜ ⎟
⎝ ⎠
+ +⎛ ⎞ ⎛ ⎞
=⎜ ⎟ ⎜ ⎟
⎝ ⎠ ⎝ ⎠
k 2
3
1
k 2
5
k 2 k 2
3 5
logo: + = + → =3 5 k 2 k 6
Resposta: C
07 A soma dos coeficientes do desenvolvimento de (1 + x2
– x3
)9
é:
A) – 1 B) 2 C) 1 D) 3 E) 4
Solução:
Sabemos:
Se p(x) = anxn
+ an – 1xn – 1
+ ... + a1x + a0, com ≠na 0, a soma dos coeficientes do polinômio é dada por p(1).
Assim: A soma dos coeficientes de (1 + x2
– x3
)9
é dada por: Scoef. =(1 + 12
– 13
)9
= (1 + 1 – 1)9
= 1
Resposta: C
08 Encontre o coeficiente de x2
no desenvolvimento de (x2
+ 2x + 1)4
.
Solução:
Lembre:
Termo geral: −
+
⎛ ⎞
= ⋅ ⋅⎜ ⎟
⎝ ⎠
n p p
p 1
n
T a b
p
Temos: (x2
+ 2x + 1)4
= [(x + 1)2
]4
= (x +1)8
Termo geral:
−
+
⎛ ⎞
= ⋅ ⋅⎜ ⎟
⎝ ⎠
8 p p
p 1
8
T x 1
p
Queremos: 8 – p = 2 → =p 6
Daí:
⎛ ⎞
= ⋅ ⋅ =⎜ ⎟
⎝ ⎠
2 6 2
7
8
T x 1 28x
6
Resposta: 28
09 Calcule n sabendo que
⎛ ⎞ ⎛ ⎞ ⎛ ⎞ ⎛ ⎞
+ + + + =⎜ ⎟ ⎜ ⎟ ⎜ ⎟ ⎜ ⎟
⎝ ⎠ ⎝ ⎠ ⎝ ⎠ ⎝ ⎠
n n n n
... 8.191
1 2 3 n
Solução:
Lembre:
⎛ ⎞ ⎛ ⎞ ⎛ ⎞ ⎛ ⎞
+ + + + =⎜ ⎟ ⎜ ⎟ ⎜ ⎟ ⎜ ⎟
⎝ ⎠ ⎝ ⎠ ⎝ ⎠ ⎝ ⎠
nn n n n
... 2
0 1 2 n
Daí:
⎛ ⎞ ⎛ ⎞ ⎛ ⎞
+ + + =⎜ ⎟ ⎜ ⎟ ⎜ ⎟
⎝ ⎠ ⎝ ⎠ ⎝ ⎠
n n n
... 8.191
1 2 n
Agora: ⎛ ⎞
= ⋅⎜ ⎟
⎝ ⎠
− =
=
= → =
n
n
n
n 13
n
2 – 8 191
0
2 1 8.191
2 8.192
2 2 n 13
Resposta: 13
⎛ ⎞
− ⎜ ⎟
⎝ ⎠
n n
2
0
14444244443
Professor: Fabrício MaiaMatemática
8
10 O número total de pares (x, y) que satisfazem a equação (x2
+ y2
– 1)2
+ (xy)2
= 0 é:
A) infinito B) 0 C) 1 D) 2 E) 4
Solução:
∈
+ = ⇔ = =
1
n n
Se a,b e n é par, então :
a b 0 a b 0
Temos: (x2
+ y2
– 1)2
+ (xy)2
= 0
Daí:
⎧ + − =
⎨
= → = =⎩
= → = → = ±
= → = → = ±
− −
2 2
2
2
x y 1 0
xy 0 x 0 ou y 0
se x 0 y 1 y 1
se y 0 x 1 x 1
pares : (0,1),(0, 1),(1,0),( 1,0)
Resposta: E
11 A parábola de equação y = x2
– 6 tem vértice M e corta o eixo x nos pontos A e B. Qual a área do triângulo
ABM?
A) 1 B) 6 C) 6 D) 6 6 E) 12 6
Solução:
Lembre: 2
f(x) ax bx c, com a 0= + + ≠
Coordenadas do vértice: = −
Δ
= − =
v
v v v
b
x
2a
y ou y f(x )
4a
Temos:
– Coordenadas do vértice:
y = x2
– 6
−
= → =
⋅
= − → = −
v v
2
v v
0
x x 0
2 1
y 0 6 y 6
Então: M(0; –6)
– Pontos de interseção com o eixo x:
2
x 6 x 6 ou x 6
A( 6; 0) e B( 6, 0)
− =→ = = −
−
Logo, a área do ΔABM é dada por:
Área =
−
= =
−
ABM
ABM
1
|D |
2
0 6 1
|D | 6 0 1 12 6
6 0 1
Área:
1
12 6 6 6 u.a
2
⋅ =
Resposta: D
12 A distância do vértice da parábola y = (x – 2)(x – 6) à reta = +
4
y x 5 é :
3
72
A)
25
29
B)
25
C) 43
43
D)
25
43
E)
5
Professor: Fabrício Maia
Matemática
9
ax + by + c = 0
P(x , y )o or
Solução:
I) = + + ≠2
f(x) ax bx c, com a 0
Coordenadas do vértice: +⎧
⎪ =
⎨
⎪ =⎩
1 2
v
v v
x x
x
2
y f(x )
II) Distância de um ponto a uma reta.
+ +
=
+
0 0
p,r
2 2
| ax by c |
d
a b
Temos: = − ⋅ − →
+
= =
= = − ⋅ − = ⋅ − = −
v
v
y (x 2) (x 6) parábola
raízes : 2 e 6
2 6
x 4
2
y f(4) (4 2) (4 6) 2 ( 2) 4
Distância do vértice à reta:
(4, – 4)
4x – 3y + 15 = 0
d = ???
⋅ − ⋅ − +
=
+ −
=
2 2
| 4 4 3 ( 4) 15 |
d
4 ( 3)
43
d
5
Resposta: E
13 Resolvendo a inequação log1/2(2x + 1) > log1/2(– 3x + 4), obtemos:
− < <
1 4
A) x
2 3
< <
4
B) 0 x
3
<
3
C) x
5
− < <
1 3
D) x
2 5
< <
3 4
E) x
5 3
Solução:
Lembre: > → <
< <
x y
a alog log x y
(0 base 1)
Temos: + > − +1 1
2 2
log (2x 1) log ( 3x 4)
Então:
⎧
+ < − + → < → <⎪
⎪⎪
+ > → > −⎨
⎪
⎪− + > → <
⎪⎩
3
2x 1 3x 4 5x 3 x
5
1
2x 1 0(C.E) x
2
4
3x 4 0 (C.E) x
3
Professor: Fabrício MaiaMatemática
10
Interseção
Resposta: D
14 Se o número complexo z = 1 – i é uma das raízes da equação x10
– a = 0, o valor de a é:
A) 16 B) 32 C) 64 D) – 16i E) – 32i
Solução:
Temos: x10
= a, se z é raiz, então z10
= a.
Daí: −
= −
= −
= − → = −
10
2 5
5
5
a = (1 i)
a [(1 i) ]
a ( 2i)
a 32i a 32i
Resposta: E
15 A reta y = ax + 1 intercepta a curva x2
+ 4y2
= 1 somente num ponto. Calcule 8a2
.
Solução:
Temos:
⎧
⎨
+ =⎩
2 2
y = ax + 1
x 4y 1
Condição:
Δ = 0 (única solução)
Daí: 2 2
2 2
2
2
(8a) 4 (1 4a ) 3 0
64a 12 48a 0
16a 12
8a 6
− ⋅ + ⋅ =
− − =
=
=
Resposta: 6
16 A condição para que o trinômio mx2
+ (m + 1)x + 1 seja sempre positivo, qualquer que seja x, é que:
A) m > 0
B) (m + 1)2
+ 4m < 0
C) − ≤2
(m 1) 0
D) ≠ >m 1,m 0
E) não há valores de m tais que o trinômio proposto, qualquer que seja x, se torne sempre positivo.
Solução:
Devemos ter:
1ª condição: a > 0 → m > 0
2ª condição: Δ < → + − ⋅ ⋅ < →
→ + + − < → − + < →
→ − < − ≥ ∀ ∈ 1
2
2 2
2 2
0 (m 1) 4 m 1 0
m 2m 1 4m 0 m 2m 1 0
(m 1) 0(absurdo, pois,(m 1) 0, m )
Resposta: E
3
5
(I)
(II)
4
3
(III)
(I) Δ (II) Δ (III)
1
2
− 3
5
−1
2
+ + + + + +
{Δ <
>
0
a 0
Substituindo (I) em (II): + + =
+ + + − =
+ + + =
2 2
2 2 2
2 2
x 4(ax 1) 1
x 4a x 8ax 4 1 0
x (1 4a ) 8ax 3 0 (Equação do 2o
grau)
Professor: Fabrício Maia
Matemática
11
17 Sejam A = {1, 2, 3} e f: A → A definida por f(1) = 3, f(2) = 1 e f(3) = 2. O conjunto-solução de f[f(x)] = 3, é:
A) {1} B) {2} C) {3} D) {1, 2, 3} E) vazio
Solução:
Temos:
se x 1 f(f(1)) f(3) 2 f(f(1)) 2(n.s)
se x 2 f(f(2)) f(1) 3 f(f(2)) 3(OK)
se x 3 f(f(3)) f(2) 1 f(f(3)) 1(n.s)
S {2}
= → = = → =
= → = = → =
= → = = → =
=
Resposta: B
18 Seja S a soma, em radianos, das raízes da equação 1 + cos x + cos 2x + cos 3x = 0, x ∈[0, π]. Calcule
π
6S
.
Solução:
Fórmula de Werner:
+ −⎛ ⎞ ⎛ ⎞
+ = ⎜ ⎟ ⎜ ⎟
⎝ ⎠ ⎝ ⎠
p q p q
cosp cosq 2cos cos
2 2
Temos:
⎧ ⎛ ⎞ ⎛ ⎞
+ = + = ⋅⎜ ⎟ ⎜ ⎟⎪⎪ ⎝ ⎠ ⎝ ⎠
⎨
⎛ ⎞ ⎛ ⎞⎪ + = ⎜ ⎟ ⎜ ⎟⎪ ⎝ ⎠ ⎝ ⎠⎩
x x
1 cosx cos0 cosx 2cos cos
2 2
5x x
cos2x cos3x 2cos cos
2 2
Então:
Daí:
π
= → = + π → = π + π
π π π
= → = + π → = +
π
= → = = π
π π
= → = π
x x
cos 0 k x 2k
2 2 2
ou
3x 3x 2k
cos 0 k x
2 2 2 3 3
ou
cosx 0 x k
2
se k 0 x , ou
3 2
se k = 1, 2, 3, ... → raízes já encontradas ou fora do intervalo dado.
Raízes: π,
π π
,
2 3
Soma das raízes =
π π π + π + π
= π + + =
π
=
6 3 2
S
2 3 6
11
S
6
=
π
6S
Logo : 11
f− = ∀ ∈
Importante :
cos( x) cos(x), x D
Resposta: 11
1
2
3
1
2
3
A A
⎛ ⎞ ⎛ ⎞ ⎛ ⎞ ⎛ ⎞
+ =⎜ ⎟ ⎜ ⎟ ⎜ ⎟ ⎜ ⎟
⎝ ⎠ ⎝ ⎠ ⎝ ⎠ ⎝ ⎠
⎡ ⎤⎛ ⎞ ⎛ ⎞ ⎛ ⎞
⋅ + =⎜ ⎟ ⎜ ⎟ ⎜ ⎟⎢ ⎥
⎝ ⎠ ⎝ ⎠ ⎝ ⎠⎣ ⎦
⎛ ⎞ ⎛ ⎞
⋅ ⋅ − =⎜ ⎟ ⎜ ⎟
⎝ ⎠ ⎝ ⎠
⎛ ⎞ ⎛ ⎞
=⎜ ⎟ ⎜ ⎟
⎝ ⎠ ⎝ ⎠
x x 5x x
2cos cos 2cos cos 0
2 2 2 2
x x 5x
2cos cos cos 0
2 2 2
x 3x
2cos 2 cos cos( x) 0
2 2
x 3x
4cos cos cos(x) 0
2 2
Professor: Fabrício MaiaMatemática
12
19 A função f(x) =
−
−
3 x(1 x)
x
2 4
é crescente, para todo x pertencente a:
25
A) ,
16
⎡ ⎞
− +∞⎟⎢
⎣ ⎠
25
B) ,
4
⎡ ⎞
− +∞⎟⎢
⎣ ⎠
5
C) ,
2
⎡ ⎞
− +∞⎟⎢
⎣ ⎠
5
D) ,
4
⎡ ⎞
−∞ ⎟⎢
⎣ ⎠
1E)
Solução:
Temos:
2
2
2
2
3 x – x
f(x) x
2 4
6x x x
f(x)
4
x 5x
f(x)
4
1 5
f(x) x x
4 4
= −
− +
=
+
=
= +
Parábola
Resposta: C
20 Se p e q são raízes não-nulas de x2
+ 5px – 8q = 0, calcule p + q.
Solução:
Girard
{ + = −
⋅ = −
1 2
1 2
x x 5p
x x 8q
Daí:
{p q 5q
p q 8q
+ = −
⋅ = −
2ª equação
p q 8q⋅ = − (como q é diferente de zero)
Temos:
= −
+ = −
+ = − ⋅ −
+ =
p 8
Logo :
p q 5p
p q 5 ( 8)
p q 40
Resposta: 40
21 Quantos valores inteiros satisfazem a inequação (2x – 7) (x – 1) ≤ 0.
A) zero B) 1 C) 2 D) 3 E) 4
crescente
−
= = −
⋅
v
5
54x
1 22
4
f ∀ ≥ −
5
é crescente x
2
Professor: Fabrício Maia
Matemática
13
Solução:
Inteiros: 1, 2, 3
Resposta: D
22 Sobre a equação 1.983x2
– 1.984x – 1.985 = 0, a afirmativa correta é:
A) não tem raízes. D) tem duas raízes positivas.
B) tem duas raízes simétricas. E) tem duas raízes negativas.
C) tem duas raízes reais distintas.
Solução:
Temos: Δ = −
Δ = − − ⋅ −
Δ = + ⋅
2
2
2
B 4ac
( 1984) 4.1983 ( 1985)
1984 4.1983 1985
Δ > →
Então :
0
Resposta: C
23 Seja f uma função real tal que f (x + 1) = (f (x))2
e f (0) = 10. Então f (4) é igual a:
A) 1016
B) 100 C) 10258
D) 101 E) 121
Solução:
Temos que:
Resposta: A
24 Se o domínio da função f, definida por f(x) = 1 – 2x, é o intervalo ]–3, 2], então Imf é:
A) ]–7, 3] B) [–3, 7[ C) ]–3, 7] D) [–3, 5[ E) ]–3, 3]
Solução:
Temos:
1 y
y 1 2x x
2
−
= − → =
Veja: x ] 3,2]∈ −
Então: 3 x 2− < ≤
Agora :
1 y
3 2
2
6 1 y 4
7 y 3
7 y 3
ou
3 y 7 y [ 3,7[
−
− < ≤
− < − ≤
− < − ≤
> ≥ −
− ≤ < → ∈ −
Resposta: B
– – –
+ +
– – –
7
2
+ + +
+ + +
1
+ +
7
2
1
– –
produto
Estudo do sinal
raízes reais e distintas
2
2 2
2 4
2 8
2 16
f(x + 1) = [f(x)]
se x 0 f(1) [f(0)] f(1) 10
se x 1 f(2) [f(1)] f(2) 10
se x 2 f(3) [f(2)] f(3) 10
se x 3 f(4) [f(3)] f(4) 10
= → = → =
= → = → =
= → = → =
= → = → =
Professor: Fabrício MaiaMatemática
14
25 Se f(2x + 3) = 4x2
+ 6x + 1, x ,∀ ∈ 1 então f(1 – x) vale:
A) 2 – x2
B) 2 + x2
C) x2
+ 2x – 4 D) 3x2
– 2x + 4 E) x2
+ x – 1
Solução:
Tomando:
k 3
2x 3 k x
2
−
+ = → =
Então:
− −⎛ ⎞ ⎛ ⎞
= + +⎜ ⎟ ⎜ ⎟
⎝ ⎠ ⎝ ⎠
= − + − +
= − +
2
2
2
k 3 k 3
f(k) 4 6 1
2 2
f(k) (k 3) 3(k 3) 1
f(k) k 3k 1
Agora:
− = − − − +
− = + −
− = + −
2
2
2
f(1 k) (1 k) 3(1 k) 1
f(1 k) k k 1
Portanto :
f(1 x) x x 1
Resposta: E
26 A distância do centro da circunferência x2
+ y2
– 6x – 8y + 21 = 0 à bissetriz do Iº e IIIº quadrantes, vale:
A) 5 B) 2 C) 3 D)
3
2
E)
2
2
Solução:
Circunferência
x2
+ y2
– 2ax – 2by + a2
+ b2
– R2
= 0
Centro (a,b)
Daí:
– 2a = – 6 → a = 3
– 2b = – 8 → b = 4
Bissetriz dos quadrantes ímpares
2 2
|1 3 1 4 0 | 1 2
distância
221 ( 1)
⋅ − ⋅ +
= = =
+ −
Resposta: E
27 A reta y =
3
x
3
é tangente a uma circunferência do centro (2, 0). O raio dessa circunferência é:
A) 3 B) 2 C) 3 D) 1 E) 0,5
x y 0− =
C(3,4)
y x=
Professor: Fabrício Maia
Matemática
15
Solução:
distância de um ponto a uma reta.
⋅ − ⋅ +
= =
+ −
=
=
2 2
| 3 2 3 0 0 | 2 3
R
12( 3) ( 3)
2 3
R
2 3
R 1
Resposta: D
28 Se S = 1! + 2! + 3! + ... + 89!, então o dígito das unidades de S é:
A) 1 B) 3 C) 5 D) 7 E) 9
Solução:
Veja:
1! = 1; 2! = 2; 3! = 6; 4! = 24
A partir de 5!, os resultados serão múltiplos de 10.
Então:
S = 1 + 2 + 6 + 24 + 5! + ... + 89!
33 múltiplo de 10
S = 3 + 30 + 10α’
múltiplo de 10
S = 3 + 10α → dígito das unidades é 3.
Resposta: B
29 O sistema linear de equações nas incógnitas x e y {kx 2y 1
2x y m
+ = −
− = é impossível se, e somente se:
A) k = – 4 e m
1
2
≠ B)
1
k 4 e m
2
≠ − = C)
1
k 4 e m
2
≠ − ≠ D) = −k 4 E) = − =
1
k 4 e m
2
Solução:
Sistema {kx 2y 1
4x 2y 2m
+ = −
− =
Somando: (k 4)x 2m 1+ = −
Impossível
+ = → = −⎧
⎪
⎨ − ≠ → ≠⎪⎩
k 4 0 k 4
1
2m 1 0 m
2
Resposta: A
30 Em um triângulo retângulo OAB, retângulo em O, com OA = a e OB = b, são dados pontos P em OA e Q em OB
de tal maneira que AP = PQ = QB = x. Nessas condições o valor de x é:
A) ab a b− −
B) a b 2ab+ −
C) 2 2
a b+
D) a b 2ab+ +
E) ab a b+ +
(2,0)
3x 3y 0− =
1442443144424443
14243
Professor: Fabrício MaiaMatemática
16
Solução:
Pitágoras
= − + −
= − + + − +
− + + + =
Δ = − + − ⋅ +
Δ = + − +
Δ =
2 2 2
2 2 2 2 2
2 2 2
2 2 2
2 2 2
x (a x) (b x)
x a 2ax x b 2bx x
x 2(a b)x a b 0
[ 2(a b)] 4 1(a b )
4(a b) 4(a b )
8ab
Daí:
+ ± + ±
= =
= + +
= + −
2 2(a b) 8ab 2(a b) 2 2ab
x
2 2
x a b 2ab (absurdo, veja figura) ou
x a b 2ab
Resposta: B
31 Num triângulo retângulo de catetos 1 e 3cm , a altura relativa à hipotenusa mede, em cm:
A) 2 B) 3 C) 3 D)
3
2
E)
2
2
Solução:
Temos:
Relações métricas
I) 2 2 2 2
a 1 ( 3) a 4 a 2= + → = → =
II)
3
1 3 a h 3 2h h
2
⋅ = ⋅ → = → =
Resposta: D
32 Sendo f(x) = 100x + 3, o valor de
−
−
−
−
8 3
8 3
f(10 ) f(10 )
10 10
é:
A) 104
B) 102
C) 10 D) 10–5
E) 10–11
Solução:
Saiba:
Se f(x) = ax + b, com a ≠ 0, então:
f(s) f(v)
a, com s v.
s v
−
= ≠
−
Assim,
8 3
8 3
f(10 ) f(10 )
100
10 10
−
−
−
=
−
(coeficiente angular)
Resposta: B
33 Se um polígono convexo de n lados tem 54 diagonais, então n é:
A) 8 B) 9 C) 10 D) 11 E) 12
Solução:
Lembre: Fórmula do número de diagonais
n(n 3)
d
2
−
= .
Então:
Resposta: E
B
o
A
P
Q
b – x
a – x
x
x
B C
h
A
a
1 3
2
n(n – 3)
54
2
n – 3n –108 0
n 12 ou n –9 (n.s)
=
=
= =
Professor: Fabrício Maia
Matemática
17
34 O polígono convexo cuja soma dos ângulos internos mede 1440º tem, exatamente:
A) 15 diagonais
B) 20 diagonais
C) 25 diagonais
D) 30 diagonais
E) 35 diagonais
Solução:
Lembre: Soma dos ângulos internos Si = (n – 2) ⋅ 180º
Então: 1440º = (n 2) 180º
1400º
n 2
180º
8 n 2
n 10
− ⋅
= −
= −
=
Portanto:
n(n 3) 10 7
d
2 2
d 35
− ⋅
= =
=
Resposta: E
35 Na figura, ABCD é um quadrado e BCE é um triângulo eqüilátero. A medida do ângulo AEB, em graus, é:
A) 30 D) 75
B) 49 E) 90
C) 60
Solução:
Figura:
ΔBCE é eqüilátero → α = 60º
ABCD é um quadrado → 30ºθ =
Veja: BC BE≡ (lado do quadrado = lado do ΔBCE)
Daí: ΔABE é isósceles
Resposta: D
36 Na figura abaixo, EFG é um triângulo retângulo, EF 2cm= , EG 6cm e EP PQ QG.= = = Então α + β + θ é igual a:
A)
3
π
B)
7
18
π
C)
4
9
π
D)
2
π
E
C
D
B
A
E
C
D
B
A
α
θ
x
2x 180º
30º 2x 180º
x 75º
θ + =
+ =
=
A
E
x
θ
x
B
α β θ
F
P Q GE
Professor: Fabrício MaiaMatemática
18
Solução:
2
EPF tg tg 1 45º
2
2 1
EQF tg tg
4 2
2 1
EGF tg tg
6 3
Δ → α = → α = →α =
Δ → β = → β =
Δ → θ = → θ =
Sabemos:
tg tg
tg( )
1 tg tg
β + θ
β + θ =
− β⋅ θ
Então:
1 1 5
2 3 6tg( ) 1
1 1 5
1
2 3 6
tg( ) 1 45º
+
β + θ = = =
− ⋅
β + θ = → β + θ =
Portanto: 90ºα +β+ θ =
Resposta: D
37 A área compreendida entre as retas 4y = x – 2, y = 2x – 4 e x = 0 é igual a:
A) 3,0 u.a B) 3,5 u.a C) 4,0 u.a D) 4,5 u.a E) 6,0 u.a
Solução:
Temos:
Gráfico:
Logo:
Área =
1 7
7 u.a
2 2
⋅ =
Resposta: B
F
E
α β θ
P Q G2 2 2
2
x 2
y
4
y 2x 4
x 0 (eixo y)
−⎧
=⎪
⎪
= −⎨
⎪ =
⎪
⎩
retas
1
A(0, );B(0, 4) e C(2,0)
2
−
−
A C
B
y
x
y 2x 4= −
x 2
y
4
−
=
ABC
1
0 1
2
D 0 4 1 1 8 7
2 0 1
−
= − = − + =
Professor: Fabrício Maia
Matemática
19
38 A razão de uma progressão geométrica, cujos termos são os três lados de um triângulo retângulo é:
1 5
A)
2
+ 1 2
B)
5
+ 1 3
C)
2
+ 1 2
D)
3
+
Solução:
x
P.G. ;x;xq
q
⎛ ⎞
⎜ ⎟
⎝ ⎠
Pitágoras
Dividindo ambos os membros por x2
2
2
4 2
2 2
1
q 1
q
q q 1 0
1 5 1 5
q q
2 2
= +
− − =
± +
= → =
Portanto:
1 5
q
2
+
=
Atenção!!! q é positivo.
Resposta: A
39 Sejam a e b números reais. Se a > b > 0, a2
– b2
= 4 e log2(a + b) – log3(a – b) = 2, então a2
+ b2
é igual a:
A) 13/2 B) 15/2 C) 17/2 D) 19/2
Solução:
Fazendo: x
2
y
3
log (a b) x a b 2
log (a b) y a b 3
+ = → + =
− = → − =
Sistema:
x y
2 3 4
x y 2 x 2 y
⎧ ⋅ =
⎨
− = → = +⎩
Substituindo: x y
2 y y
2 y y
y
y
2 3 4
2 3 4
2 2 3 4
4 6 4
6 1 y 0 x 2
+
⋅ =
⋅ =
⋅ ⋅ =
⋅ =
= → = → =
Assim:
{a b 4
a b 1
+ =
− =
Resolvendo: = =
5 3
a e b
2 2
Logo: 2 2 34 17
a b
4 2
+ = =
Resposta: C
C
A x B
xqx
q
2
2 2
2
2 2 2
2
x
(xq) x
q
x
x q x
q
⎛ ⎞
= + ⎜ ⎟
⎝ ⎠
= +
Professor: Fabrício MaiaMatemática
20
40 Se x1 e x2 são as raízes da equação x x2log 3 log (3x)
3 x ,= então 9(x1 + x2) é igual a:
A) 22 B) 24 C) 26 D) 28
Solução:
Lembre: alog b
x
a
I) a b
II)Se log b x a b
=
= → =
Temos: x x2log 3 log 3x
3 x=
Então: x2log 3
3 3x=
Tomando: k
3 x
1
log x k log 3 e x 3
k
= → = =
Substituindo:
1
2.
kk
2
k 1k
3 3 3
3 3 +
= ⋅
=
Comparando:
2
2
1
1 2
2
k 1
k
k k 2 0
1
k 2 x 3 x
9
ou
k 1 x 3 3
Logo :
1
9(x x ) 9 3 1 27 28
9
−
+ =
+ − =
= − → = → =
= → = →=
⎛ ⎞
+ = + = + =⎜ ⎟
⎝ ⎠
Resposta: D
41 O número de raízes de equação
3
2
+ cos x = 0 é:
A) 0 B) 1 C) 2 D) 3 E) maior que 3
Solução:
Sabemos: 1 cosx 1, x.− ≤ ≤ ∀
Temos:
Resposta: A
42 O número de raízes da equação tg2
x – sen2
x = 0, 0 x 2 ,≤ < π é:
A) 0 B) 1 C) 2 D) 3 E) maior que 3
Solução:
Temos:
Daí: senx 0
x 0 ou x
=
= = π
Resposta: C
3
cos x 0
2
3
cosx
2
cosx 1,5
+ =
= −
= − (absurdo, pois o mínimo de cos x é – 1)
2
2
2
2 2 2
2 2
2 2
4
sen x
sen x 0
cos x
sen x sen xcos x 0
sen x(1 cos x) 0
sen x sen x 0
sen x 0
− =
− =
− =
⋅ =
=
Professor: Fabrício Maia
Matemática
21
43 Determine n, sabendo que
n
p
p 0
n
2 729
p=
⎛ ⎞
⋅ =⎜ ⎟
⎝ ⎠
∑
Solução:
n
n n p p
p 0
n
(a b) a b
p
−
=
⎛ ⎞
+ = ⋅ ⋅⎜ ⎟
⎝ ⎠
∑ (binômio de Newton)
Veja:
n
n p p n n
p 0
n
1 2 (1 2) 3
p
−
=
⎛ ⎞
⋅ ⋅ = + =⎜ ⎟
⎝ ⎠
∑
Então: n
3 729 n 6= → =
Resposta: 6
44 O domínio real da função f(x) = 2senx 1− para 0 x 2 ,≤ < π é:
5
A) x
6 6
π π
≤ ≤
5
B) 0 x ou x 2
6 6
π π
≤ ≤ ≤ < π C) 0 x≤ < π
2
D) x
3 3
π π
≤ ≤
Solução:
Condição:
2senx 1 0
1
senx
2
− ≥
≥
Daí:
5
x
6 6
π π
≤ ≤
Resposta: A
45 Seja M um conjunto de 20 elementos. O número de subconjuntos de M que contêm exatamente 18 elementos é:
A) 360 B) 190 C) 180 D) 120 E) 18
Solução:
Temos: M = {a1, a2, a3 ..., a20}.
A ordem dos elementos não altera um conjunto.
Daí: nº de subconjuntos com 18 elementos = C20, 18
Resposta: B
46 Se Cn, 2 + 2.An,2 + 100 = A2n, 2, então n é igual a:
A) 24 B) 8 C) 6 D) 10 E)
25
3
−
Solução:
Temos: n,2
n,2
2n,2
n! n (n 1)
C
(n 2)!2! 2
n!
A n (n 1)
(n 2)
(2n)!
A (2n) (2n 1)
(2n 2)!
⋅ −
= =
−
= = ⋅ −
−
= = ⋅ −
−
Então:
Resposta: B
2 2 2
2
n(n 1)
2 n(n 1) 100 (2n)(2n 1)
2
n n 4n 4n 200 8n 4n
3n n 200 0
25
n 8 ou n (n.s)
3
−
+ ⋅ − + = −
− + − + = −
+ − =
= = −
150º
c
30º
s
1
2
Professor: Fabrício MaiaMatemática
22
47 Deseja-se acondicionar em um certo número de caixas, 1590 bolinhas brancas, 1060 amarelas e 583 azuis, de
modo que cada caixa contenha bolinhas de todas as cores. Calcule o número máximo de caixas de modo que
qualquer dessas caixas contenha, para cada cor, quantidades iguais de bolinhas.
Solução:
x → número de caixas.
p → quantidade de bolas brancas em cada caixa.
q → quantidade de bolas amarelas em cada caixa.
r → quantidade de bolas azuis em cada caixa.
Temos:
Veja: x m.d.c. (1590,1060,583)
x 53
=
=
* MDC (1590, 1060) = 530 * MDC (530, 583) = 53
Resposta: 53
48 Sejam N o conjunto dos números inteiros positivos e E = {(x,y) ∈ N2
; x4
y4
– 10x2
y2
+ 9 = 0}. Determine o número
de elementos de E.
Solução:
Temos: 4 4 2 2
2 2 2 2 2
x y 10x y 9 0
(x y ) 10x y 9 0
− + =
− + =
Fazendo: 2 2
x y k=
Equação: 2
2 2 2
k 10k 9 0
k 1 x y 1 (xy) 1
− + =
= → = → =
ou 2 2 2
k 9 x y 9 (xy) 9= → = → =
Como x e y são inteiros positivos, tem-se:
xy 1 (1,1)
ou
xy 3 (1,3) ou (3,1)
E {(1,1),(1,3),(3,1)}
= →
= →
=
Resposta: 3
49 Considere a função real definida por
2x 3 3
f(x) ,x
1 1 2x
3 2
+
= ≠ −
+
.
Então o valor da soma 1 f(1) 2 f(2) 3 f(3) ... 20 f(20)⋅ + ⋅ + ⋅ + + ⋅ é:
A) 120 B) 600 C) 210 D) 620 E) 1.260
1590
p
x
1060
q
x
583
r
x
=
=
=
1060
1 2
530
0
1590
530
583 530
1 10
53
0
Professor: Fabrício Maia
Matemática
23
Solução:
Temos:
2x 3 6
f(x) 2x 3
2x 3 2x 3
6
+
= = + ⋅
+ +
Então: f(x) 6=
Agora:
Soma:
Soma:
Soma:
Resposta: E
50 Sejam x e y números reais satisfazendo as equações logy x + logx y = 2 e x2
y + y2
= 12x. Determine o valor do
produto xy.
Solução:
Tomando: = → =y x
1
log x m log y
m
Daí: + =
− + =
= → =
2
1
m 2
m
m 2m 1 0
m 1 x y
Substituindo na 2ª equação
Resposta: 9
51 Os conjuntos A e B possuem 3 e 4 elementos, respectivamente. Quantas funções de A em B têm o conjunto
imagem igual a B?
A) Nenhuma B) 34
C) 43
D) 3! E) 4!
Solução:
Veja:
I) Numa função de A em B devemos ter todos os elementos de A associados a um único valor em B.
II) Se o conjunto imagem é o próprio B, então existe um elemento em A com duas imagens, pois todos os
elementos de A estão associados.
Portanto, não existem funções de A em B sobrejetoras.
Resposta: A
52 As funções injetoras de A = {1, 2, 3, 4} em B = {5, 6, 7, 8, 9, 0} são em número de:
A) 720 B) 360 C) 15 D) 24 E) 30
Solução:
Lembre:
Se f é injetora, então:
1 2 1 2x x f(x ) f(x )≠ → ≠
Daí:
Pelo princípio fundamental da contagem, tem-se 6.5.4.3 = 360 funções injetoras
Resposta: B
Soma 1.260=
1 6 2 6 3 6 ... 20 6
(1 2 3 ... 20) 6
(1 20) 20
6 21 10 6
2
⋅ + ⋅ + ⋅ + + ⋅
+ + + + ⋅
+ ⋅
⋅ = ⋅ ⋅ →
2 2
2 2
3 2
2
x y y 12x
x x x 12x
x x 12x 0
x(x x 12) 0
x 0 (n.s) ou x 4(n.s)
ou
x 3 y 3
+ =
⋅ + =
+ − =
+ − =
= = −
= → =
f(1): 6 possibilidades em B
f(2): 5 possibilidades em B
f(3): 4 possibilidades em B
f(4): 3 possibilidades em B
1
2
3
4
A
Professor: Fabrício MaiaMatemática
24
53 Para ser aprovado numa disciplina, um aluno precisa ter média maior ou igual a 50, obtida num conjunto de 5
provas, sendo quatro parciais, com peso 1 (um) cada, e uma prova-exame, com peso 2 (dois). Um certo aluno
obteve em matemática, nas quatro provas parciais, notas iguais a 30, 60, 50 e 70. Esse aluno, para ser aprovado
nessa disciplina, deverá obter, na prova-exame, nota mínima igual a:
A) 20 B) 35 C) 40 D) 45 E) 50
Solução:
Média ponderada =
1 30 1 60 1 50 1 70 2 x 210 2x
6 6
⋅ + ⋅ + ⋅ + ⋅ + ⋅ +
=
Temos:
min
210 2x
50
6
210 2x 300
2x 90
x 45 x 45
+
≥
+ ≥
≥
≥ → =
Resposta: D
54 O resto da divisão do inteiro n por 12 é igual a 7. O resto da divisão n por 4 é:
A) 0 B) 1 C) 2 D) 3 E) 4
Solução:
Temos:
Daí: n 12q 7
n 12q 4 3
n 4(3q 1) 3
n 4q' 3, onde q' 3q 1
= +
= + +
= + +
= + = +
Veja:
Resposta: D
55 Qual dos cinco números relacionados abaixo não é um divisor de 1015
?
A) 25 B) 50 C) 64 D) 75 E) 250
Solução:
Temos: 15 15 15 15
10 (2 5) 2 5= ⋅ = ⋅
Veja:
(A) 25 = 52
divide 1015
(OK)
(B) 50 = 2⋅ 52
divide 1015
(OK)
(C) 64 = 26
divide 1015
(OK)
(D) 75 = 3⋅ 52
não divide 1015
(problema fator 3)
(B) 250 = 2⋅ 53
divide 1015
(OK)
Resposta: D
56 A fração geratriz de 3,74151515... é:
37.415
A)
10.000
3.741.515
B)
10.000
37.041
C)
9.900
37.041
D)
9.000
370.415
E)
99.000
Solução:
Temos:
37.415 374 37.041
3,7415
9.900 9.900
−
= =
Resposta: C
n
7 q
12
n
3
4
q’
Professor: Fabrício Maia
Matemática
25
57 Se A e B são conjuntos, A – (A – B) é igual a:
A) A B) B C) A – B D) A ∪ B E) A ∩ B
Solução: Lembre: A B {x / x A e x B}− = ∈ ∉
É fácil ver:
A – (A – B) = A ∩ B
Resposta: E
58 O retângulo abaixo de dimensões a e b está decomposto em quadrados. Qual o valor da razão
a
?
b
5
A)
3
2
B)
3
C) 2
3
D)
2
1
E)
2
Solução:
Sendo x a medida do lado do menor quadrado, os outros quadrados terão seus lados com as medidas indicadas
na figura.
Assim: a = 5x e b = 3x
Portanto,
a 5
b 3
=
Resposta: A
59 A equação x4
+ ax3
+ bx2
+ cx + d = 0, de coeficientes reais, admite as raízes 2 – i e 3 + 2i. Então d é:
A) 75 B) 65 C) 25 D) 15 E) 10
Solução:
Sabemos que:
Se os coeficientes de um polinômio P(x) são reais, então: a + bi raiz de P(x) → a – bi também é:
Temos:
2 – i raiz → 2 + i também é.
3 + 2i raiz → 3 – 2i também é.
Daí, aplicando Girard na equação:
Resposta: B
B
A–B
A
b
a
x
1442443
14444244443
1424314243
x x
2x
2x
2x
3x
5x
3x
1 2 3 4
2 2
x .x .x .x d
(2 i) (2 i) (3 2i) (3 2i) d
(4 i ) (9 4i ) d
5 13 d
d 65
=
− ⋅ + ⋅ + ⋅ − =
− ⋅ − =
⋅ =
=
Professor: Fabrício MaiaMatemática
26
60 O número de soluções reais da equação
2
2
2x 8x
x
x 4x
−
=
−
é:
A) 0 B) 1 C) 2 D) 3 E) 4
Solução:
Temos:
2
2
2 3 2
3 2
2
2x 8x
x
x 4x
2x 8x x 4x
x 6x 8x 0
x(x 6x 8) 0
x 0 (n. serve) denominador nulo
−
=
−
− = −
− + =
− + =
= →
ou
2
x 6x 8 0
ou
x 2
ou
x 4 (n. serve) denominador nulo
S {2}
− + =
=
= →
=
Resposta: B
61 Determine o número de soluções reais da equação 2x
= 2log x .
A) Nenhuma B) Uma C) Duas D) Três E) Infinitas
Solução:
Graficamente:
Como não existe interseção, a equação não admite soluções.
Resposta: A
62 Se n é o maior número inteiro pertencente ao domínio da função 2f(x) 1 log x= − , determine o valor de n3
+ 3n2
+ 2.
A) 2 B) 20 C) 21 D) 22 E) 32
Solução:
Domínio → campo de existência → condição de existência da função →
1
2 21 log x 0 log x 1 0 x 2 0 x 2− ≥ → ≤ → < ≤ → < ≤ → maior inteiro x = 2.
Logo, a expressão é igual a 3 2
2 3 2 2 22+ ⋅ + = .
Resposta: D
y
x
0
2y log x=
x
y 2=
Professor: Fabrício Maia
Matemática
27
63 Dado x 1≠ e positivo, calcule o valor de
1
Lnxx
e
.
A) 0 B) 1 C) 2 D) 3 E) 4
Solução:
Sabemos que:
alog b
a
b
e
I) a b
1
II) log b
log a
III)Lx Lnx log x
=
=
= =
Então, a expressão dada
1
Lnxx
e
é igual a:
xlog e
x e
Exp. 1
e e
= = = .
Resposta: B
64 Prove que
10 10 10log c log a log b
a b c
1
b c a
⎛ ⎞ ⎛ ⎞ ⎛ ⎞
⋅ ⋅ =⎜ ⎟ ⎜ ⎟ ⎜ ⎟
⎝ ⎠ ⎝ ⎠ ⎝ ⎠
.
Prova:
Tomemos:
x
10
y
10
z
10
log a x 10 a
log b y 10 b
log c z 10 c
= → =
= → =
= → =
Então:
c.q.p
65 Determine o produto das soluções reais da equação 3log x 3
9 x x⋅ = .
A) 4 B) 8 C) 25 D) 27 E) 90
Solução:
Tomemos: k
3log x k 3 x= → =
Assim:
2
k k k 3
k 2 3k 2
2
9 (3 ) (3 )
3 3 k 2 3k
k 3k 2 0
k 1 x 3
ou
k 2 x 9
+
⋅ =
= → + =
→ − + =
= → =
= → =
Portanto, o produto das soluções é 27.
Resposta: D
⎛ ⎞ ⎛ ⎞ ⎛ ⎞
= ⋅ ⋅⎜ ⎟ ⎜ ⎟ ⎜ ⎟
⎝ ⎠ ⎝ ⎠ ⎝ ⎠
z x yx y z
y z x
10 10 10
10 10 10
= ⋅ ⋅
xz xy yz
yz xz xy
10 10 10
10 10 10
1º membro
1º membro
1º membro = 1
Professor: Fabrício MaiaMatemática
28
66 Seja x tal que x
10 10log 2,log (2 1)− e x
10log (2 3)+ estão, nessa ordem, em progressão aritmética. Calcule 22x
.
A) 1 B) 4 C) 8 D) 16 E) 25
Solução:
Temos que: x x
10 10 10
x x
10 10 10
x 2 x
10 10
x 2 x
(log 2,log (2 1),log (2 3) P.A
2 log (2 1) log 2 log (2 3)
log (2 1) log 2(2 3)
(2 1) 2 (2 3)
− + →
→ ⋅ − = + + →
→ − = + →
→ − = ⋅ +
Tome: 2x
= a
Então:
a2
– 2a + 1 = 2a + 6 → a2
– 4a – 5 = 0
a = 5 → 2x
= 5 ou a = – 1 (não serve)
Portanto, 22x
= 25
Resposta: E
67 As dimensões de um paralelepípedo retângulo são proporcionais a 3,5 e 7. Sabendo que a diagonal mede
4 83cm, calcule o volume do paralelepípedo.
A) 105cm3
B) 1575cm3
C) 4725cm3
D) 6720cm3
E) 8575cm3
Solução:
Diagonal (D) → 2 2 2
2
D (3k) (5k) (7k)
4 83 83k
4 83 k 83
k 4
= + +
=
=
=
Volume (V) →
3
V 12 20 28
V 6720cm
= ⋅ ⋅
=
Resposta: D
68 Um prisma reto de altura igual a 9cm tem como base um triângulo. Sabendo que dois dos lados desse triângulo
medem 3cm e 4cm e que o ângulo formado por esses lados mede 45º, determine o volume do prisma.
3
A) 3 2cm 3
B) 9 2cm 3
C) 27 2cm 3
D) 54 2cm 3
E) 81 2cm
Solução:
Volume do prisma: (Área da base) x (altura)
Então:
3
3 4 sen45º
V 9
2
2
V 6 9 27 2cm
2
⋅ ⋅⎛ ⎞
= ⋅⎜ ⎟
⎝ ⎠
= ⋅ ⋅ =
Resposta: C
3k
5k
7k
D
9
4
3
45º
Professor: Fabrício Maia
Matemática
29
69 A aresta, a diagonal e o volume de um cubo estão, nessa ordem, em progressão geométrica. Determine a área
total desse cubo.
A) 3 B) 6 C) 9 D) 18 E) 27
Solução:
aresta = a
diagonal = a 3
volume = a3
P.G. (a, a 3 , a3
) → ( a 3 )2
= a ⋅ a3
→
3a2
= a4
→ a2
= 3
Portanto, a área total será 18u.a.
Resposta: D
70 Uma esfera de raio r é inscrita num cone equilátero com geratriz de comprimento g. Determine o valor de
g 3
r
.
A) 3 B) 6 C) 8 D) 9 E) 12
Solução:
0 = incentro, baricentro, circuncentro, ortocentro.
Veja:
I) g = 2R (geratriz)
II)
g 3 g 3
3r 6
2 r
= → =
Resposta: B
71 O raio da base de um cone circular reto mede 4cm e sua altura
25
cm
π
. Determine, em cm3
, o volume do cilindro
circular reto de maior área lateral, inscrito no cone.
A) 4 B) 10 C) 25 D) 40 E) 50
Solução:
D
a
a
a
14243
2R
0
2R
2R
2r
r
4
h
α
r
25
π
Professor: Fabrício MaiaMatemática
30
Área (lateral do cilindro) = L2 rh Aπ =
Veja:
25
h 25
tg h (4 r)
4 r 4 4
πα = = → = −
− π
Subst. h na área lateral, vem:
2
L L
25 25
A 2 r (4 r) A (4r r )
4 2
⎡ ⎤
= π − → = −⎢ ⎥π⎣ ⎦
Para que AL seja máxima, basta que r seja igual a abscissa do vértice da parábola.
Então: r = 2 →
2 325 25
h V 50cm
2 2
2= → = π ⋅ =
π π
⋅
Resposta: E
72 Determine a área (em m2
) do setor circular hachurado na figura abaixo, sabendo que o ângulo ˆABCmede rad
6
π
e o diâmetro AB mede
6
8
π
m.
A) 24 D) 54
B) 48 E) 54 3
C) 48 3
Solução:
[setor] =
2
R
6
π
ABCΔ é retângulo
R 3
cos30º
26
8
= =
π
Então: = ⋅ ⋅ → = ⋅ ⋅ →
π π
π
→ = ⋅ ⋅ → = ⋅ → =
π
2
2
2 2
6 6
2R 3 8 4R 3 64
6 R
R 3 16 3 16 [setor] 48m .
6
Resposta: B
73 Dado um cilindro de revolução de raio r e altura h. sabe-se que a média harmônica entre o raio r e a altura h
é 4 e que sua área total é 2πm2
. Mostre que o raio r satisfaz a sentença r3
– r + 2 = 0.
Solução:
B
C
A
B
C
A
30º
R
R
h
r
123
parábola
Professor: Fabrício Maia
Matemática
31
Área total
2
2
2 rh 2 r 2
rh r 1 (I)
π + π = π
+ =
Média harmônica
2rh
4
r h
2r
h (II)
r 2
=
+
=
−
Subst. (II) em (I), vem:
2
2 2
2 2 2 3 2
3 3
2r 2r
r r 1 r 1
r 2 r 2
2r r (r 2) r 2 2r r 2r r 2
r r 2 r r 2 0
⎛ ⎞
+ = → + = →⎜ ⎟− −⎝ ⎠
→ + − = − → + − = − →
→ = − → − + =
74 Seja o determinante D(x) =
1 2senx senx
.
cosx 1 2senx
− −
+
Calcule o valor de D
π⎛ ⎞
⎜ ⎟12⎝ ⎠
.
1
A)
2
2
B)
2
3
C)
2
1
D) 3
2
+
3 1
E)
2 4
+
Solução:
2
D(x) 1 2sen x senxcos x
sen(2x)
D(x) cos(2x)
2
= − +
= +
Resposta: E
75 Seja R a raiz positiva da equação x2
+ x –
3
4
= 0. Se R =
senAº cosAº
,
sen11º cos11º onde 0 < A < 90. Calcule o valor de A.
A) 30 B) 41 C) 60 D) 75 E) 80
Solução:
Temos: 2 3
x x 0
4
1 1
x R
2 2
3
x (não serve)
2
+ − =
= → =
−
=
Assim, R senAºcos11º sen11ºcos Aº
R sen(Aº 11º)
1
sen(Aº 11º)
2
= −
= −
= −
Então: Aº – 11º = 30º
Aº = 41º → A = 41
Resposta: B
sen
6
D cos
6 2
π⎛ ⎞
⎜ ⎟π π⎛ ⎞ ⎛ ⎞ ⎝ ⎠= +⎜ ⎟ ⎜ ⎟12⎝ ⎠ ⎝ ⎠
3 1
D
2 4
π⎛ ⎞
= +⎜ ⎟12⎝ ⎠
ou
Professor: Fabrício MaiaMatemática
32
76 Determine a soma das raízes da equação.
A) 0 D) 4
B) 1 E) 5
C) 2
Solução: Aplicando chió, vem:
Daí:
x 1 0 0
0 x 1 0 0
0 0 x 5
−
+ =
−
(x 1) (x 1)(x 5) 0− ⋅ + − =
x = 1, – 1 ou 5
Portanto, a soma das raízes é 5.
Resposta: E
77 Se o sistema {x my 3
mx 4y 6
+ =
+ =
tem infinitas soluções. Determine o valor de m4
– 8m2
+ 23.
A) 6 B) 7 C) 8 D) 9 E) 12
Solução:
Sejam:
r: a1x + b1y + c1 = 0
s: a2x + b2y + c2 = 0
Se r e s são coincidentes, então:
11 1
2 2 2
ba c
a b c
= =
Assim, temos:
1 m 3
m 2
m 4 6
= = → =
* retas coincidentes → infinitas soluções.
Portanto, m4
– 8m2
+ 23 = 7
Resposta: B
78 Se (xo, yo, zo) é uma solução do sistema 2
x y 2
xy z 1
+ =⎧
⎨
+ =⎩
encontre o valor de 2 2 2
o o ox y 2z .+ −
A) 0 B) 1 C) 2 D) 3 E) 4
Solução:
2 2
o o o o o o
2 2
o o o o o o
x y 2 x y 2x y 4
x y z 1 2x y 2z 2
⎧ + = → + + =
⎨
+ = →− − = −⎩
Somando: o o
2 2 2
ox y 2z 2+ − =
Resposta: C
1 1 1 1
1 x 1 1
0
1 1 x 2 1
1 1 1 x 4
=
+
−
1 1 1 1
1 x 1 1
0
1 1 x 2 1
1 1 1 x – 4
=
+
Professor: Fabrício Maia
Matemática
33
79 Considere a função real definida no conjunto dos números reais não-negativos por f(x) = x + x – 2. Determine
o número real k, tal que f(2k
) = 0.
A) 0 B) 1 C) 2 D) 3 E) 4
Solução:
Temos que:
+ − =
− = −
− ⋅ + =
− ⋅ + =
= → =
= → =
k k
k k
2k k k
2k k
k
k
2 2 2 0
2 2 2
2 4 2 4 2
2 5 2 4 0
2 1 k 0
ou
2 4 k 2 (não serve)
Veja: se k = 2 → f(2k
) = f(4) = 4 ≠ 0
Resposta: A
80 Sendo a reta y = ax + b tangente à elipse x2
+ 4y2
= 1, determine o valor de 8(b2
– a2
).
A) 0 B) 1 C) 2 D) 3 E) 4
Solução:
Substituindo a reta na equação da elipse, vem:
x2
+ 4y2
= 1
x2
+ 4(a2
x2
+ 2abx + b2
) = 1
(1 + 4a2
) x2
+ 8abx + 4b2
– 1 = 0
Como a reta é tangente, então a interseção é um único ponto.
Δ = 0 (único ponto)
Daí:
(8ab)2
– 4(1 + 4a2
) ⋅ (4b2
– 1) = 0
64a2
b2
– 16b2
+ 4 – 64a2
b2
+ 16a2
= 0
8a2
– 8b2
+ 2 = 0
8(b2
– a2
) = 2
Resposta: C
81 Determine o valor de b para o qual a reta y = x + b não intercepta os ramos da hipérbole x2
– y2
= 1.
A) 0 B) 1 C) 2 D) 3 E) 4
Solução:
Interseção → x2
– ( x + b)2
= 1
→ x2
– x2
– 2bx – b2
= 1
→ – 2bx =1+ b2
2
1 b
x
2b
+
→ =
−
Veja: para que não exista interseção, basta tomarmos b = 0.
Resposta: A
82 Determine o menor inteiro n > o, de modo que
n
3 1
i
2 2
⎛ ⎞
+⎜ ⎟⎜ ⎟
⎝ ⎠
seja real positivo.
A) 6 B) 10 C) 12 D) 16 E) 24
(x da interseção)
Professor: Fabrício MaiaMatemática
34
Solução:
Temos que:
n
n3 1
i (cos30º isen30º) cos(n 30º) isen(n
2 2
30º)
⎛ ⎞
+ = + = ⋅ +⎜ ⎟⎜ ⎟
⎝ ⎠
⋅
Então:
n 30º k 360º
n 12k
⋅ = ⋅
=
Portanto: n = 12 (menor inteiro positivo)
Resposta: C
83 Encontre o módulo do complexo 1 , tal que 2
i.=1
A) 1 B) 2 C) 3 D) 2 E) 3
Solução:
Temos: 2 2
2 2
i | | |i |
| | | 0 1i| | | | | 0 1
| | | | 1 | | 1
= → = →
→ ⋅ = + → ⋅ = + →
→ ⋅ = → =
1 1
1 1 1 1
1 1 1
Resposta: A
84 Se A , B e C são números reais, tais que 2 2
1 A Bx C
,
x(x 2x 2) x x 2x 2
+
= +
+ + + +
para todo x, x *∈ 1 , calcule o valor de
A + B + C.
Solução:
2 2
2
2 2
2
A Bx C 1
, x *
x x 2x 2 x(x 2x 2)
A(x 2x 2) (Bx C)x 1
, x *
x(x 2x 2) x(x 2x 2)
1
A
2A B 0
1
(A B)x (2A C)x 2A 1 2A C 0 B
22A 1
C 1
+
+ = ∀ ∈
+ + + +
+ + + +
= ∀ ∈
+ + + +
⎧
=⎪
+ =⎧ ⎪⎪ ⎪
+ + + + ≡ → + = → = −⎨ ⎨
⎪ ⎪=⎩ = −⎪
⎪⎩
1
1
Portanto: A + B + C = – 1
85 Determine um polinômio P(x) de grau 2 que verifique a identidade P(x + 1) ≡ x2
+ 2x + 3.
Solução:
Supondo P(x) = ax2
= bx + c, temos: P(x + 1) = a(x + 1)2
+ b(x + 1) + c = ax2
+ (2a + b)x + (a + b + c).
Então:
P(x + 1) ≡ x2
+ 2x + 3
a 1 a 1
2a b 2 b 0
a b c 3 c 2
= =⎧ ⎧
⎪ ⎪
⇔ + = ⇔ =⎨ ⎨
⎪ ⎪+ + = =⎩ ⎩
Logo, P(x) = x2
+ 2.
14243
um
14243
zero
Professor: Fabrício Maia
Matemática
35
86 Que condições devem satisfazer os números a, b e c para que o polinômio ax2
+ bx + c seja o quadrado de um
polinômio do 1º grau?
Solução:
Devemos ter ax2
+ bx + c ≡ (mx + n)2
, com m ≠ 0; portanto:
2
2
a m
b 2mn
c n
⎧ =
⎪
=⎨
⎪ =⎩
Podemos eliminar m e n e obter a relação entre a, b e c e calculando b2
.
b2
= (2mn)2
= 4m2
n2
= 4ac
Resposta: A condição é b2
= 4ac e a ≠ 0 (pois m ≠ 0)
87 Na figura abaixo indicamos 9 pontos, entre os quais não há 3 colineares, exceto os 4 que marcamos numa
mesma reta. Quantos triângulos existem com vértices nestes pontos?
Solução:
Se não houvessem 3 pontos colineares, o número de triângulos seria C9, 3. Desse número, devemos subtrair as
combinações formadas por 3 pontos escolhidos entre os 4 alinhados, isto é, C4, 3, pois essas combinações não
correspondem a triângulos. Assim, o número de triângulos que podemos formar é C9, 3 – C4, 3.
Temos:
9,3
9! 9x8x7x 6!
C
3!6!
= =
3x2x1x 6!
4,3
84
4! 4 x 3!
C
3!1!
=
= =
3
4
!x 1
=
Logo: C9,3 – C4, 3 = 84 – 4 = 80.
88 Um químico possui 10 tipos de substâncias. De quantos modos possíveis poderá associar 6 dessas substâncias se,
entre as 10, duas somente não podem ser juntadas porque produzem mistura explosiva?
Solução:
Cada mistura de 6 das 10 substâncias corresponde a uma combinação das 10 substâncias tomadas 6 a 6, uma
vez que não importa a ordem das substâncias na mistura. Assim, o total de misturas seria C10, 6 se não houvesse
problema com nenhuma mistura. Devemos, porém, subtrair desse número as combinações em que entrariam as
duas substâncias que, se misturadas, provocam explosão. As combinações em que entram essas duas substâncias
são formadas por elas duas e mais quatro substâncias escolhidas entre as outras oito substâncias (excluímos
aquelas duas). O número de modos de escolher 4 substâncias em 8 é C8, 4.
Concluímos que o número de misturas não explosivas que podem ser produzidas é C10, 6 – C8, 4.
Solução:
Temos:
10,6
10! 10x9x8x7x 6!
C
6!4!
= =
6!
8,4
210
x 4x3x2x1
8! 8x7x6x5x 4!
C
4!4!
=
= =
4x3x2x1x 4!
70=
Logo: C10, 6 – C8, 4 = 210 – 70 = 140.
D
H
A
I
G
F
E
CB
Professor: Fabrício MaiaMatemática
36
89 Dê a condição sobre o inteiro positivo n para que o desenvolvimento de
n
2 1
x
x
⎛ ⎞
−⎜ ⎟
⎝ ⎠
apresente um termo
independente de x e não-nulo.
Solução:
O termo geral do desenvolvimento de
n
2 1
x
x
⎛ ⎞
−⎜ ⎟
⎝ ⎠
é
k
2 n k 1n
T (x )
k x
− ⎛ ⎞⎛ ⎞
= − =⎜ ⎟ ⎜ ⎟
⎝ ⎠ ⎝ ⎠
2n 2k k kn
x ( 1) x
k
− −⎛ ⎞
− =⎜ ⎟
⎝ ⎠
k 2n 3kn
( 1) x
k
−⎛ ⎞
−⎜ ⎟
⎝ ⎠
Para o termo independente de x devemos ter 2n – 3k = 0, logo k =
2n
3
. Como k dever ser inteiro, concluímos
que n deve ser um múltiplo de 3.
90 Calcule a e b de modo que a fração algébrica
2
2
x ax b
2x 1
+ +
+
tenha o mesmo valor numérico para todo x ∈ 1 .
Solução:
Devemos ter:
2
2
x ax b
2x 1
+ +
+
= k, x∀ ∈1 ; logo: x2
+ ax + b ≡ 2kx2
+ k
1 2k
a 0
b k
=⎧
⎪
=⎨
⎪ =⎩
A resposta é a = 0 e b =
1
2
.
91 Calcule o valor numérico de x4
+ 4x3
y + 6x2
y2
+ 4xy3
+ y4
, para 4 4
3 1 3 1
x e y
3 3
+ −
= = .
Solução:
x4
+ 4x3
y + 6x2
y2
+ 4xy3
+ y4
= (x + y)4
=
4 4
4 2
4 4 4
3 1 3 1 2 3 2 3
48.
33 3 3
⎛ ⎞ ⎛ ⎞+ − ⋅
+ = = =⎜ ⎟ ⎜ ⎟⎜ ⎟ ⎜ ⎟
⎝ ⎠ ⎝ ⎠
92 O número 2 é raiz dupla de ax3
+ bx + 16. Determine a e b.
Solução:
Como admite raiz dupla, o grau da equação ax3
+ bx + 16 = 0 é maior que 1. Então, a ≠ 0 e concluímos que o
grau é 3. Há, portanto, 3 raízes. Supondo que as raízes são 2, 2 e α, com α ≠ 2, temos pelas relações de Girard:
2 2 0 4
4
b a
2 2 2 2 , logo 4 4 , logo a 1
a b b 12
16 4
2 2
a a
⎧ ⎧
⎪ ⎪+ + α = α = −
α = −⎧⎪ ⎪⎪ ⎪ ⎪
⋅ + α + α = α + = =⎨ ⎨ ⎨
⎪ ⎪ ⎪ = −⎩− −⎪ ⎪⋅ ⋅
⎪ ⎪⎩ ⎩
α = α =
Portanto: a = 1 e b = – 12
93 Qual é o valor de
12
k
k 0
12
9 ?
k=
⎛ ⎞
⎜ ⎟
⎝ ⎠
∑
Solução:
12 12
k 12 k k
k 0 k 0
12 12
9 1 9
k k
−
= =
⎛ ⎞ ⎛ ⎞
= ⋅ ⋅⎜ ⎟ ⎜ ⎟
⎝ ⎠ ⎝ ⎠
∑ ∑
Notando que 12 k k12
1 9
k
−⎛ ⎞
⋅ ⋅⎜ ⎟
⎝ ⎠
é o termo geral do binômio (1 + 9)12
, concluímos que:
12
k 12 12
k 0
12
9 (1 9) 10
k=
⎛ ⎞
= + =⎜ ⎟
⎝ ⎠
∑ ( o que dá 1 trilhão).
Este fator é igual a 1, portanto não altera o valor do termo.
Professor: Fabrício Maia
Matemática
37
94 Numa urna há 12 etiquetas numeradas, 6 com números positivos e 6 com números negativos. De quantos
modos podemos escolher 4 etiquetas diferentes tal que o produto dos números nelas marcados seja positivo?
Solução:
Teremos o produto positivo em cada caso seguinte:
I) Escolhendo 4 etiquetas com números positivos; ou
II) Escolhendo 4 etiquetas com números negativos; ou
III) Escolhendo 2 etiquetas com números positivos e 2 com números negativos.
Números disponíveis: 6 positivos 6 negativos
Possibilidades: 4 positivos 0 negativos
ou
0 positivos 4 negativos
ou
2 positivos 2 negativos
Vamos calcular o número de possibilidades de cada caso (lembrando que não importa a ordem das etiquetas).
I) O número de modos a escolher 4 números positivos, dispondo de 6 números positivos, é 6,4C .
6,4
6! 6 x 5 x 4!
C
4!2!
= =
4!
15
x 2 x 1
=
II) Como temos também 6 números negativos, o número de modos de escolher 4 deles é 6,4C = 15.
III) Dos 6 positivos devemos escolher 2( 6,2C ) e, para cada escolha destes, dos 6 negativos devemos escolher
também 2 (C6, 2). O número de possibilidades deste caso é 6,2 6,2C C⋅ . Como 6,2
6!
C 15,
2!4!
= = temos
15 ⋅ 15 = 225 possibilidades.
Então, o total de possibilidades para o produto positivo é 15 + 15 + 225 = 255.
95 Encontre o coeficiente de x5
no desenvolvimento de (1 – x) (1 + x)8
.
Solução:
Quando multiplicamos (1 – x) pelo polinômio obtido desenvolvendo (1 + x)8
, o termo em x5
resulta da adição de
dois produtos:
(1 – x) (1 + ... + termo em x4
+ termo em x5
+ ... + x8
)
Termo em x5
= 1⋅ [termo em x5
de (1 + x)8
] + [(– x) ⋅ termo em x4
de (1 + x)8
]
O termo geral de (1 + x)8
é T =
8 k k k8 8
1 x x .
k k
−⎛ ⎞ ⎛ ⎞
⋅ ⋅ =⎜ ⎟ ⎜ ⎟
⎝ ⎠ ⎝ ⎠
Para k = 5 temos T =
5 5 5 58! 8 x 7 x 68
x x x 56x .
5 5! 3! 3 x 2 x 1
⎛ ⎞
= = =⎜ ⎟
⎝ ⎠
Para k = 4 temos T =
4 4 4 48! 8 x 7 x 6 x 58
x x x 70x .
4 4! 4! 4 x 3 x 2 x 1
⎛ ⎞
= = =⎜ ⎟
⎝ ⎠
Então, no produto (1 – x) (1 + x) 8
temos:
Termo em x5
= [1 x 56x5
] + [(– x) ⋅ 70x4
] = 56x5
– 70x5
= – 14x5
O coeficiente pedido é igual a – 14.
↓ ↓
6,4C
6,4C
⋅6,2 6,2C C
Professor: Fabrício MaiaMatemática
38
96 Se A é uma matriz quadrada de ordem três com detA = 5, então o valor de det(2A) é:
A) 6 B) 11 C) 15 D) 30 E) 40
Solução:
Sabemos que:
det(k.A) = kn
⋅ det(A), onde:
n → é a ordem da matriz A
Então: det(2A) = 23
⋅ det(A) = 8 ⋅ 5 = 40.
Resposta: E
97 Se a matriz A satisfaz A2
– 2A + I = 0, então A– 1
:
A) não existe.
B) é igual a I.
C) é igual a A.
D) é igual a A – 2I.
E) é igual a 2I – A.
Solução:
Sabemos que: A ⋅ A– 1
= A –1
⋅ A = I
Então:
A2
– 2A + I = 0 → I = 2A – A2
→
→ I = 2AI – A2
→ I = 2IA – AA →
→ I = (2I – A) ⋅ A → A–1
= 2I – A
Resposta: E
98 Uma loja, realizando uma promoção, oferece um desconto de 20% nos preços dos seus produtos. Para voltar
aos preços iniciais, os preços promocionais devem sofrer um acréscimo de A%.
Determine o valor de A.
A) 10 B) 20 C) 25 D) 30 E) 40
Solução:
Preço inicial = P → com desconto =
80
P
100
Para voltar ao preço inicial, temos:
Resposta: C
14444244443
novo preço
+ ⋅ =
⋅ =
=
=
80 A 80
P P P
100 100 100
A 80 20
P P
100 100 100
A 1
100 4
A 25
Professor: Fabrício Maia
Matemática
39
99 Sejam p e q números inteiros positivos e consecutivos. Se
1 1 11
,
p q 30
+ = então p + q é igual a:
A) 9 B) 11 C) 13 D) 15 E) 17
Solução:
1 1 11 q p 11
p q 30 pq 30
+
+ = ⇒ =
Como p e q são inteiros positivos consecutivos, então p e q são primos entre si, isto é, m.d.c. (p, q) = 1.
Assim, p = 5 e q = 6 ou p = 6 e q = 5.
Portanto, p + q = 11
Resposta: B
100 O gráfico da função f(x) = sen x no intervalo [
2
π ,π[ e:
A) crescente B) decrescente C) constante D) nula E) negativa
Solução:
Esboço do gráfico de sen x no de [0,2π]
Resposta: B
101 Um dos ângulos de um triângulo retângulo é α. Se tg α = 2,4 , os lados desse triângulo são proporcionais a:
A) 30,40,50 B) 80,150,170 C) 120, 350, 370 D) 50, 120, 130 E) 61, 60, 11
Solução:
Se tgα =2,4 → tgα =
24
10
→ tgα = 12
5
cateto oposto = 12k e cateto adjacente = 5k → hipotenusa = 13k.
Tomando k =10 → lados: 50,120 e 130
Resposta: D
102 A distância do ponto de interseção das retas 2x – 3y + 26 = 0 e 5x + 2y – 49 = 0 à origem é:
A) 13 B) 23 C) 15 D) 18 E) 17
Solução:
Resolvendo o sistema formado pelas equações acima, encontramos como interseção o ponto (5, 12). Assim, a
distância do ponto (5, 12) ao ponto (0, 0) é igual a d=
2 2
(5-0) (12 0) 169 13.+ − = =
Resposta: A
103 As promoções do tipo “leve 3 pague 2”, comuns no comércio, acenam com um desconto, sobre cada unidade
vendida, de:
A)
50
%
3
B) 20% C) 25% D) 30% E)
100
%
3
Solução:
Observe:
I) Quem leva 3 e paga 2 está comprando 3 e tendo um desconto de 1;
II) Se 1 é p% de 3 → 1 =
P 100
3 P
100 3
⋅ → = ou P = 33,33% (aproximadamente)
Resposta: E
0
y
x
decrescente
Professor: Fabrício MaiaMatemática
40
104 Sabendo que cos 36º =
1 5
2
+
, então cos 72º vale:
A)
1 5
2
+
B)
5 1
4
−
C)
5 1
2
−
D)
1 5
2
−
E)
1 5
4
−
Solução:
Sabemos que: cos2x = 2cos2
x – 1 (arco duplo)
Tomando x = 36º, encontramos:
cos72º = 2cos2
36º – 1 → cos72º = 2
2
1 5 5 1
1 cos72º
4 4
⎛ ⎞+ −
⋅ − → =⎜ ⎟⎜ ⎟
⎝ ⎠
Resposta: B
105 Se y = cos2280º, então y é igual a:
A) – cos12º B) – cos60º C) – cos30º D) cos12º E) cos60º
Solução:
Sabemos que: se a + b = 180º, então sen a = sen b e cos a = – cos b
Dividindo 2280º por 360º, encontramos:
2280º = 120º + 6.360º
Assim: cos 2280º = cos 120º = – cos 60º
Resposta: B
106 A área máxima da região limitada por um triângulo retângulo inscrito em um círculo de raio R é:
A) 2R2
B) πr2
C) R2
D)
2
R
2
E) 2πr2
Solução:
I) A área de um triângulo é igual a base x altura
2
.
II) Tome AB como base → base = 2R (diâmetro), pois O é o centro.
III) De todas as alturas relativas à hipotenusa AB, a maior é EO = R, onde R é o raio.
Logo, o triângulo de área máxima tem área igual a
22R R
R .
2
⋅
=
Veja figura:
Os ângulos ˆ ˆ ˆ ˆACB,ADB,AEB e AFB são retos.
Resposta: C
107 Se p é natural maior que 1, não divisível por 2 e nem por 3, então p2
– 1 é divisível por:
A) 18 B) 24 C) 36 D) 9 E) 27
Solução:
É fácil ver que:
I) Se p não é múltiplo de 2 → p – 1 e p + 1 são pares consecutivos, logo (p – 1) ⋅ (p + 1) é múltiplo de 8.
II) Se p não é múltiplo de 3 → p – 1 ou p + 1 será um múltiplo de 3, logo (p – 1) ⋅ (p + 1) é múltiplo de 3.
De (I) e (II), concluímos: O produto (p – 1) ⋅ (p + 1) = p2
– 1 é um múltiplo de 24.
Resposta: B
A B
OG H
E
D
C
I
F
Professor: Fabrício Maia
Matemática
41
108 Sejam a e b números reais positivos tais que a + 3b = 30. Determine qual é o maior valor possível para ab.
Solução:
Sabemos que: Se a1, a2, a3, ..., an são números reais positivos, então:
Média aritmética =
1 2 3 na a a ... a
n
+ + + +
e Média geométrica = n
1 2 3 na a a ...a⋅ ⋅ ⋅
Relação importante entre as duas médias: M A M G⋅ ≥ ⋅
Daí,
a 3b 30
a 3b 3ab 15 3ab 225 3ab ab 75
2 2
+
≥ ⋅ → ≥ → ≥ → ≥ → ≤
Portanto, o maior valor para ab é 75.
Resposta: 75
109 Seja x um número natural, que ao ser dividido por 9 deixa resto 5, e ao ser dividido por 3 deixa resto 2.
Sabendo que a soma dos quocientes é 9, podemos afirmar que x e igual a:
A) 28 B) 35 C) 27 D) 33 E) 23
Solução:
Temos que: x = 9a + 5, onde a é o quociente da divisão de x por 9.
x = 3b + 2, onde b é o quociente da divisao de x por 3.
Como a soma dos quocientes é 9, vem:
x 5 x 2
9 x 23
9 3
− −
+ = → =
Resposta: E
110 Se 10tgx + 16cosx – 17secx = 0, então senx é igual a:
A)
1 1
ou
2 8
B)
1
2
± C)
1 1
ou
4 12
D) 0 E)
1 1
ou
2 4
−
Solução:
Temos: 10tgx + 16cosx – 17secx = 0
senx 1
10 16cosx –17 0
cosx cosx
→ ⋅ + ⋅ = → 10senx + 16cos2
x – 17 = 0
→ 10senx + 16⋅ (1 – sen2
x) – 17 = 0 → 16sen2
x – 10senx + 1 = 0
1 1
senx ou senx
2 8
→ = =
Resposta: A
111 Um atleta, correndo com velocidade constante, completou a maratona em M horas. A fração do percurso
que ele correu em 2M minutos, foi:
A)
1
2
B)
1
6
C)
1
15
D)
1
30
E)
1
20
Solução:
Temos que: M horas = M⋅ 60 minutos
Veja:
Se M⋅ 60 minutos = P (percurso completo) → 2⋅ M minutos =
P
30
(fração do percurso)
Resposta: D
Professor: Fabrício MaiaMatemática
42
112 Sendo R = 02
– 12
+ 22
– 32
+ 42
– 52
+ ... + 982
– 992
+ 1002
, calcule o valor de
R
.
202
A) 22 B) 23 C) 24 D) 25 E) 26
Solução:
Sabemos que: (I) a2
– b2
= (a – b)⋅ (a + b) e (II) Sn =
1 n(a a ) n
2
+ ⋅
(soma dos termos de uma P.A.)
Veja:
R = (22
– 12
) + (42
– 32
) + (62
– 52
)+ ... + (1002
– 992
)
R = (2 – 1) ⋅ (2 + 1) + (4 – 3)⋅ (4 + 3) + (6 – 5)⋅ (6 + 5) + ... + (100 – 99)⋅ (100 + 99)
R = 3 + 7 + 11 + ... + 199
R =
(3 199) 50 R
202.25 25.
2 202
+ ⋅
= → =
Resposta: D
113 O primeiro termo a de uma progressão aritmética de razão 13 satisfaz 0 ≤ a ≤ 10. Se um dos termos da
progressão é 35, determine o valor de a.
A) 5 B) 6 C) 7 D) 8 E) 9
Solução:
Temos: P.A. (a, a + 13, a + 26, a + 39, ... , 35, ...)
Usando a fórmula do termo geral, encontramos:
an = a1 + (n – 1)⋅ r → 35 = a + (n – 1)⋅ 13 → 35 = a + 13n – 13 → a = 48 – 13n → 0 ≤ 48 –13n ≤ 10 →
– 48 ≤ –13n ≤ –38 → 48 ≥13n ≥ 38 → 2, ... ≤ n ≤ 3, ... → n = 3 → a = 48 – 13⋅ 3 → a = 9.
Resposta: E
114 O algarismo das unidades do número N = 1.3.5.7. ... . 1993 é:
A) 1 B) 3 C) 5 D) 7 E) 9
Solução:
Note que o produto de qualquer número ímpar por 5 sempre termina em 5; logo, com o número N só tem
fatores ímpares, seu algarismo das unidades é 5.
Resposta: C
115 Se
x y
7 3
= e x⋅ y = 189, então: x – y vale com x e y positivos:
A) 12 B) 4 C) 9 D) 30 E) 21
Solução:
Temos:
x y
k
7 3
= = → x = 7k e y = 3k
Daí: 7k⋅ 3k = 189 → 21k2
= 189 → k2
= 9 → k = 3
Para: k = 3 → x = 21 e y = 9
Portanto: x – y = 12
Resposta: A
116 A planta de um apartamento está confeccionada na escala 1:50. Então a área real, em m2
, de uma sala
retangular, cujas medidas na planta são 12cm e 14cm, é:
A) 24 B) 26 C) 28 D) 42 E) 54
Solução:
Sabemos que escala é a razão entre o comprimento no desenho e o comprimento real, medidos na mesma
unidade.
Professor: Fabrício Maia
Matemática
43
Daí:
1 12
50 x
= → x = 600cm → x = 6m
1 14
50 x
= → y = 700cm → y = 7m
Logo, a área da sala será de 6m x 7m = 42m2
.
Resposta: D
117 Prove que todo triângulo ABC vale a relação: ˆ ˆc a cosB b cosA.= ⋅ + ⋅
Solução:
mˆ ˆcosA m bcosA
b
nˆ ˆcosB n acosB
a
ˆ ˆ ˆ ˆlogo, m n bcosA acosB c acosB bcosA. (OK)
= → =
= → =
+ = + → = +
Obs: Sendo ˆA ou ˆB obtuso, chegamos com raciocínio análogo ao mesmo resultado.
118 Considerem-se todas as divisões de números inteiros positivos por 17, cujo resto é igual ao quadrado do
quociente. A soma dos quocientes dessas divisões é:
A) 10 C) 172
E) 12
+ 22
+ 32
+ ... + 172
B) 17 D) 1 + 2 + 3 + ... + 17
Solução:
Temos que: n dividido por 17 tem quociente q ≠ 0 e resto r com r = q2
.
Veja:
(I) Possíveis restos de uma divisão por 17 são: 0, 1, 2, 3 ,... , 16
(II) Como r tem que ser um quadrado perfeito, devemos ter:
r = 0 → q = 0 (não sastisfaz)
r = 1 → q = 1
r = 4 → q = 2
r = 9 → q = 3
r = 16 → q = 4
Logo, a soma dos quocientes é 10.
Resposta: A
119 Determine o valor do produto P = cos36º⋅ cos72º.
Solução:
Sabemos que sen2x = 2senxcosx → 2sen36º⋅ P = 2sen36º⋅ cos36ºcos72º → 2sen36º⋅ P = sen72ºcos72º→
→ 4sen36º⋅ P = 2sen72ºcos72º → 4sen36º⋅ P = sen144º → 4 ⋅ P = 1, pois sen36º = sen144º (suplementares)
→ P =
1
4
.
Resposta:
1
4
A B
C
ab
m nH
c
Professor: Fabrício MaiaMatemática
44
120 Sejam f(x) =
1
x 1−
, x > 1 e g uma função tal que (gof)(x) = x. Determine o valor de g
1
64
⎛ ⎞
⎜ ⎟
⎝ ⎠
.
Solução:
g(f(x)) = x → g
1
x 1
⎛ ⎞
⎜ ⎟−⎝ ⎠
= x
Veja: se
1 1
x 1 64
=
−
→ x – 1 = 64 → x = 65 →
1
g
64
⎛ ⎞
⎜ ⎟
⎝ ⎠
= 65
Resposta: 65
121 O triângulo ABC está inscrito em um círculo de raio R. Se cosA =
3
5
, o comprimento do lado BC é igual a:
A)
2R
5
B)
3R
5
C)
4R
5
D)
6R
5
E)
8R
5
Solução:
Temos que:
3ˆcosA
5
= ( ˆA é agudo) → sen ˆA =
4
5
Lei dos senos
a b c BC
2R 2R
ˆ ˆ ˆ ˆsenA senB senAsenC
= = = → = → BC = 2R⋅ ˆsenA → BC = 2R⋅
4
5
→ BC =
8R
5
Resposta: E
122 Seja f(x) =
x x
x x
e e
e e
−
−
−
+
definida em R. Se g for a função inversa de f, o valor de
7
g
25
e
⎛ ⎞
⎜ ⎟
⎝ ⎠ será:
A) – 1 B) 0 C)
1
e
D) 1 E)
4
3
Solução:
Como g é a inversa de f, temos:
x x
1
x x
7 7 7 e e
g( ) f ( )
25 25 25 e e
−
−
−
−
= → = →
+
7⋅ ex
+ 7⋅ e–x
= 25⋅ ex
– 25⋅ e–x
→ – 18⋅ ex
= – 32⋅ e– x
→
7
g( )
2x x 25
16 4 4
e e e .
9 3 3
→ = → = → =
Resposta: E
123 A média aritmética dos ângulos internos de um eneágono convexo vale:
A) 40º B) 70º C) 120º D) 135º E) 140º
Solução:
Sabemos que a soma dos ângulos internos de um polígono convexo é dada pela fórmula S = (n – 2)⋅ 180º.
Daí, a soma dos ângulos internos de um eneágono convexo é igual a S = (9 – 2)⋅ 180º = 7.180º.
Portanto, a média aritmética será igual a
7.180º
7 20º 140º.
9
= ⋅ =
Resposta: E
Professor: Fabrício Maia
Matemática
45
124 Uma solução tem 75% de ácido puro. Quantos gramas de ácido puro devemos adicionar a 48 gramas da
solução para que a nova solução contenha 76% de ácido puro?
Solução:
Veja:
Em 48 gramas de solução temos
75
. 48 36
100
= gramas de ácido puro.
Adicionando x gramas de ácido puro a solução, teremos:
Nova solução = (48 + x) gramas
Quantidade de ácido puro = (36 + x) gramas
Assim:
36 x 76
48 x 100
+
= →
+
x = 2 gramas
Resposta: 2 gramas
125 O gráfico de uma função f é o segmento de reta que une os pontos (– 3, 4) e (3, 0). Se f–1
é a função inversa
de f, então o valor de f– 1
(2) é igual a:
A) 1 B)
2
3
C) 3 D) 0 E)
3
2
Solução:
Temos: f(x) = ax + b, com a ≠ 0.
Pontos: (– 3, 4) e (3, 0)
Para x = – 3 → a ⋅ (– 3) + b = 4
Para x = 3 → a⋅ 3 + b = 0
Resolvendo, encontramos: b = 2 e a =
2
3
−
Daí, a função f é dada por: f(x) =
2
x 2
3
− +
Veja: cálculo de f – 1
(2)
Para y = 2 → 2 =
2
x 2
3
− + → x = 0 → f – 1
(2) = 0.
Resposta: D
126 Um elevador pode levar 20 adultos ou 24 crianças. Se 15 adultos já estão no elevador, quantas crianças podem
ainda entrar?
A) 5 B) 6 C) 7 D) 8 E) 9
Solução:
Se 20 adultos equivalem a 24 crianças → 5 adultos equivalem a 6 crianças
Veja:
(I) O elevador pode levar 20 adultos
(II) Tem 15 adultos no elevador → faltam 5 adultos (equivalem a 6 crianças) .
Resposta:B
127 Uma torneira enche um tanque em 4 horas. O ralo do tanque pode esvaziá-lo em 3 horas. Estando o tanque
cheio, abrimos, simultaneamente, a torneira e o ralo. Então o tanque:
A) nunca se esvazia.
B) esvazia-se em 1 hora.
C) esvazia-se em 4 horas.
D) esvazia-se em 7 horas.
E) esvazia-se em 12 horas.
Solução:
Capacidade do tanque: T
(I) Torneira enche T em 4 horas → em 1 hora enche T
4
do tanque
(II) Ralo esvazia o tanque T em 3 horas → em 1 hora esvazia T
3
do tanque
Professor: Fabrício MaiaMatemática
46
Assim, o tanque em uma hora esvazia
1
12
de sua capacidade.
Veja:
T T 3T 4T T
4 3 12 12
−
− = = − .
Portanto, o tanque esvazia-se em 12 horas.
Resposta: E
128 Determine o valor de – 63⋅ log8(sen15º ⋅ sen75º).
Solução:
Fatos que ajudam:
(I) Se a + b = 90º, então sen a = cos b e sen b = cos a
(II) Sen 2a = 2sen a cos a
(III) n
m
a
log b = a
m
log b
n
⋅
Temos que sen15º⋅ sen75º = sen15º⋅ cos15º =
2 sen15º cos15º sen30º 1
2 2 4
⋅ ⋅
= =
Então, a expressão vale:
Exp. = – 63⋅ log8
3
2
22
1 2
63 log 2 63 ( ) log 2 42 1 42
4 3
−⎛ ⎞
= − ⋅ = ⋅ − ⋅ = ⋅ =⎜ ⎟
⎝ ⎠
Resposta: 42
129 Considere um quadrilátero convexo ABCD de área igual a 66cm2
. Determine, em cm2
, a área do quadrilátero
cujos vértices são os pontos médios dos lados do quadrilátero ABCD.
Solução:
Fatos que ajudam: ÁREAS
(I) Seja ABC um triângulo qualquer e seja MNP o triângulo que tem vértices nos pontos médios dos lados do
triângulo ABC, temos que: Área(MNP) =
Área(ABC)
4
.
(II) Seja ABCD um quadrilátero qualquer e seja MNQP o quadrilátero que tem vértices nos pontos médios dos
lados de ABCD, temos que: Área(MNQP) =
Área(ABCD)
2
.
Usando o resultado (II) no enunciado da questão, concluímos:
Resposta: 33cm2
130 Se x é um número real tal que x2
+ 2
1
3
x
= , determine o valor de x4
+ 4
1
.
x
Solução:
Se
2
2 2 2 4 2 4 4
2 2 2 4 4 4
1 1 1 1 1 1
x 3 x 3 x 2 x 9 x 2 9 x 7
x x x x x x
⎛ ⎞
+ = → + = → + ⋅ ⋅ + = → + + = → + =⎜ ⎟
⎝ ⎠
.
Resposta: 7
131 O menor número natural n, diferente de zero, que torna o produto de 3888 por n um cubo perfeito é:
A) 6 B) 12 C) 15 D) 18 E) 24
Solução:
Fatorando o número 3888, obtemos 3888 = 24
⋅ 35
Para formar um cubo perfeito devemos multiplicar os dois membros no mínimo por 22
⋅ 31
, para que as potências
dos números 2 e 3 sejam múltiplos de 3. Assim, o menor número que devemos multiplicar por 3888 para obter
um cubo perfeito é 12.
Resposta: B
Professor: Fabrício Maia
Matemática
47
132 Quantos números inteiros há entre 602
e 612
que não são quadrados perfeitos?
A) 118 B) 119 C) 120 D) 121 E) 122
Solução:
Veja: 602
e 612
são quadrados perfeitos consecutivos, então, qualquer inteiro entre eles não é quadrado perfeito.
Inteiros que não são quadrados perfeitos: 3601, 3602, 3603, ... ,3720
Quantidade de inteiros que não são quadrados é igual a 3720 – 3601 + 1 = 120.
Resposta: C
133 O período da função f (x) = sen4
x + cos4
x vale:
A) 2π B) π C)
3π
2
D)
π
2
E)
4
π
Solução:
Sabemos: Se f é periódica → f(x + p) = f(x), para todo x no domínio da função. O menor valor positivo de
p, chamamos de período de f.
Tomando x = 0, encontramos:
f (p) = f (0) → sen4
p + cos4
p = sen4
0 + cos4
0 → sen4
p + cos4
p = 1 → (sen2
p + cos2
p)2
– 2sen2
p⋅ cos2
p = 1 →
1 – 2sen2
pcos2
p =1 → 2sen2
p cos2
p = 0 → senp = 0 ou cosp = 0.
* Se senp = 0 → p = 0, π, 2π,...
* Se cosp = 0 → p =
3
, ,...
2
π π
2
Agora, devemos verificar se p =
2
π
satisfaz a condição f(x + p) = f(x), para todo x no domínio da função.
Veja: f(x + p) = f(x +
2
π
) = sen4
(x +
2
π
) + cos4
(x +
2
π
) = (cosx)4
+ (–senx)4
= sen4
x + cos4
x = f (x) (OK)
Resposta: D
Obs: No ciclo trigonométrico, encontramos facilmente: sen(x +
2
π
) = cosx e cos(x +
2
π
) = – senx (verifique!)
134 O conjunto solução da equação x⋅ (log53x
+ log521) + log5(
3
7
)x
= 0 é igual a:
A) { } B) {0} C) {1} D) {0, 2} E) {0, – 2}
Solução:
Temos que:
x⋅ (log53x
+ log521) + log5(
3
7
)x
= 0 → x⋅ log5(3x
⋅ 21) + log5(
3
7
)x
= 0 → log5(3x
⋅ 21)x
+ log5(
3
7
)x
= 0
→ log5[(3x
⋅ 21)x
⋅ (
3
7
)x
] = 0 → (3x
⋅ 21)x
⋅ (
3
7
)x
= 1 → (3x
⋅ 21⋅
3
7
)x
= 1 → (3x
⋅ 3⋅ 3)x
= 1 →
→ (3x + 2
)x
= 1 → 2
x 2x
3 1+
= → x2
+ 2x = 0 → x = – 2 ou x = 0.
Resposta: E
135 Um número é composto de 2 algarismos, cuja soma é 9. Invertendo a ordem dos algarismos, obtemos um novo
número igual a
4
7
do original. Qual é o número?
Solução:
* Número original: ab = a⋅ 10 + b (forma polinomial)
* Invertendo os algarismos obtemos um novo número: ba = b⋅ 10 + a (forma polinomial)
Professor: Fabrício MaiaMatemática
48
Equações do problema: a + b = 9 e b⋅ 10 + a =
4
7
⋅ (a⋅ 10 + b)
Segunda equação: 70b + 7a = 40a + 4b → 66b = 33a → a = 2b
Substituindo na primeira equação, teremos:
a + b = 9 → 2b + b = 9 → 3b = 9 → b = 3 → a = 6
Resposta: 63
136 A distância entre dois lados paralelos de um hexágono regular é igual a 2 3 cm. A medida do lado desse
hexágono, em centímetros, é:
A) 3 B) 2 C) 2,5 D) 3 E) 4
Solução:
Veja:
(I) BF = 2 3 , pois BC//FE
(II) ae = i
360º
60º a 120º
6
= → =
Aplicando a lei dos cossenos no triângulo ABF, teremos:
2 2 2 2 2
(2 3) x x 2 x x cos120º 12 2x x x 2= + − ⋅ ⋅ ⋅ → = + → =
Resposta: B
137 Qualquer que seja x, o valor de sen6
x + cos6
x + 3sen2
xcos2
é:
A) 0 B) sen2
x C) cos2
x D) 1 E) senx⋅ cosx
Solução:
Lembre: produto notável → (a + b)3
= a3
+ b3
+ 3ab(a + b)
Temos que:
sen2
x + cos2
x = 1→ (sen2
x + cos2
x)3
= 13
→ sen6
x + cos6
x + 3sen2
xcos2
x(sen2
x + cos2
x) = 1
→ sen6
x + cos6
x + 3sen2
xcos2
x⋅ 1= 1 → sen6
x + cos6
x + 3sen2
xcos2
x = 1.
Resposta: D
138 Quantas soluções reais e distintas possui a equação x2
+ 9 = 3senx?
A) 0 B) 1 C) 2 D) 3 E) infinitas
Solução:
Observe:
(I) x2
+ 9 é sempre maior ou igual a 9, para todo x real.
(II) 3sen x assume no máximo o valor 3.
(III) A igualdade não ocorre para nenhum valor real de x.
Logo, a equação não possui solução.
Resposta: A
139 O resto da divisão de P(x) = x5
+ 4x4
+ 2x3
+ x2
+ x –1 por q(x) = x + 2 é:
A) 17 B) 15 C) 0 D) – 15 E) – 17
Solução:
Veja:
(I) – 2 é raiz de q(x)
(II) P(–2) é o resto da divisão de P(x) por q(x).
Então, pelo Teorema do resto, encontramos:
resto = P(– 2) = – 32 + 64 – 16 + 4 – 2 – 1 = 17
Resposta: A
C
D
E
Professor: Fabrício Maia
Matemática
49
140 O valor mínimo de cosx + secx, para 0 < x <
π
2
é igual a:
A) 0 B) 1 C) 2 D) 2 E) 3
Solução:
Sabemos que (n – 1)2
≥ 0, n∀ ∈1
Então: n2
– 2n + 1 ≥ 0 → n2
+ 1 ≥ 2n
Considerando n positivo, tem-se:
2
n 1 1
2 n 2
n n
+
≥ ⇔ + ≥
Portanto, um número positivo adicionado ao seu inverso é sempre maior ou igual a 2.
Resposta: D
Anotações
Professor: Fabrício MaiaMatemática
50
04698/06-TP©
02/06/06

Contenu connexe

Tendances

Uso de balança no estudo da equação do 1º grau
Uso de balança no estudo da equação do 1º grauUso de balança no estudo da equação do 1º grau
Uso de balança no estudo da equação do 1º grauGraciele Schulz Arnholz
 
Operações com frações adição, subtração.ppt
Operações com frações adição, subtração.pptOperações com frações adição, subtração.ppt
Operações com frações adição, subtração.pptGalbertoGomesOliveir1
 
Divisão com frações e problemas
Divisão com frações e problemasDivisão com frações e problemas
Divisão com frações e problemastcrisouza
 
AULA 16 - PLANO CARTESIANO.ppt
AULA 16 - PLANO CARTESIANO.pptAULA 16 - PLANO CARTESIANO.ppt
AULA 16 - PLANO CARTESIANO.pptProfessorJooHlio
 
Círculo e circunferência
Círculo e circunferênciaCírculo e circunferência
Círculo e circunferênciamariacferreira
 
PosiçãO Relativa Entre Reta E CircunferêNcia
PosiçãO Relativa Entre Reta E CircunferêNciaPosiçãO Relativa Entre Reta E CircunferêNcia
PosiçãO Relativa Entre Reta E CircunferêNciaISJ
 
Conjuntos numéricos e intervalos na reta real
Conjuntos numéricos e intervalos na reta realConjuntos numéricos e intervalos na reta real
Conjuntos numéricos e intervalos na reta realAntonio Carlos Luguetti
 
Função 1º grau definição e notação de função - exemplos resolvidos
Função 1º grau   definição e notação de função - exemplos resolvidosFunção 1º grau   definição e notação de função - exemplos resolvidos
Função 1º grau definição e notação de função - exemplos resolvidosAdriano Souza
 
atividades áreas
atividades áreas atividades áreas
atividades áreas Frank Junior
 
1) matrizes 2012 (prevest)
1) matrizes 2012 (prevest)1) matrizes 2012 (prevest)
1) matrizes 2012 (prevest)Márcio Queiroz
 
290711169 livro-de-matematica-alvaro-andrini-6-ano-pdf
290711169 livro-de-matematica-alvaro-andrini-6-ano-pdf290711169 livro-de-matematica-alvaro-andrini-6-ano-pdf
290711169 livro-de-matematica-alvaro-andrini-6-ano-pdfDaniel De Oliveira Silva
 
Reta numérica (7 ano professor ryldon)
Reta numérica  (7 ano professor ryldon)Reta numérica  (7 ano professor ryldon)
Reta numérica (7 ano professor ryldon)ryldon
 

Tendances (20)

Uso de balança no estudo da equação do 1º grau
Uso de balança no estudo da equação do 1º grauUso de balança no estudo da equação do 1º grau
Uso de balança no estudo da equação do 1º grau
 
Esferas
EsferasEsferas
Esferas
 
Resolução lista 1 pirâmide
Resolução lista 1   pirâmideResolução lista 1   pirâmide
Resolução lista 1 pirâmide
 
Operações com frações adição, subtração.ppt
Operações com frações adição, subtração.pptOperações com frações adição, subtração.ppt
Operações com frações adição, subtração.ppt
 
Arranjo simples
Arranjo simplesArranjo simples
Arranjo simples
 
Divisão com frações e problemas
Divisão com frações e problemasDivisão com frações e problemas
Divisão com frações e problemas
 
AULA 16 - PLANO CARTESIANO.ppt
AULA 16 - PLANO CARTESIANO.pptAULA 16 - PLANO CARTESIANO.ppt
AULA 16 - PLANO CARTESIANO.ppt
 
Círculo e circunferência
Círculo e circunferênciaCírculo e circunferência
Círculo e circunferência
 
A história da matemática
A história da matemáticaA história da matemática
A história da matemática
 
Segmentos tangentes
Segmentos tangentesSegmentos tangentes
Segmentos tangentes
 
Esferas
EsferasEsferas
Esferas
 
PosiçãO Relativa Entre Reta E CircunferêNcia
PosiçãO Relativa Entre Reta E CircunferêNciaPosiçãO Relativa Entre Reta E CircunferêNcia
PosiçãO Relativa Entre Reta E CircunferêNcia
 
Conjuntos numéricos e intervalos na reta real
Conjuntos numéricos e intervalos na reta realConjuntos numéricos e intervalos na reta real
Conjuntos numéricos e intervalos na reta real
 
Geometria espacial cilindros
Geometria espacial cilindrosGeometria espacial cilindros
Geometria espacial cilindros
 
Função 1º grau definição e notação de função - exemplos resolvidos
Função 1º grau   definição e notação de função - exemplos resolvidosFunção 1º grau   definição e notação de função - exemplos resolvidos
Função 1º grau definição e notação de função - exemplos resolvidos
 
atividades áreas
atividades áreas atividades áreas
atividades áreas
 
Relações de Girard
Relações de GirardRelações de Girard
Relações de Girard
 
1) matrizes 2012 (prevest)
1) matrizes 2012 (prevest)1) matrizes 2012 (prevest)
1) matrizes 2012 (prevest)
 
290711169 livro-de-matematica-alvaro-andrini-6-ano-pdf
290711169 livro-de-matematica-alvaro-andrini-6-ano-pdf290711169 livro-de-matematica-alvaro-andrini-6-ano-pdf
290711169 livro-de-matematica-alvaro-andrini-6-ano-pdf
 
Reta numérica (7 ano professor ryldon)
Reta numérica  (7 ano professor ryldon)Reta numérica  (7 ano professor ryldon)
Reta numérica (7 ano professor ryldon)
 

En vedette

Mat 140 questoes resolvidas vol i
Mat 140 questoes resolvidas vol iMat 140 questoes resolvidas vol i
Mat 140 questoes resolvidas vol itrigono_metrico
 
Matematica questões resolvidas i
Matematica questões resolvidas iMatematica questões resolvidas i
Matematica questões resolvidas icon_seguir
 
Mat 140 questoes resolvidas vol iii
Mat 140 questoes resolvidas vol iiiMat 140 questoes resolvidas vol iii
Mat 140 questoes resolvidas vol iiitrigono_metrico
 
EXPRESSÕES NUMÉRICAS - Gabarito das expressões numéricas
EXPRESSÕES NUMÉRICAS - Gabarito das expressões numéricasEXPRESSÕES NUMÉRICAS - Gabarito das expressões numéricas
EXPRESSÕES NUMÉRICAS - Gabarito das expressões numéricasOtávio Sales
 
Equação do 1º grau
Equação do 1º grauEquação do 1º grau
Equação do 1º grauRiva Davio
 
Apostila de matrizes determinantes e sistemas 2008
Apostila de matrizes determinantes e sistemas 2008Apostila de matrizes determinantes e sistemas 2008
Apostila de matrizes determinantes e sistemas 2008Jussileno Souza
 
Perguntas para o ensino fundamental maior
Perguntas para o ensino fundamental maiorPerguntas para o ensino fundamental maior
Perguntas para o ensino fundamental maiorFábio Brito
 
Exercícios matrizes ii gabarito
Exercícios matrizes ii gabaritoExercícios matrizes ii gabarito
Exercícios matrizes ii gabaritoOtávio Sales
 
Funções - Exercícios
Funções - ExercíciosFunções - Exercícios
Funções - ExercíciosEverton Moraes
 
Geometria analítica anotações de aula 1° semestre 2010
Geometria analítica anotações de aula 1° semestre 2010Geometria analítica anotações de aula 1° semestre 2010
Geometria analítica anotações de aula 1° semestre 2010Marcos Azevedo
 
340 questões de raciocínio lógico
340 questões de raciocínio lógico340 questões de raciocínio lógico
340 questões de raciocínio lógicoMARIOJR2013
 
Matemática exercícios resolvidos - 01 m1 geometria métrica plana
Matemática   exercícios resolvidos - 01 m1 geometria métrica planaMatemática   exercícios resolvidos - 01 m1 geometria métrica plana
Matemática exercícios resolvidos - 01 m1 geometria métrica planaevandrovv
 
Ap geometria analitica resolvidos
Ap geometria analitica resolvidosAp geometria analitica resolvidos
Ap geometria analitica resolvidostrigono_metrico
 
919 Questões de Física (resolvidas)
919 Questões de Física (resolvidas)919 Questões de Física (resolvidas)
919 Questões de Física (resolvidas)Adriano Capilupe
 
Ap exercicios matematica respostas 002
Ap exercicios matematica respostas  002Ap exercicios matematica respostas  002
Ap exercicios matematica respostas 002trigono_metrico
 
Mat exercicios resolvidos
Mat exercicios resolvidosMat exercicios resolvidos
Mat exercicios resolvidoscomentada
 
Matematica questoes trigonometria gabarito
Matematica questoes trigonometria gabaritoMatematica questoes trigonometria gabarito
Matematica questoes trigonometria gabaritotrigono_metrico
 
MatemáTica Estudo Dos Log
MatemáTica Estudo Dos LogMatemáTica Estudo Dos Log
MatemáTica Estudo Dos Logeducacao f
 
Aula 15 matrizes e determinantes(parte ii)
Aula 15    matrizes e determinantes(parte ii)Aula 15    matrizes e determinantes(parte ii)
Aula 15 matrizes e determinantes(parte ii)J M
 

En vedette (20)

Mat 140 questoes resolvidas vol i
Mat 140 questoes resolvidas vol iMat 140 questoes resolvidas vol i
Mat 140 questoes resolvidas vol i
 
Matematica questões resolvidas i
Matematica questões resolvidas iMatematica questões resolvidas i
Matematica questões resolvidas i
 
Mat 140 questoes resolvidas vol iii
Mat 140 questoes resolvidas vol iiiMat 140 questoes resolvidas vol iii
Mat 140 questoes resolvidas vol iii
 
Banco de exercícios gerais de matematica todo em
Banco de exercícios gerais de matematica todo emBanco de exercícios gerais de matematica todo em
Banco de exercícios gerais de matematica todo em
 
EXPRESSÕES NUMÉRICAS - Gabarito das expressões numéricas
EXPRESSÕES NUMÉRICAS - Gabarito das expressões numéricasEXPRESSÕES NUMÉRICAS - Gabarito das expressões numéricas
EXPRESSÕES NUMÉRICAS - Gabarito das expressões numéricas
 
Equação do 1º grau
Equação do 1º grauEquação do 1º grau
Equação do 1º grau
 
Apostila de matrizes determinantes e sistemas 2008
Apostila de matrizes determinantes e sistemas 2008Apostila de matrizes determinantes e sistemas 2008
Apostila de matrizes determinantes e sistemas 2008
 
Perguntas para o ensino fundamental maior
Perguntas para o ensino fundamental maiorPerguntas para o ensino fundamental maior
Perguntas para o ensino fundamental maior
 
Exercícios matrizes ii gabarito
Exercícios matrizes ii gabaritoExercícios matrizes ii gabarito
Exercícios matrizes ii gabarito
 
Funções - Exercícios
Funções - ExercíciosFunções - Exercícios
Funções - Exercícios
 
Geometria analítica anotações de aula 1° semestre 2010
Geometria analítica anotações de aula 1° semestre 2010Geometria analítica anotações de aula 1° semestre 2010
Geometria analítica anotações de aula 1° semestre 2010
 
340 questões de raciocínio lógico
340 questões de raciocínio lógico340 questões de raciocínio lógico
340 questões de raciocínio lógico
 
Matemática exercícios resolvidos - 01 m1 geometria métrica plana
Matemática   exercícios resolvidos - 01 m1 geometria métrica planaMatemática   exercícios resolvidos - 01 m1 geometria métrica plana
Matemática exercícios resolvidos - 01 m1 geometria métrica plana
 
Ap geometria analitica resolvidos
Ap geometria analitica resolvidosAp geometria analitica resolvidos
Ap geometria analitica resolvidos
 
919 Questões de Física (resolvidas)
919 Questões de Física (resolvidas)919 Questões de Física (resolvidas)
919 Questões de Física (resolvidas)
 
Ap exercicios matematica respostas 002
Ap exercicios matematica respostas  002Ap exercicios matematica respostas  002
Ap exercicios matematica respostas 002
 
Mat exercicios resolvidos
Mat exercicios resolvidosMat exercicios resolvidos
Mat exercicios resolvidos
 
Matematica questoes trigonometria gabarito
Matematica questoes trigonometria gabaritoMatematica questoes trigonometria gabarito
Matematica questoes trigonometria gabarito
 
MatemáTica Estudo Dos Log
MatemáTica Estudo Dos LogMatemáTica Estudo Dos Log
MatemáTica Estudo Dos Log
 
Aula 15 matrizes e determinantes(parte ii)
Aula 15    matrizes e determinantes(parte ii)Aula 15    matrizes e determinantes(parte ii)
Aula 15 matrizes e determinantes(parte ii)
 

Similaire à Matemática 140 questoes resolvidas

Algebra basica
Algebra basicaAlgebra basica
Algebra basicanyltton
 
Resolucao dos exercicios_integrais
Resolucao dos exercicios_integraisResolucao dos exercicios_integrais
Resolucao dos exercicios_integraisWilson Kushima
 
Ita2008 3dia
Ita2008 3diaIta2008 3dia
Ita2008 3diacavip
 
Prova do Colégio Militar do Rio de Janeiro, COMENTADA
Prova do Colégio Militar do Rio de Janeiro, COMENTADAProva do Colégio Militar do Rio de Janeiro, COMENTADA
Prova do Colégio Militar do Rio de Janeiro, COMENTADAthieresaulas
 
Fisica exercicios resolvidos 011
Fisica exercicios resolvidos  011Fisica exercicios resolvidos  011
Fisica exercicios resolvidos 011comentada
 
Ita2009 3dia
Ita2009 3diaIta2009 3dia
Ita2009 3diacavip
 
Exercicios resolvidos
Exercicios resolvidosExercicios resolvidos
Exercicios resolvidostexa0111
 
Exercicio matriz cc_06
Exercicio matriz cc_06Exercicio matriz cc_06
Exercicio matriz cc_06joelcio
 
Lista de exercícios 6 - Mat Elem
Lista de exercícios 6 - Mat ElemLista de exercícios 6 - Mat Elem
Lista de exercícios 6 - Mat ElemCarlos Campani
 
Simave proeb 2011 para 3º ano
Simave proeb 2011 para 3º anoSimave proeb 2011 para 3º ano
Simave proeb 2011 para 3º anoIdelma
 

Similaire à Matemática 140 questoes resolvidas (20)

Stevin
StevinStevin
Stevin
 
Algebra basica
Algebra basicaAlgebra basica
Algebra basica
 
Resolucao dos exercicios_integrais
Resolucao dos exercicios_integraisResolucao dos exercicios_integrais
Resolucao dos exercicios_integrais
 
Resolucao dos exercicios_integrais
Resolucao dos exercicios_integraisResolucao dos exercicios_integrais
Resolucao dos exercicios_integrais
 
Ita2008 3dia
Ita2008 3diaIta2008 3dia
Ita2008 3dia
 
Solução de equaes de 2º grau
Solução de equaes de 2º grauSolução de equaes de 2º grau
Solução de equaes de 2º grau
 
Prova do Colégio Militar do Rio de Janeiro, COMENTADA
Prova do Colégio Militar do Rio de Janeiro, COMENTADAProva do Colégio Militar do Rio de Janeiro, COMENTADA
Prova do Colégio Militar do Rio de Janeiro, COMENTADA
 
Fisica exercicios resolvidos 011
Fisica exercicios resolvidos  011Fisica exercicios resolvidos  011
Fisica exercicios resolvidos 011
 
Ita2009 3dia
Ita2009 3diaIta2009 3dia
Ita2009 3dia
 
Apostila pré cálculo
Apostila pré cálculoApostila pré cálculo
Apostila pré cálculo
 
Gab complexo formatrigonometrica2010
Gab complexo formatrigonometrica2010Gab complexo formatrigonometrica2010
Gab complexo formatrigonometrica2010
 
Remember 08
Remember 08Remember 08
Remember 08
 
Exercicios resolvidos
Exercicios resolvidosExercicios resolvidos
Exercicios resolvidos
 
Exercicio matriz cc_06
Exercicio matriz cc_06Exercicio matriz cc_06
Exercicio matriz cc_06
 
387 matemática ime 2010
387 matemática ime 2010387 matemática ime 2010
387 matemática ime 2010
 
Lista de exercícios 6 - Mat Elem
Lista de exercícios 6 - Mat ElemLista de exercícios 6 - Mat Elem
Lista de exercícios 6 - Mat Elem
 
Lista efomm math aleph
Lista efomm math alephLista efomm math aleph
Lista efomm math aleph
 
Simave proeb 2011 para 3º ano
Simave proeb 2011 para 3º anoSimave proeb 2011 para 3º ano
Simave proeb 2011 para 3º ano
 
Remember 09
Remember 09Remember 09
Remember 09
 
Fatec1 mat
Fatec1 matFatec1 mat
Fatec1 mat
 

Dernier

Slides Lição 5, Betel, Ordenança para uma vida de vigilância e oração, 2Tr24....
Slides Lição 5, Betel, Ordenança para uma vida de vigilância e oração, 2Tr24....Slides Lição 5, Betel, Ordenança para uma vida de vigilância e oração, 2Tr24....
Slides Lição 5, Betel, Ordenança para uma vida de vigilância e oração, 2Tr24....LuizHenriquedeAlmeid6
 
Considere a seguinte situação fictícia: Durante uma reunião de equipe em uma...
Considere a seguinte situação fictícia:  Durante uma reunião de equipe em uma...Considere a seguinte situação fictícia:  Durante uma reunião de equipe em uma...
Considere a seguinte situação fictícia: Durante uma reunião de equipe em uma...azulassessoria9
 
11oC_-_Mural_de_Portugues_4m35.pptxTrabalho do Ensino Profissional turma do 1...
11oC_-_Mural_de_Portugues_4m35.pptxTrabalho do Ensino Profissional turma do 1...11oC_-_Mural_de_Portugues_4m35.pptxTrabalho do Ensino Profissional turma do 1...
11oC_-_Mural_de_Portugues_4m35.pptxTrabalho do Ensino Profissional turma do 1...licinioBorges
 
"É melhor praticar para a nota" - Como avaliar comportamentos em contextos de...
"É melhor praticar para a nota" - Como avaliar comportamentos em contextos de..."É melhor praticar para a nota" - Como avaliar comportamentos em contextos de...
"É melhor praticar para a nota" - Como avaliar comportamentos em contextos de...Rosalina Simão Nunes
 
PRÉDIOS HISTÓRICOS DE ASSARÉ Prof. Francisco Leite.pdf
PRÉDIOS HISTÓRICOS DE ASSARÉ Prof. Francisco Leite.pdfPRÉDIOS HISTÓRICOS DE ASSARÉ Prof. Francisco Leite.pdf
PRÉDIOS HISTÓRICOS DE ASSARÉ Prof. Francisco Leite.pdfprofesfrancleite
 
Nós Propomos! " Pinhais limpos, mundo saudável"
Nós Propomos! " Pinhais limpos, mundo saudável"Nós Propomos! " Pinhais limpos, mundo saudável"
Nós Propomos! " Pinhais limpos, mundo saudável"Ilda Bicacro
 
Ficha de trabalho com palavras- simples e complexas.pdf
Ficha de trabalho com palavras- simples e complexas.pdfFicha de trabalho com palavras- simples e complexas.pdf
Ficha de trabalho com palavras- simples e complexas.pdfFtimaMoreira35
 
PROGRAMA DE AÇÃO 2024 - MARIANA DA SILVA MORAES.pdf
PROGRAMA DE AÇÃO 2024 - MARIANA DA SILVA MORAES.pdfPROGRAMA DE AÇÃO 2024 - MARIANA DA SILVA MORAES.pdf
PROGRAMA DE AÇÃO 2024 - MARIANA DA SILVA MORAES.pdfMarianaMoraesMathias
 
Slides Lição 04, Central Gospel, O Tribunal De Cristo, 1Tr24.pptx
Slides Lição 04, Central Gospel, O Tribunal De Cristo, 1Tr24.pptxSlides Lição 04, Central Gospel, O Tribunal De Cristo, 1Tr24.pptx
Slides Lição 04, Central Gospel, O Tribunal De Cristo, 1Tr24.pptxLuizHenriquedeAlmeid6
 
o ciclo do contato Jorge Ponciano Ribeiro.pdf
o ciclo do contato Jorge Ponciano Ribeiro.pdfo ciclo do contato Jorge Ponciano Ribeiro.pdf
o ciclo do contato Jorge Ponciano Ribeiro.pdfCamillaBrito19
 
2° ano_PLANO_DE_CURSO em PDF referente ao 2° ano do Ensino fundamental
2° ano_PLANO_DE_CURSO em PDF referente ao 2° ano do Ensino fundamental2° ano_PLANO_DE_CURSO em PDF referente ao 2° ano do Ensino fundamental
2° ano_PLANO_DE_CURSO em PDF referente ao 2° ano do Ensino fundamentalAntônia marta Silvestre da Silva
 
Rota das Ribeiras Camp, Projeto Nós Propomos!
Rota das Ribeiras Camp, Projeto Nós Propomos!Rota das Ribeiras Camp, Projeto Nós Propomos!
Rota das Ribeiras Camp, Projeto Nós Propomos!Ilda Bicacro
 
COMPETÊNCIA 2 da redação do enem prodção textual professora vanessa cavalcante
COMPETÊNCIA 2 da redação do enem prodção textual professora vanessa cavalcanteCOMPETÊNCIA 2 da redação do enem prodção textual professora vanessa cavalcante
COMPETÊNCIA 2 da redação do enem prodção textual professora vanessa cavalcanteVanessaCavalcante37
 
JOGO FATO OU FAKE - ATIVIDADE LUDICA(1).pptx
JOGO FATO OU FAKE - ATIVIDADE LUDICA(1).pptxJOGO FATO OU FAKE - ATIVIDADE LUDICA(1).pptx
JOGO FATO OU FAKE - ATIVIDADE LUDICA(1).pptxTainTorres4
 
CRUZADINHA - Leitura e escrita dos números
CRUZADINHA   -   Leitura e escrita dos números CRUZADINHA   -   Leitura e escrita dos números
CRUZADINHA - Leitura e escrita dos números Mary Alvarenga
 
PROVA - ESTUDO CONTEMPORÂNEO E TRANSVERSAL: COMUNICAÇÃO ASSERTIVA E INTERPESS...
PROVA - ESTUDO CONTEMPORÂNEO E TRANSVERSAL: COMUNICAÇÃO ASSERTIVA E INTERPESS...PROVA - ESTUDO CONTEMPORÂNEO E TRANSVERSAL: COMUNICAÇÃO ASSERTIVA E INTERPESS...
PROVA - ESTUDO CONTEMPORÂNEO E TRANSVERSAL: COMUNICAÇÃO ASSERTIVA E INTERPESS...azulassessoria9
 
Música Meu Abrigo - Texto e atividade
Música   Meu   Abrigo  -   Texto e atividadeMúsica   Meu   Abrigo  -   Texto e atividade
Música Meu Abrigo - Texto e atividadeMary Alvarenga
 
Projeto de Extensão - ENGENHARIA DE SOFTWARE - BACHARELADO.pdf
Projeto de Extensão - ENGENHARIA DE SOFTWARE - BACHARELADO.pdfProjeto de Extensão - ENGENHARIA DE SOFTWARE - BACHARELADO.pdf
Projeto de Extensão - ENGENHARIA DE SOFTWARE - BACHARELADO.pdfHELENO FAVACHO
 
ATIVIDADE - CHARGE.pptxDFGHJKLÇ~ÇLJHUFTDRSEDFGJHKLÇ
ATIVIDADE - CHARGE.pptxDFGHJKLÇ~ÇLJHUFTDRSEDFGJHKLÇATIVIDADE - CHARGE.pptxDFGHJKLÇ~ÇLJHUFTDRSEDFGJHKLÇ
ATIVIDADE - CHARGE.pptxDFGHJKLÇ~ÇLJHUFTDRSEDFGJHKLÇJaineCarolaineLima
 
ENSINO RELIGIOSO 7º ANO INOVE NA ESCOLA.pdf
ENSINO RELIGIOSO 7º ANO INOVE NA ESCOLA.pdfENSINO RELIGIOSO 7º ANO INOVE NA ESCOLA.pdf
ENSINO RELIGIOSO 7º ANO INOVE NA ESCOLA.pdfLeloIurk1
 

Dernier (20)

Slides Lição 5, Betel, Ordenança para uma vida de vigilância e oração, 2Tr24....
Slides Lição 5, Betel, Ordenança para uma vida de vigilância e oração, 2Tr24....Slides Lição 5, Betel, Ordenança para uma vida de vigilância e oração, 2Tr24....
Slides Lição 5, Betel, Ordenança para uma vida de vigilância e oração, 2Tr24....
 
Considere a seguinte situação fictícia: Durante uma reunião de equipe em uma...
Considere a seguinte situação fictícia:  Durante uma reunião de equipe em uma...Considere a seguinte situação fictícia:  Durante uma reunião de equipe em uma...
Considere a seguinte situação fictícia: Durante uma reunião de equipe em uma...
 
11oC_-_Mural_de_Portugues_4m35.pptxTrabalho do Ensino Profissional turma do 1...
11oC_-_Mural_de_Portugues_4m35.pptxTrabalho do Ensino Profissional turma do 1...11oC_-_Mural_de_Portugues_4m35.pptxTrabalho do Ensino Profissional turma do 1...
11oC_-_Mural_de_Portugues_4m35.pptxTrabalho do Ensino Profissional turma do 1...
 
"É melhor praticar para a nota" - Como avaliar comportamentos em contextos de...
"É melhor praticar para a nota" - Como avaliar comportamentos em contextos de..."É melhor praticar para a nota" - Como avaliar comportamentos em contextos de...
"É melhor praticar para a nota" - Como avaliar comportamentos em contextos de...
 
PRÉDIOS HISTÓRICOS DE ASSARÉ Prof. Francisco Leite.pdf
PRÉDIOS HISTÓRICOS DE ASSARÉ Prof. Francisco Leite.pdfPRÉDIOS HISTÓRICOS DE ASSARÉ Prof. Francisco Leite.pdf
PRÉDIOS HISTÓRICOS DE ASSARÉ Prof. Francisco Leite.pdf
 
Nós Propomos! " Pinhais limpos, mundo saudável"
Nós Propomos! " Pinhais limpos, mundo saudável"Nós Propomos! " Pinhais limpos, mundo saudável"
Nós Propomos! " Pinhais limpos, mundo saudável"
 
Ficha de trabalho com palavras- simples e complexas.pdf
Ficha de trabalho com palavras- simples e complexas.pdfFicha de trabalho com palavras- simples e complexas.pdf
Ficha de trabalho com palavras- simples e complexas.pdf
 
PROGRAMA DE AÇÃO 2024 - MARIANA DA SILVA MORAES.pdf
PROGRAMA DE AÇÃO 2024 - MARIANA DA SILVA MORAES.pdfPROGRAMA DE AÇÃO 2024 - MARIANA DA SILVA MORAES.pdf
PROGRAMA DE AÇÃO 2024 - MARIANA DA SILVA MORAES.pdf
 
Slides Lição 04, Central Gospel, O Tribunal De Cristo, 1Tr24.pptx
Slides Lição 04, Central Gospel, O Tribunal De Cristo, 1Tr24.pptxSlides Lição 04, Central Gospel, O Tribunal De Cristo, 1Tr24.pptx
Slides Lição 04, Central Gospel, O Tribunal De Cristo, 1Tr24.pptx
 
o ciclo do contato Jorge Ponciano Ribeiro.pdf
o ciclo do contato Jorge Ponciano Ribeiro.pdfo ciclo do contato Jorge Ponciano Ribeiro.pdf
o ciclo do contato Jorge Ponciano Ribeiro.pdf
 
2° ano_PLANO_DE_CURSO em PDF referente ao 2° ano do Ensino fundamental
2° ano_PLANO_DE_CURSO em PDF referente ao 2° ano do Ensino fundamental2° ano_PLANO_DE_CURSO em PDF referente ao 2° ano do Ensino fundamental
2° ano_PLANO_DE_CURSO em PDF referente ao 2° ano do Ensino fundamental
 
Rota das Ribeiras Camp, Projeto Nós Propomos!
Rota das Ribeiras Camp, Projeto Nós Propomos!Rota das Ribeiras Camp, Projeto Nós Propomos!
Rota das Ribeiras Camp, Projeto Nós Propomos!
 
COMPETÊNCIA 2 da redação do enem prodção textual professora vanessa cavalcante
COMPETÊNCIA 2 da redação do enem prodção textual professora vanessa cavalcanteCOMPETÊNCIA 2 da redação do enem prodção textual professora vanessa cavalcante
COMPETÊNCIA 2 da redação do enem prodção textual professora vanessa cavalcante
 
JOGO FATO OU FAKE - ATIVIDADE LUDICA(1).pptx
JOGO FATO OU FAKE - ATIVIDADE LUDICA(1).pptxJOGO FATO OU FAKE - ATIVIDADE LUDICA(1).pptx
JOGO FATO OU FAKE - ATIVIDADE LUDICA(1).pptx
 
CRUZADINHA - Leitura e escrita dos números
CRUZADINHA   -   Leitura e escrita dos números CRUZADINHA   -   Leitura e escrita dos números
CRUZADINHA - Leitura e escrita dos números
 
PROVA - ESTUDO CONTEMPORÂNEO E TRANSVERSAL: COMUNICAÇÃO ASSERTIVA E INTERPESS...
PROVA - ESTUDO CONTEMPORÂNEO E TRANSVERSAL: COMUNICAÇÃO ASSERTIVA E INTERPESS...PROVA - ESTUDO CONTEMPORÂNEO E TRANSVERSAL: COMUNICAÇÃO ASSERTIVA E INTERPESS...
PROVA - ESTUDO CONTEMPORÂNEO E TRANSVERSAL: COMUNICAÇÃO ASSERTIVA E INTERPESS...
 
Música Meu Abrigo - Texto e atividade
Música   Meu   Abrigo  -   Texto e atividadeMúsica   Meu   Abrigo  -   Texto e atividade
Música Meu Abrigo - Texto e atividade
 
Projeto de Extensão - ENGENHARIA DE SOFTWARE - BACHARELADO.pdf
Projeto de Extensão - ENGENHARIA DE SOFTWARE - BACHARELADO.pdfProjeto de Extensão - ENGENHARIA DE SOFTWARE - BACHARELADO.pdf
Projeto de Extensão - ENGENHARIA DE SOFTWARE - BACHARELADO.pdf
 
ATIVIDADE - CHARGE.pptxDFGHJKLÇ~ÇLJHUFTDRSEDFGJHKLÇ
ATIVIDADE - CHARGE.pptxDFGHJKLÇ~ÇLJHUFTDRSEDFGJHKLÇATIVIDADE - CHARGE.pptxDFGHJKLÇ~ÇLJHUFTDRSEDFGJHKLÇ
ATIVIDADE - CHARGE.pptxDFGHJKLÇ~ÇLJHUFTDRSEDFGJHKLÇ
 
ENSINO RELIGIOSO 7º ANO INOVE NA ESCOLA.pdf
ENSINO RELIGIOSO 7º ANO INOVE NA ESCOLA.pdfENSINO RELIGIOSO 7º ANO INOVE NA ESCOLA.pdf
ENSINO RELIGIOSO 7º ANO INOVE NA ESCOLA.pdf
 

Matemática 140 questoes resolvidas

  • 1. Professor: Fabrício Maia Matemática 5 ○ ○ ○ ○ ○ ○ ○ ○ ○ ○ ○ ○ ○ ○ ○ ○ ○ ○ ○ ○ ○ ○ ○ ○ ○ ○ ○ ○ ○ ○ ○ ○ ○ ○ ○ ○ ○ ○ ○ ○ ○ ○ ○ ○ ○ ○ ○ ○ ○ ○ ○ ○ ○ ○ ○ ○ ○ ○ ○ ○ ○ ○ 140 questões resolvidas “A força não provém da capacidade física e sim de uma vontade indomável” (Mahatma Gandhi) 01 Os valores de b para os quais a parábola y = x2 + bx tem um único ponto em comum com a reta y = x – 1 são: A) – 1 e 3 D) 0 e – 1 B) – 1 e 2 E) 0 e 2 C) – 3 e – 1 Solução: Temos: ⎧ = + ⎨ = −⎩ 2 y x bx y x 1 Comparando: + = − + − + = 2 2 x bx x 1 x (b 1)x 1 0 Como as equações têm um único ponto comum, então: Δ = − − ⋅ ⋅ = − = 2 2 0 (b 1) 4 1 1 0 (b 1) 4 Daí: − = → = − = − → = −b 1 2 b 3 ou b 1 2 b 1 Resposta: A 02 Se f(x) = 4x + 1 e g(x) = 4x , a solução da inequação f(x) > g (2 – x) é: A) x > 0 D) x > 1,5 B) x > 0,5 E) x > 2 C) x > 1 Solução: Temos: f(x) > g(2 –x) 4x + 1 > 42 – x (base > 1) Daí: x + 1 > 2 –x > → > 1 2x 1 x 2 Resposta: B 03 log 50 + log 40 + log 20 + log 2,5 é igual a: A) 1 D) 10 B) 3 E) 1.000 C) 5 Solução: Lembre: ⋅ + =b c b c a a alog log log Temos: log 50 + log 40 + log 20 + log 2,5 = log 100000 = log 105 = 5 Soma = 5 Resposta: C “A força não provém da capacidade física e sim de uma vontade indomável” (Mahatma Gandhi) 01 Os valores de b para os quais a parábola y = x2 + bx tem um único ponto em comum com a reta y = x – 1 são: A) – 1 e 3 D) 0 e – 1 B) – 1 e 2 E) 0 e 2 C) – 3 e – 1 Solução: Temos: ⎧ = + ⎨ = −⎩ 2 y x bx y x 1 Comparando: + = − + − + = 2 2 x bx x 1 x (b 1)x 1 0 Como as equações têm um único ponto comum, então: Δ = − − ⋅ ⋅ = − = 2 2 0 (b 1) 4 1 1 0 (b 1) 4 Daí: − = → = − = − → = −b 1 2 b 3 ou b 1 2 b 1 Resposta: A 02 Se f(x) = 4x + 1 e g(x) = 4x , a solução da inequação f(x) > g (2 – x) é: A) x > 0 D) x > 1,5 B) x > 0,5 E) x > 2 C) x > 1 Solução: Temos: f(x) > g(2 –x) 4x + 1 > 42 – x (base > 1) Daí: x + 1 > 2 –x > → > 1 2x 1 x 2 Resposta: B 03 log 50 + log 40 + log 20 + log 2,5 é igual a: A) 1 D) 10 B) 3 E) 1.000 C) 5 Solução: Lembre: ⋅ + =b c b c a a alog log log Temos: log 50 + log 40 + log 20 + log 2,5 = log 100000 = log 105 = 5 Soma = 5 Resposta: C
  • 2. Professor: Fabrício MaiaMatemática 6 04 Utilizando a tabela abaixo, conclui-se que 5 371.293 é igual a: A) 11 B) 13 C) 14 D) 15 E) 17 Solução: Tomando: n = 5 371.293 Daí: log n = log 5 371.293 → log n = 1 5log (371. 293) log n = 1 log371.293 5 ⋅ (veja tabela) → = ⋅ 1 log n 5,55 5 → log n = 1,11 (veja tabela) logo: n = 13 Resposta: B 05 O número de pontos de interseção dos gráficos de y = 3 logx e de y = log 9x, sendo x > 0, é: A) 0 B) 1 C) 2 D) 3 E) 4 Solução: Temos: { = ⋅ = = > < ≠ x a Sabemos : y 3 logx f(x) log y log 9x (x 0 e 0 a 1) Comparando: ⋅ = =3 3 logx log9x logx log9x Daí: = − = − = = − = → = = − 3 3 2 2 x 9x x 9x 0 x(x 9) 0 x 0(n.s) ou x 9 0 x 3 ou x 3(n.s) Resposta: B 06 A equação + +⎛ ⎞ ⎛ ⎞ +⎜ ⎟ ⎜ ⎟ ⎝ ⎠ ⎝ ⎠ = +⎛ ⎞ ⎜ ⎟ ⎝ ⎠ k 1 k 1 2 3 1 k 2 5 A) não admite soluções. B) admite uma solução entre 1 e 5. C) admite uma solução entre 5 e 12. D) admite uma solução entre 12 e 20. E) admite uma solução maior que 20. Solução: Lembre: n n n 1 p p 1 p 1 +⎛ ⎞ ⎛ ⎞ ⎛ ⎞ + =⎜ ⎟ ⎜ ⎟ ⎜ ⎟+ +⎝ ⎠ ⎝ ⎠ ⎝ ⎠ Daí: + + +⎛ ⎞ ⎛ ⎞ ⎛ ⎞ + =⎜ ⎟ ⎜ ⎟ ⎜ ⎟ ⎝ ⎠ ⎝ ⎠ ⎝ ⎠ k 1 k 1 k 2 2 3 3 9 11 13 15 17 ... 371.293 log NN 0,95 1,04 1,11 1,18 1,23 ... 5,55
  • 3. Professor: Fabrício Maia Matemática 7 Substituindo: +⎛ ⎞ ⎜ ⎟ ⎝ ⎠ = +⎛ ⎞ ⎜ ⎟ ⎝ ⎠ + +⎛ ⎞ ⎛ ⎞ =⎜ ⎟ ⎜ ⎟ ⎝ ⎠ ⎝ ⎠ k 2 3 1 k 2 5 k 2 k 2 3 5 logo: + = + → =3 5 k 2 k 6 Resposta: C 07 A soma dos coeficientes do desenvolvimento de (1 + x2 – x3 )9 é: A) – 1 B) 2 C) 1 D) 3 E) 4 Solução: Sabemos: Se p(x) = anxn + an – 1xn – 1 + ... + a1x + a0, com ≠na 0, a soma dos coeficientes do polinômio é dada por p(1). Assim: A soma dos coeficientes de (1 + x2 – x3 )9 é dada por: Scoef. =(1 + 12 – 13 )9 = (1 + 1 – 1)9 = 1 Resposta: C 08 Encontre o coeficiente de x2 no desenvolvimento de (x2 + 2x + 1)4 . Solução: Lembre: Termo geral: − + ⎛ ⎞ = ⋅ ⋅⎜ ⎟ ⎝ ⎠ n p p p 1 n T a b p Temos: (x2 + 2x + 1)4 = [(x + 1)2 ]4 = (x +1)8 Termo geral: − + ⎛ ⎞ = ⋅ ⋅⎜ ⎟ ⎝ ⎠ 8 p p p 1 8 T x 1 p Queremos: 8 – p = 2 → =p 6 Daí: ⎛ ⎞ = ⋅ ⋅ =⎜ ⎟ ⎝ ⎠ 2 6 2 7 8 T x 1 28x 6 Resposta: 28 09 Calcule n sabendo que ⎛ ⎞ ⎛ ⎞ ⎛ ⎞ ⎛ ⎞ + + + + =⎜ ⎟ ⎜ ⎟ ⎜ ⎟ ⎜ ⎟ ⎝ ⎠ ⎝ ⎠ ⎝ ⎠ ⎝ ⎠ n n n n ... 8.191 1 2 3 n Solução: Lembre: ⎛ ⎞ ⎛ ⎞ ⎛ ⎞ ⎛ ⎞ + + + + =⎜ ⎟ ⎜ ⎟ ⎜ ⎟ ⎜ ⎟ ⎝ ⎠ ⎝ ⎠ ⎝ ⎠ ⎝ ⎠ nn n n n ... 2 0 1 2 n Daí: ⎛ ⎞ ⎛ ⎞ ⎛ ⎞ + + + =⎜ ⎟ ⎜ ⎟ ⎜ ⎟ ⎝ ⎠ ⎝ ⎠ ⎝ ⎠ n n n ... 8.191 1 2 n Agora: ⎛ ⎞ = ⋅⎜ ⎟ ⎝ ⎠ − = = = → = n n n n 13 n 2 – 8 191 0 2 1 8.191 2 8.192 2 2 n 13 Resposta: 13 ⎛ ⎞ − ⎜ ⎟ ⎝ ⎠ n n 2 0 14444244443
  • 4. Professor: Fabrício MaiaMatemática 8 10 O número total de pares (x, y) que satisfazem a equação (x2 + y2 – 1)2 + (xy)2 = 0 é: A) infinito B) 0 C) 1 D) 2 E) 4 Solução: ∈ + = ⇔ = = 1 n n Se a,b e n é par, então : a b 0 a b 0 Temos: (x2 + y2 – 1)2 + (xy)2 = 0 Daí: ⎧ + − = ⎨ = → = =⎩ = → = → = ± = → = → = ± − − 2 2 2 2 x y 1 0 xy 0 x 0 ou y 0 se x 0 y 1 y 1 se y 0 x 1 x 1 pares : (0,1),(0, 1),(1,0),( 1,0) Resposta: E 11 A parábola de equação y = x2 – 6 tem vértice M e corta o eixo x nos pontos A e B. Qual a área do triângulo ABM? A) 1 B) 6 C) 6 D) 6 6 E) 12 6 Solução: Lembre: 2 f(x) ax bx c, com a 0= + + ≠ Coordenadas do vértice: = − Δ = − = v v v v b x 2a y ou y f(x ) 4a Temos: – Coordenadas do vértice: y = x2 – 6 − = → = ⋅ = − → = − v v 2 v v 0 x x 0 2 1 y 0 6 y 6 Então: M(0; –6) – Pontos de interseção com o eixo x: 2 x 6 x 6 ou x 6 A( 6; 0) e B( 6, 0) − =→ = = − − Logo, a área do ΔABM é dada por: Área = − = = − ABM ABM 1 |D | 2 0 6 1 |D | 6 0 1 12 6 6 0 1 Área: 1 12 6 6 6 u.a 2 ⋅ = Resposta: D 12 A distância do vértice da parábola y = (x – 2)(x – 6) à reta = + 4 y x 5 é : 3 72 A) 25 29 B) 25 C) 43 43 D) 25 43 E) 5
  • 5. Professor: Fabrício Maia Matemática 9 ax + by + c = 0 P(x , y )o or Solução: I) = + + ≠2 f(x) ax bx c, com a 0 Coordenadas do vértice: +⎧ ⎪ = ⎨ ⎪ =⎩ 1 2 v v v x x x 2 y f(x ) II) Distância de um ponto a uma reta. + + = + 0 0 p,r 2 2 | ax by c | d a b Temos: = − ⋅ − → + = = = = − ⋅ − = ⋅ − = − v v y (x 2) (x 6) parábola raízes : 2 e 6 2 6 x 4 2 y f(4) (4 2) (4 6) 2 ( 2) 4 Distância do vértice à reta: (4, – 4) 4x – 3y + 15 = 0 d = ??? ⋅ − ⋅ − + = + − = 2 2 | 4 4 3 ( 4) 15 | d 4 ( 3) 43 d 5 Resposta: E 13 Resolvendo a inequação log1/2(2x + 1) > log1/2(– 3x + 4), obtemos: − < < 1 4 A) x 2 3 < < 4 B) 0 x 3 < 3 C) x 5 − < < 1 3 D) x 2 5 < < 3 4 E) x 5 3 Solução: Lembre: > → < < < x y a alog log x y (0 base 1) Temos: + > − +1 1 2 2 log (2x 1) log ( 3x 4) Então: ⎧ + < − + → < → <⎪ ⎪⎪ + > → > −⎨ ⎪ ⎪− + > → < ⎪⎩ 3 2x 1 3x 4 5x 3 x 5 1 2x 1 0(C.E) x 2 4 3x 4 0 (C.E) x 3
  • 6. Professor: Fabrício MaiaMatemática 10 Interseção Resposta: D 14 Se o número complexo z = 1 – i é uma das raízes da equação x10 – a = 0, o valor de a é: A) 16 B) 32 C) 64 D) – 16i E) – 32i Solução: Temos: x10 = a, se z é raiz, então z10 = a. Daí: − = − = − = − → = − 10 2 5 5 5 a = (1 i) a [(1 i) ] a ( 2i) a 32i a 32i Resposta: E 15 A reta y = ax + 1 intercepta a curva x2 + 4y2 = 1 somente num ponto. Calcule 8a2 . Solução: Temos: ⎧ ⎨ + =⎩ 2 2 y = ax + 1 x 4y 1 Condição: Δ = 0 (única solução) Daí: 2 2 2 2 2 2 (8a) 4 (1 4a ) 3 0 64a 12 48a 0 16a 12 8a 6 − ⋅ + ⋅ = − − = = = Resposta: 6 16 A condição para que o trinômio mx2 + (m + 1)x + 1 seja sempre positivo, qualquer que seja x, é que: A) m > 0 B) (m + 1)2 + 4m < 0 C) − ≤2 (m 1) 0 D) ≠ >m 1,m 0 E) não há valores de m tais que o trinômio proposto, qualquer que seja x, se torne sempre positivo. Solução: Devemos ter: 1ª condição: a > 0 → m > 0 2ª condição: Δ < → + − ⋅ ⋅ < → → + + − < → − + < → → − < − ≥ ∀ ∈ 1 2 2 2 2 2 0 (m 1) 4 m 1 0 m 2m 1 4m 0 m 2m 1 0 (m 1) 0(absurdo, pois,(m 1) 0, m ) Resposta: E 3 5 (I) (II) 4 3 (III) (I) Δ (II) Δ (III) 1 2 − 3 5 −1 2 + + + + + + {Δ < > 0 a 0 Substituindo (I) em (II): + + = + + + − = + + + = 2 2 2 2 2 2 2 x 4(ax 1) 1 x 4a x 8ax 4 1 0 x (1 4a ) 8ax 3 0 (Equação do 2o grau)
  • 7. Professor: Fabrício Maia Matemática 11 17 Sejam A = {1, 2, 3} e f: A → A definida por f(1) = 3, f(2) = 1 e f(3) = 2. O conjunto-solução de f[f(x)] = 3, é: A) {1} B) {2} C) {3} D) {1, 2, 3} E) vazio Solução: Temos: se x 1 f(f(1)) f(3) 2 f(f(1)) 2(n.s) se x 2 f(f(2)) f(1) 3 f(f(2)) 3(OK) se x 3 f(f(3)) f(2) 1 f(f(3)) 1(n.s) S {2} = → = = → = = → = = → = = → = = → = = Resposta: B 18 Seja S a soma, em radianos, das raízes da equação 1 + cos x + cos 2x + cos 3x = 0, x ∈[0, π]. Calcule π 6S . Solução: Fórmula de Werner: + −⎛ ⎞ ⎛ ⎞ + = ⎜ ⎟ ⎜ ⎟ ⎝ ⎠ ⎝ ⎠ p q p q cosp cosq 2cos cos 2 2 Temos: ⎧ ⎛ ⎞ ⎛ ⎞ + = + = ⋅⎜ ⎟ ⎜ ⎟⎪⎪ ⎝ ⎠ ⎝ ⎠ ⎨ ⎛ ⎞ ⎛ ⎞⎪ + = ⎜ ⎟ ⎜ ⎟⎪ ⎝ ⎠ ⎝ ⎠⎩ x x 1 cosx cos0 cosx 2cos cos 2 2 5x x cos2x cos3x 2cos cos 2 2 Então: Daí: π = → = + π → = π + π π π π = → = + π → = + π = → = = π π π = → = π x x cos 0 k x 2k 2 2 2 ou 3x 3x 2k cos 0 k x 2 2 2 3 3 ou cosx 0 x k 2 se k 0 x , ou 3 2 se k = 1, 2, 3, ... → raízes já encontradas ou fora do intervalo dado. Raízes: π, π π , 2 3 Soma das raízes = π π π + π + π = π + + = π = 6 3 2 S 2 3 6 11 S 6 = π 6S Logo : 11 f− = ∀ ∈ Importante : cos( x) cos(x), x D Resposta: 11 1 2 3 1 2 3 A A ⎛ ⎞ ⎛ ⎞ ⎛ ⎞ ⎛ ⎞ + =⎜ ⎟ ⎜ ⎟ ⎜ ⎟ ⎜ ⎟ ⎝ ⎠ ⎝ ⎠ ⎝ ⎠ ⎝ ⎠ ⎡ ⎤⎛ ⎞ ⎛ ⎞ ⎛ ⎞ ⋅ + =⎜ ⎟ ⎜ ⎟ ⎜ ⎟⎢ ⎥ ⎝ ⎠ ⎝ ⎠ ⎝ ⎠⎣ ⎦ ⎛ ⎞ ⎛ ⎞ ⋅ ⋅ − =⎜ ⎟ ⎜ ⎟ ⎝ ⎠ ⎝ ⎠ ⎛ ⎞ ⎛ ⎞ =⎜ ⎟ ⎜ ⎟ ⎝ ⎠ ⎝ ⎠ x x 5x x 2cos cos 2cos cos 0 2 2 2 2 x x 5x 2cos cos cos 0 2 2 2 x 3x 2cos 2 cos cos( x) 0 2 2 x 3x 4cos cos cos(x) 0 2 2
  • 8. Professor: Fabrício MaiaMatemática 12 19 A função f(x) = − − 3 x(1 x) x 2 4 é crescente, para todo x pertencente a: 25 A) , 16 ⎡ ⎞ − +∞⎟⎢ ⎣ ⎠ 25 B) , 4 ⎡ ⎞ − +∞⎟⎢ ⎣ ⎠ 5 C) , 2 ⎡ ⎞ − +∞⎟⎢ ⎣ ⎠ 5 D) , 4 ⎡ ⎞ −∞ ⎟⎢ ⎣ ⎠ 1E) Solução: Temos: 2 2 2 2 3 x – x f(x) x 2 4 6x x x f(x) 4 x 5x f(x) 4 1 5 f(x) x x 4 4 = − − + = + = = + Parábola Resposta: C 20 Se p e q são raízes não-nulas de x2 + 5px – 8q = 0, calcule p + q. Solução: Girard { + = − ⋅ = − 1 2 1 2 x x 5p x x 8q Daí: {p q 5q p q 8q + = − ⋅ = − 2ª equação p q 8q⋅ = − (como q é diferente de zero) Temos: = − + = − + = − ⋅ − + = p 8 Logo : p q 5p p q 5 ( 8) p q 40 Resposta: 40 21 Quantos valores inteiros satisfazem a inequação (2x – 7) (x – 1) ≤ 0. A) zero B) 1 C) 2 D) 3 E) 4 crescente − = = − ⋅ v 5 54x 1 22 4 f ∀ ≥ − 5 é crescente x 2
  • 9. Professor: Fabrício Maia Matemática 13 Solução: Inteiros: 1, 2, 3 Resposta: D 22 Sobre a equação 1.983x2 – 1.984x – 1.985 = 0, a afirmativa correta é: A) não tem raízes. D) tem duas raízes positivas. B) tem duas raízes simétricas. E) tem duas raízes negativas. C) tem duas raízes reais distintas. Solução: Temos: Δ = − Δ = − − ⋅ − Δ = + ⋅ 2 2 2 B 4ac ( 1984) 4.1983 ( 1985) 1984 4.1983 1985 Δ > → Então : 0 Resposta: C 23 Seja f uma função real tal que f (x + 1) = (f (x))2 e f (0) = 10. Então f (4) é igual a: A) 1016 B) 100 C) 10258 D) 101 E) 121 Solução: Temos que: Resposta: A 24 Se o domínio da função f, definida por f(x) = 1 – 2x, é o intervalo ]–3, 2], então Imf é: A) ]–7, 3] B) [–3, 7[ C) ]–3, 7] D) [–3, 5[ E) ]–3, 3] Solução: Temos: 1 y y 1 2x x 2 − = − → = Veja: x ] 3,2]∈ − Então: 3 x 2− < ≤ Agora : 1 y 3 2 2 6 1 y 4 7 y 3 7 y 3 ou 3 y 7 y [ 3,7[ − − < ≤ − < − ≤ − < − ≤ > ≥ − − ≤ < → ∈ − Resposta: B – – – + + – – – 7 2 + + + + + + 1 + + 7 2 1 – – produto Estudo do sinal raízes reais e distintas 2 2 2 2 4 2 8 2 16 f(x + 1) = [f(x)] se x 0 f(1) [f(0)] f(1) 10 se x 1 f(2) [f(1)] f(2) 10 se x 2 f(3) [f(2)] f(3) 10 se x 3 f(4) [f(3)] f(4) 10 = → = → = = → = → = = → = → = = → = → =
  • 10. Professor: Fabrício MaiaMatemática 14 25 Se f(2x + 3) = 4x2 + 6x + 1, x ,∀ ∈ 1 então f(1 – x) vale: A) 2 – x2 B) 2 + x2 C) x2 + 2x – 4 D) 3x2 – 2x + 4 E) x2 + x – 1 Solução: Tomando: k 3 2x 3 k x 2 − + = → = Então: − −⎛ ⎞ ⎛ ⎞ = + +⎜ ⎟ ⎜ ⎟ ⎝ ⎠ ⎝ ⎠ = − + − + = − + 2 2 2 k 3 k 3 f(k) 4 6 1 2 2 f(k) (k 3) 3(k 3) 1 f(k) k 3k 1 Agora: − = − − − + − = + − − = + − 2 2 2 f(1 k) (1 k) 3(1 k) 1 f(1 k) k k 1 Portanto : f(1 x) x x 1 Resposta: E 26 A distância do centro da circunferência x2 + y2 – 6x – 8y + 21 = 0 à bissetriz do Iº e IIIº quadrantes, vale: A) 5 B) 2 C) 3 D) 3 2 E) 2 2 Solução: Circunferência x2 + y2 – 2ax – 2by + a2 + b2 – R2 = 0 Centro (a,b) Daí: – 2a = – 6 → a = 3 – 2b = – 8 → b = 4 Bissetriz dos quadrantes ímpares 2 2 |1 3 1 4 0 | 1 2 distância 221 ( 1) ⋅ − ⋅ + = = = + − Resposta: E 27 A reta y = 3 x 3 é tangente a uma circunferência do centro (2, 0). O raio dessa circunferência é: A) 3 B) 2 C) 3 D) 1 E) 0,5 x y 0− = C(3,4) y x=
  • 11. Professor: Fabrício Maia Matemática 15 Solução: distância de um ponto a uma reta. ⋅ − ⋅ + = = + − = = 2 2 | 3 2 3 0 0 | 2 3 R 12( 3) ( 3) 2 3 R 2 3 R 1 Resposta: D 28 Se S = 1! + 2! + 3! + ... + 89!, então o dígito das unidades de S é: A) 1 B) 3 C) 5 D) 7 E) 9 Solução: Veja: 1! = 1; 2! = 2; 3! = 6; 4! = 24 A partir de 5!, os resultados serão múltiplos de 10. Então: S = 1 + 2 + 6 + 24 + 5! + ... + 89! 33 múltiplo de 10 S = 3 + 30 + 10α’ múltiplo de 10 S = 3 + 10α → dígito das unidades é 3. Resposta: B 29 O sistema linear de equações nas incógnitas x e y {kx 2y 1 2x y m + = − − = é impossível se, e somente se: A) k = – 4 e m 1 2 ≠ B) 1 k 4 e m 2 ≠ − = C) 1 k 4 e m 2 ≠ − ≠ D) = −k 4 E) = − = 1 k 4 e m 2 Solução: Sistema {kx 2y 1 4x 2y 2m + = − − = Somando: (k 4)x 2m 1+ = − Impossível + = → = −⎧ ⎪ ⎨ − ≠ → ≠⎪⎩ k 4 0 k 4 1 2m 1 0 m 2 Resposta: A 30 Em um triângulo retângulo OAB, retângulo em O, com OA = a e OB = b, são dados pontos P em OA e Q em OB de tal maneira que AP = PQ = QB = x. Nessas condições o valor de x é: A) ab a b− − B) a b 2ab+ − C) 2 2 a b+ D) a b 2ab+ + E) ab a b+ + (2,0) 3x 3y 0− = 1442443144424443 14243
  • 12. Professor: Fabrício MaiaMatemática 16 Solução: Pitágoras = − + − = − + + − + − + + + = Δ = − + − ⋅ + Δ = + − + Δ = 2 2 2 2 2 2 2 2 2 2 2 2 2 2 2 2 2 x (a x) (b x) x a 2ax x b 2bx x x 2(a b)x a b 0 [ 2(a b)] 4 1(a b ) 4(a b) 4(a b ) 8ab Daí: + ± + ± = = = + + = + − 2 2(a b) 8ab 2(a b) 2 2ab x 2 2 x a b 2ab (absurdo, veja figura) ou x a b 2ab Resposta: B 31 Num triângulo retângulo de catetos 1 e 3cm , a altura relativa à hipotenusa mede, em cm: A) 2 B) 3 C) 3 D) 3 2 E) 2 2 Solução: Temos: Relações métricas I) 2 2 2 2 a 1 ( 3) a 4 a 2= + → = → = II) 3 1 3 a h 3 2h h 2 ⋅ = ⋅ → = → = Resposta: D 32 Sendo f(x) = 100x + 3, o valor de − − − − 8 3 8 3 f(10 ) f(10 ) 10 10 é: A) 104 B) 102 C) 10 D) 10–5 E) 10–11 Solução: Saiba: Se f(x) = ax + b, com a ≠ 0, então: f(s) f(v) a, com s v. s v − = ≠ − Assim, 8 3 8 3 f(10 ) f(10 ) 100 10 10 − − − = − (coeficiente angular) Resposta: B 33 Se um polígono convexo de n lados tem 54 diagonais, então n é: A) 8 B) 9 C) 10 D) 11 E) 12 Solução: Lembre: Fórmula do número de diagonais n(n 3) d 2 − = . Então: Resposta: E B o A P Q b – x a – x x x B C h A a 1 3 2 n(n – 3) 54 2 n – 3n –108 0 n 12 ou n –9 (n.s) = = = =
  • 13. Professor: Fabrício Maia Matemática 17 34 O polígono convexo cuja soma dos ângulos internos mede 1440º tem, exatamente: A) 15 diagonais B) 20 diagonais C) 25 diagonais D) 30 diagonais E) 35 diagonais Solução: Lembre: Soma dos ângulos internos Si = (n – 2) ⋅ 180º Então: 1440º = (n 2) 180º 1400º n 2 180º 8 n 2 n 10 − ⋅ = − = − = Portanto: n(n 3) 10 7 d 2 2 d 35 − ⋅ = = = Resposta: E 35 Na figura, ABCD é um quadrado e BCE é um triângulo eqüilátero. A medida do ângulo AEB, em graus, é: A) 30 D) 75 B) 49 E) 90 C) 60 Solução: Figura: ΔBCE é eqüilátero → α = 60º ABCD é um quadrado → 30ºθ = Veja: BC BE≡ (lado do quadrado = lado do ΔBCE) Daí: ΔABE é isósceles Resposta: D 36 Na figura abaixo, EFG é um triângulo retângulo, EF 2cm= , EG 6cm e EP PQ QG.= = = Então α + β + θ é igual a: A) 3 π B) 7 18 π C) 4 9 π D) 2 π E C D B A E C D B A α θ x 2x 180º 30º 2x 180º x 75º θ + = + = = A E x θ x B α β θ F P Q GE
  • 14. Professor: Fabrício MaiaMatemática 18 Solução: 2 EPF tg tg 1 45º 2 2 1 EQF tg tg 4 2 2 1 EGF tg tg 6 3 Δ → α = → α = →α = Δ → β = → β = Δ → θ = → θ = Sabemos: tg tg tg( ) 1 tg tg β + θ β + θ = − β⋅ θ Então: 1 1 5 2 3 6tg( ) 1 1 1 5 1 2 3 6 tg( ) 1 45º + β + θ = = = − ⋅ β + θ = → β + θ = Portanto: 90ºα +β+ θ = Resposta: D 37 A área compreendida entre as retas 4y = x – 2, y = 2x – 4 e x = 0 é igual a: A) 3,0 u.a B) 3,5 u.a C) 4,0 u.a D) 4,5 u.a E) 6,0 u.a Solução: Temos: Gráfico: Logo: Área = 1 7 7 u.a 2 2 ⋅ = Resposta: B F E α β θ P Q G2 2 2 2 x 2 y 4 y 2x 4 x 0 (eixo y) −⎧ =⎪ ⎪ = −⎨ ⎪ = ⎪ ⎩ retas 1 A(0, );B(0, 4) e C(2,0) 2 − − A C B y x y 2x 4= − x 2 y 4 − = ABC 1 0 1 2 D 0 4 1 1 8 7 2 0 1 − = − = − + =
  • 15. Professor: Fabrício Maia Matemática 19 38 A razão de uma progressão geométrica, cujos termos são os três lados de um triângulo retângulo é: 1 5 A) 2 + 1 2 B) 5 + 1 3 C) 2 + 1 2 D) 3 + Solução: x P.G. ;x;xq q ⎛ ⎞ ⎜ ⎟ ⎝ ⎠ Pitágoras Dividindo ambos os membros por x2 2 2 4 2 2 2 1 q 1 q q q 1 0 1 5 1 5 q q 2 2 = + − − = ± + = → = Portanto: 1 5 q 2 + = Atenção!!! q é positivo. Resposta: A 39 Sejam a e b números reais. Se a > b > 0, a2 – b2 = 4 e log2(a + b) – log3(a – b) = 2, então a2 + b2 é igual a: A) 13/2 B) 15/2 C) 17/2 D) 19/2 Solução: Fazendo: x 2 y 3 log (a b) x a b 2 log (a b) y a b 3 + = → + = − = → − = Sistema: x y 2 3 4 x y 2 x 2 y ⎧ ⋅ = ⎨ − = → = +⎩ Substituindo: x y 2 y y 2 y y y y 2 3 4 2 3 4 2 2 3 4 4 6 4 6 1 y 0 x 2 + ⋅ = ⋅ = ⋅ ⋅ = ⋅ = = → = → = Assim: {a b 4 a b 1 + = − = Resolvendo: = = 5 3 a e b 2 2 Logo: 2 2 34 17 a b 4 2 + = = Resposta: C C A x B xqx q 2 2 2 2 2 2 2 2 x (xq) x q x x q x q ⎛ ⎞ = + ⎜ ⎟ ⎝ ⎠ = +
  • 16. Professor: Fabrício MaiaMatemática 20 40 Se x1 e x2 são as raízes da equação x x2log 3 log (3x) 3 x ,= então 9(x1 + x2) é igual a: A) 22 B) 24 C) 26 D) 28 Solução: Lembre: alog b x a I) a b II)Se log b x a b = = → = Temos: x x2log 3 log 3x 3 x= Então: x2log 3 3 3x= Tomando: k 3 x 1 log x k log 3 e x 3 k = → = = Substituindo: 1 2. kk 2 k 1k 3 3 3 3 3 + = ⋅ = Comparando: 2 2 1 1 2 2 k 1 k k k 2 0 1 k 2 x 3 x 9 ou k 1 x 3 3 Logo : 1 9(x x ) 9 3 1 27 28 9 − + = + − = = − → = → = = → = →= ⎛ ⎞ + = + = + =⎜ ⎟ ⎝ ⎠ Resposta: D 41 O número de raízes de equação 3 2 + cos x = 0 é: A) 0 B) 1 C) 2 D) 3 E) maior que 3 Solução: Sabemos: 1 cosx 1, x.− ≤ ≤ ∀ Temos: Resposta: A 42 O número de raízes da equação tg2 x – sen2 x = 0, 0 x 2 ,≤ < π é: A) 0 B) 1 C) 2 D) 3 E) maior que 3 Solução: Temos: Daí: senx 0 x 0 ou x = = = π Resposta: C 3 cos x 0 2 3 cosx 2 cosx 1,5 + = = − = − (absurdo, pois o mínimo de cos x é – 1) 2 2 2 2 2 2 2 2 2 2 4 sen x sen x 0 cos x sen x sen xcos x 0 sen x(1 cos x) 0 sen x sen x 0 sen x 0 − = − = − = ⋅ = =
  • 17. Professor: Fabrício Maia Matemática 21 43 Determine n, sabendo que n p p 0 n 2 729 p= ⎛ ⎞ ⋅ =⎜ ⎟ ⎝ ⎠ ∑ Solução: n n n p p p 0 n (a b) a b p − = ⎛ ⎞ + = ⋅ ⋅⎜ ⎟ ⎝ ⎠ ∑ (binômio de Newton) Veja: n n p p n n p 0 n 1 2 (1 2) 3 p − = ⎛ ⎞ ⋅ ⋅ = + =⎜ ⎟ ⎝ ⎠ ∑ Então: n 3 729 n 6= → = Resposta: 6 44 O domínio real da função f(x) = 2senx 1− para 0 x 2 ,≤ < π é: 5 A) x 6 6 π π ≤ ≤ 5 B) 0 x ou x 2 6 6 π π ≤ ≤ ≤ < π C) 0 x≤ < π 2 D) x 3 3 π π ≤ ≤ Solução: Condição: 2senx 1 0 1 senx 2 − ≥ ≥ Daí: 5 x 6 6 π π ≤ ≤ Resposta: A 45 Seja M um conjunto de 20 elementos. O número de subconjuntos de M que contêm exatamente 18 elementos é: A) 360 B) 190 C) 180 D) 120 E) 18 Solução: Temos: M = {a1, a2, a3 ..., a20}. A ordem dos elementos não altera um conjunto. Daí: nº de subconjuntos com 18 elementos = C20, 18 Resposta: B 46 Se Cn, 2 + 2.An,2 + 100 = A2n, 2, então n é igual a: A) 24 B) 8 C) 6 D) 10 E) 25 3 − Solução: Temos: n,2 n,2 2n,2 n! n (n 1) C (n 2)!2! 2 n! A n (n 1) (n 2) (2n)! A (2n) (2n 1) (2n 2)! ⋅ − = = − = = ⋅ − − = = ⋅ − − Então: Resposta: B 2 2 2 2 n(n 1) 2 n(n 1) 100 (2n)(2n 1) 2 n n 4n 4n 200 8n 4n 3n n 200 0 25 n 8 ou n (n.s) 3 − + ⋅ − + = − − + − + = − + − = = = − 150º c 30º s 1 2
  • 18. Professor: Fabrício MaiaMatemática 22 47 Deseja-se acondicionar em um certo número de caixas, 1590 bolinhas brancas, 1060 amarelas e 583 azuis, de modo que cada caixa contenha bolinhas de todas as cores. Calcule o número máximo de caixas de modo que qualquer dessas caixas contenha, para cada cor, quantidades iguais de bolinhas. Solução: x → número de caixas. p → quantidade de bolas brancas em cada caixa. q → quantidade de bolas amarelas em cada caixa. r → quantidade de bolas azuis em cada caixa. Temos: Veja: x m.d.c. (1590,1060,583) x 53 = = * MDC (1590, 1060) = 530 * MDC (530, 583) = 53 Resposta: 53 48 Sejam N o conjunto dos números inteiros positivos e E = {(x,y) ∈ N2 ; x4 y4 – 10x2 y2 + 9 = 0}. Determine o número de elementos de E. Solução: Temos: 4 4 2 2 2 2 2 2 2 x y 10x y 9 0 (x y ) 10x y 9 0 − + = − + = Fazendo: 2 2 x y k= Equação: 2 2 2 2 k 10k 9 0 k 1 x y 1 (xy) 1 − + = = → = → = ou 2 2 2 k 9 x y 9 (xy) 9= → = → = Como x e y são inteiros positivos, tem-se: xy 1 (1,1) ou xy 3 (1,3) ou (3,1) E {(1,1),(1,3),(3,1)} = → = → = Resposta: 3 49 Considere a função real definida por 2x 3 3 f(x) ,x 1 1 2x 3 2 + = ≠ − + . Então o valor da soma 1 f(1) 2 f(2) 3 f(3) ... 20 f(20)⋅ + ⋅ + ⋅ + + ⋅ é: A) 120 B) 600 C) 210 D) 620 E) 1.260 1590 p x 1060 q x 583 r x = = = 1060 1 2 530 0 1590 530 583 530 1 10 53 0
  • 19. Professor: Fabrício Maia Matemática 23 Solução: Temos: 2x 3 6 f(x) 2x 3 2x 3 2x 3 6 + = = + ⋅ + + Então: f(x) 6= Agora: Soma: Soma: Soma: Resposta: E 50 Sejam x e y números reais satisfazendo as equações logy x + logx y = 2 e x2 y + y2 = 12x. Determine o valor do produto xy. Solução: Tomando: = → =y x 1 log x m log y m Daí: + = − + = = → = 2 1 m 2 m m 2m 1 0 m 1 x y Substituindo na 2ª equação Resposta: 9 51 Os conjuntos A e B possuem 3 e 4 elementos, respectivamente. Quantas funções de A em B têm o conjunto imagem igual a B? A) Nenhuma B) 34 C) 43 D) 3! E) 4! Solução: Veja: I) Numa função de A em B devemos ter todos os elementos de A associados a um único valor em B. II) Se o conjunto imagem é o próprio B, então existe um elemento em A com duas imagens, pois todos os elementos de A estão associados. Portanto, não existem funções de A em B sobrejetoras. Resposta: A 52 As funções injetoras de A = {1, 2, 3, 4} em B = {5, 6, 7, 8, 9, 0} são em número de: A) 720 B) 360 C) 15 D) 24 E) 30 Solução: Lembre: Se f é injetora, então: 1 2 1 2x x f(x ) f(x )≠ → ≠ Daí: Pelo princípio fundamental da contagem, tem-se 6.5.4.3 = 360 funções injetoras Resposta: B Soma 1.260= 1 6 2 6 3 6 ... 20 6 (1 2 3 ... 20) 6 (1 20) 20 6 21 10 6 2 ⋅ + ⋅ + ⋅ + + ⋅ + + + + ⋅ + ⋅ ⋅ = ⋅ ⋅ → 2 2 2 2 3 2 2 x y y 12x x x x 12x x x 12x 0 x(x x 12) 0 x 0 (n.s) ou x 4(n.s) ou x 3 y 3 + = ⋅ + = + − = + − = = = − = → = f(1): 6 possibilidades em B f(2): 5 possibilidades em B f(3): 4 possibilidades em B f(4): 3 possibilidades em B 1 2 3 4 A
  • 20. Professor: Fabrício MaiaMatemática 24 53 Para ser aprovado numa disciplina, um aluno precisa ter média maior ou igual a 50, obtida num conjunto de 5 provas, sendo quatro parciais, com peso 1 (um) cada, e uma prova-exame, com peso 2 (dois). Um certo aluno obteve em matemática, nas quatro provas parciais, notas iguais a 30, 60, 50 e 70. Esse aluno, para ser aprovado nessa disciplina, deverá obter, na prova-exame, nota mínima igual a: A) 20 B) 35 C) 40 D) 45 E) 50 Solução: Média ponderada = 1 30 1 60 1 50 1 70 2 x 210 2x 6 6 ⋅ + ⋅ + ⋅ + ⋅ + ⋅ + = Temos: min 210 2x 50 6 210 2x 300 2x 90 x 45 x 45 + ≥ + ≥ ≥ ≥ → = Resposta: D 54 O resto da divisão do inteiro n por 12 é igual a 7. O resto da divisão n por 4 é: A) 0 B) 1 C) 2 D) 3 E) 4 Solução: Temos: Daí: n 12q 7 n 12q 4 3 n 4(3q 1) 3 n 4q' 3, onde q' 3q 1 = + = + + = + + = + = + Veja: Resposta: D 55 Qual dos cinco números relacionados abaixo não é um divisor de 1015 ? A) 25 B) 50 C) 64 D) 75 E) 250 Solução: Temos: 15 15 15 15 10 (2 5) 2 5= ⋅ = ⋅ Veja: (A) 25 = 52 divide 1015 (OK) (B) 50 = 2⋅ 52 divide 1015 (OK) (C) 64 = 26 divide 1015 (OK) (D) 75 = 3⋅ 52 não divide 1015 (problema fator 3) (B) 250 = 2⋅ 53 divide 1015 (OK) Resposta: D 56 A fração geratriz de 3,74151515... é: 37.415 A) 10.000 3.741.515 B) 10.000 37.041 C) 9.900 37.041 D) 9.000 370.415 E) 99.000 Solução: Temos: 37.415 374 37.041 3,7415 9.900 9.900 − = = Resposta: C n 7 q 12 n 3 4 q’
  • 21. Professor: Fabrício Maia Matemática 25 57 Se A e B são conjuntos, A – (A – B) é igual a: A) A B) B C) A – B D) A ∪ B E) A ∩ B Solução: Lembre: A B {x / x A e x B}− = ∈ ∉ É fácil ver: A – (A – B) = A ∩ B Resposta: E 58 O retângulo abaixo de dimensões a e b está decomposto em quadrados. Qual o valor da razão a ? b 5 A) 3 2 B) 3 C) 2 3 D) 2 1 E) 2 Solução: Sendo x a medida do lado do menor quadrado, os outros quadrados terão seus lados com as medidas indicadas na figura. Assim: a = 5x e b = 3x Portanto, a 5 b 3 = Resposta: A 59 A equação x4 + ax3 + bx2 + cx + d = 0, de coeficientes reais, admite as raízes 2 – i e 3 + 2i. Então d é: A) 75 B) 65 C) 25 D) 15 E) 10 Solução: Sabemos que: Se os coeficientes de um polinômio P(x) são reais, então: a + bi raiz de P(x) → a – bi também é: Temos: 2 – i raiz → 2 + i também é. 3 + 2i raiz → 3 – 2i também é. Daí, aplicando Girard na equação: Resposta: B B A–B A b a x 1442443 14444244443 1424314243 x x 2x 2x 2x 3x 5x 3x 1 2 3 4 2 2 x .x .x .x d (2 i) (2 i) (3 2i) (3 2i) d (4 i ) (9 4i ) d 5 13 d d 65 = − ⋅ + ⋅ + ⋅ − = − ⋅ − = ⋅ = =
  • 22. Professor: Fabrício MaiaMatemática 26 60 O número de soluções reais da equação 2 2 2x 8x x x 4x − = − é: A) 0 B) 1 C) 2 D) 3 E) 4 Solução: Temos: 2 2 2 3 2 3 2 2 2x 8x x x 4x 2x 8x x 4x x 6x 8x 0 x(x 6x 8) 0 x 0 (n. serve) denominador nulo − = − − = − − + = − + = = → ou 2 x 6x 8 0 ou x 2 ou x 4 (n. serve) denominador nulo S {2} − + = = = → = Resposta: B 61 Determine o número de soluções reais da equação 2x = 2log x . A) Nenhuma B) Uma C) Duas D) Três E) Infinitas Solução: Graficamente: Como não existe interseção, a equação não admite soluções. Resposta: A 62 Se n é o maior número inteiro pertencente ao domínio da função 2f(x) 1 log x= − , determine o valor de n3 + 3n2 + 2. A) 2 B) 20 C) 21 D) 22 E) 32 Solução: Domínio → campo de existência → condição de existência da função → 1 2 21 log x 0 log x 1 0 x 2 0 x 2− ≥ → ≤ → < ≤ → < ≤ → maior inteiro x = 2. Logo, a expressão é igual a 3 2 2 3 2 2 22+ ⋅ + = . Resposta: D y x 0 2y log x= x y 2=
  • 23. Professor: Fabrício Maia Matemática 27 63 Dado x 1≠ e positivo, calcule o valor de 1 Lnxx e . A) 0 B) 1 C) 2 D) 3 E) 4 Solução: Sabemos que: alog b a b e I) a b 1 II) log b log a III)Lx Lnx log x = = = = Então, a expressão dada 1 Lnxx e é igual a: xlog e x e Exp. 1 e e = = = . Resposta: B 64 Prove que 10 10 10log c log a log b a b c 1 b c a ⎛ ⎞ ⎛ ⎞ ⎛ ⎞ ⋅ ⋅ =⎜ ⎟ ⎜ ⎟ ⎜ ⎟ ⎝ ⎠ ⎝ ⎠ ⎝ ⎠ . Prova: Tomemos: x 10 y 10 z 10 log a x 10 a log b y 10 b log c z 10 c = → = = → = = → = Então: c.q.p 65 Determine o produto das soluções reais da equação 3log x 3 9 x x⋅ = . A) 4 B) 8 C) 25 D) 27 E) 90 Solução: Tomemos: k 3log x k 3 x= → = Assim: 2 k k k 3 k 2 3k 2 2 9 (3 ) (3 ) 3 3 k 2 3k k 3k 2 0 k 1 x 3 ou k 2 x 9 + ⋅ = = → + = → − + = = → = = → = Portanto, o produto das soluções é 27. Resposta: D ⎛ ⎞ ⎛ ⎞ ⎛ ⎞ = ⋅ ⋅⎜ ⎟ ⎜ ⎟ ⎜ ⎟ ⎝ ⎠ ⎝ ⎠ ⎝ ⎠ z x yx y z y z x 10 10 10 10 10 10 = ⋅ ⋅ xz xy yz yz xz xy 10 10 10 10 10 10 1º membro 1º membro 1º membro = 1
  • 24. Professor: Fabrício MaiaMatemática 28 66 Seja x tal que x 10 10log 2,log (2 1)− e x 10log (2 3)+ estão, nessa ordem, em progressão aritmética. Calcule 22x . A) 1 B) 4 C) 8 D) 16 E) 25 Solução: Temos que: x x 10 10 10 x x 10 10 10 x 2 x 10 10 x 2 x (log 2,log (2 1),log (2 3) P.A 2 log (2 1) log 2 log (2 3) log (2 1) log 2(2 3) (2 1) 2 (2 3) − + → → ⋅ − = + + → → − = + → → − = ⋅ + Tome: 2x = a Então: a2 – 2a + 1 = 2a + 6 → a2 – 4a – 5 = 0 a = 5 → 2x = 5 ou a = – 1 (não serve) Portanto, 22x = 25 Resposta: E 67 As dimensões de um paralelepípedo retângulo são proporcionais a 3,5 e 7. Sabendo que a diagonal mede 4 83cm, calcule o volume do paralelepípedo. A) 105cm3 B) 1575cm3 C) 4725cm3 D) 6720cm3 E) 8575cm3 Solução: Diagonal (D) → 2 2 2 2 D (3k) (5k) (7k) 4 83 83k 4 83 k 83 k 4 = + + = = = Volume (V) → 3 V 12 20 28 V 6720cm = ⋅ ⋅ = Resposta: D 68 Um prisma reto de altura igual a 9cm tem como base um triângulo. Sabendo que dois dos lados desse triângulo medem 3cm e 4cm e que o ângulo formado por esses lados mede 45º, determine o volume do prisma. 3 A) 3 2cm 3 B) 9 2cm 3 C) 27 2cm 3 D) 54 2cm 3 E) 81 2cm Solução: Volume do prisma: (Área da base) x (altura) Então: 3 3 4 sen45º V 9 2 2 V 6 9 27 2cm 2 ⋅ ⋅⎛ ⎞ = ⋅⎜ ⎟ ⎝ ⎠ = ⋅ ⋅ = Resposta: C 3k 5k 7k D 9 4 3 45º
  • 25. Professor: Fabrício Maia Matemática 29 69 A aresta, a diagonal e o volume de um cubo estão, nessa ordem, em progressão geométrica. Determine a área total desse cubo. A) 3 B) 6 C) 9 D) 18 E) 27 Solução: aresta = a diagonal = a 3 volume = a3 P.G. (a, a 3 , a3 ) → ( a 3 )2 = a ⋅ a3 → 3a2 = a4 → a2 = 3 Portanto, a área total será 18u.a. Resposta: D 70 Uma esfera de raio r é inscrita num cone equilátero com geratriz de comprimento g. Determine o valor de g 3 r . A) 3 B) 6 C) 8 D) 9 E) 12 Solução: 0 = incentro, baricentro, circuncentro, ortocentro. Veja: I) g = 2R (geratriz) II) g 3 g 3 3r 6 2 r = → = Resposta: B 71 O raio da base de um cone circular reto mede 4cm e sua altura 25 cm π . Determine, em cm3 , o volume do cilindro circular reto de maior área lateral, inscrito no cone. A) 4 B) 10 C) 25 D) 40 E) 50 Solução: D a a a 14243 2R 0 2R 2R 2r r 4 h α r 25 π
  • 26. Professor: Fabrício MaiaMatemática 30 Área (lateral do cilindro) = L2 rh Aπ = Veja: 25 h 25 tg h (4 r) 4 r 4 4 πα = = → = − − π Subst. h na área lateral, vem: 2 L L 25 25 A 2 r (4 r) A (4r r ) 4 2 ⎡ ⎤ = π − → = −⎢ ⎥π⎣ ⎦ Para que AL seja máxima, basta que r seja igual a abscissa do vértice da parábola. Então: r = 2 → 2 325 25 h V 50cm 2 2 2= → = π ⋅ = π π ⋅ Resposta: E 72 Determine a área (em m2 ) do setor circular hachurado na figura abaixo, sabendo que o ângulo ˆABCmede rad 6 π e o diâmetro AB mede 6 8 π m. A) 24 D) 54 B) 48 E) 54 3 C) 48 3 Solução: [setor] = 2 R 6 π ABCΔ é retângulo R 3 cos30º 26 8 = = π Então: = ⋅ ⋅ → = ⋅ ⋅ → π π π → = ⋅ ⋅ → = ⋅ → = π 2 2 2 2 6 6 2R 3 8 4R 3 64 6 R R 3 16 3 16 [setor] 48m . 6 Resposta: B 73 Dado um cilindro de revolução de raio r e altura h. sabe-se que a média harmônica entre o raio r e a altura h é 4 e que sua área total é 2πm2 . Mostre que o raio r satisfaz a sentença r3 – r + 2 = 0. Solução: B C A B C A 30º R R h r 123 parábola
  • 27. Professor: Fabrício Maia Matemática 31 Área total 2 2 2 rh 2 r 2 rh r 1 (I) π + π = π + = Média harmônica 2rh 4 r h 2r h (II) r 2 = + = − Subst. (II) em (I), vem: 2 2 2 2 2 2 3 2 3 3 2r 2r r r 1 r 1 r 2 r 2 2r r (r 2) r 2 2r r 2r r 2 r r 2 r r 2 0 ⎛ ⎞ + = → + = →⎜ ⎟− −⎝ ⎠ → + − = − → + − = − → → = − → − + = 74 Seja o determinante D(x) = 1 2senx senx . cosx 1 2senx − − + Calcule o valor de D π⎛ ⎞ ⎜ ⎟12⎝ ⎠ . 1 A) 2 2 B) 2 3 C) 2 1 D) 3 2 + 3 1 E) 2 4 + Solução: 2 D(x) 1 2sen x senxcos x sen(2x) D(x) cos(2x) 2 = − + = + Resposta: E 75 Seja R a raiz positiva da equação x2 + x – 3 4 = 0. Se R = senAº cosAº , sen11º cos11º onde 0 < A < 90. Calcule o valor de A. A) 30 B) 41 C) 60 D) 75 E) 80 Solução: Temos: 2 3 x x 0 4 1 1 x R 2 2 3 x (não serve) 2 + − = = → = − = Assim, R senAºcos11º sen11ºcos Aº R sen(Aº 11º) 1 sen(Aº 11º) 2 = − = − = − Então: Aº – 11º = 30º Aº = 41º → A = 41 Resposta: B sen 6 D cos 6 2 π⎛ ⎞ ⎜ ⎟π π⎛ ⎞ ⎛ ⎞ ⎝ ⎠= +⎜ ⎟ ⎜ ⎟12⎝ ⎠ ⎝ ⎠ 3 1 D 2 4 π⎛ ⎞ = +⎜ ⎟12⎝ ⎠ ou
  • 28. Professor: Fabrício MaiaMatemática 32 76 Determine a soma das raízes da equação. A) 0 D) 4 B) 1 E) 5 C) 2 Solução: Aplicando chió, vem: Daí: x 1 0 0 0 x 1 0 0 0 0 x 5 − + = − (x 1) (x 1)(x 5) 0− ⋅ + − = x = 1, – 1 ou 5 Portanto, a soma das raízes é 5. Resposta: E 77 Se o sistema {x my 3 mx 4y 6 + = + = tem infinitas soluções. Determine o valor de m4 – 8m2 + 23. A) 6 B) 7 C) 8 D) 9 E) 12 Solução: Sejam: r: a1x + b1y + c1 = 0 s: a2x + b2y + c2 = 0 Se r e s são coincidentes, então: 11 1 2 2 2 ba c a b c = = Assim, temos: 1 m 3 m 2 m 4 6 = = → = * retas coincidentes → infinitas soluções. Portanto, m4 – 8m2 + 23 = 7 Resposta: B 78 Se (xo, yo, zo) é uma solução do sistema 2 x y 2 xy z 1 + =⎧ ⎨ + =⎩ encontre o valor de 2 2 2 o o ox y 2z .+ − A) 0 B) 1 C) 2 D) 3 E) 4 Solução: 2 2 o o o o o o 2 2 o o o o o o x y 2 x y 2x y 4 x y z 1 2x y 2z 2 ⎧ + = → + + = ⎨ + = →− − = −⎩ Somando: o o 2 2 2 ox y 2z 2+ − = Resposta: C 1 1 1 1 1 x 1 1 0 1 1 x 2 1 1 1 1 x 4 = + − 1 1 1 1 1 x 1 1 0 1 1 x 2 1 1 1 1 x – 4 = +
  • 29. Professor: Fabrício Maia Matemática 33 79 Considere a função real definida no conjunto dos números reais não-negativos por f(x) = x + x – 2. Determine o número real k, tal que f(2k ) = 0. A) 0 B) 1 C) 2 D) 3 E) 4 Solução: Temos que: + − = − = − − ⋅ + = − ⋅ + = = → = = → = k k k k 2k k k 2k k k k 2 2 2 0 2 2 2 2 4 2 4 2 2 5 2 4 0 2 1 k 0 ou 2 4 k 2 (não serve) Veja: se k = 2 → f(2k ) = f(4) = 4 ≠ 0 Resposta: A 80 Sendo a reta y = ax + b tangente à elipse x2 + 4y2 = 1, determine o valor de 8(b2 – a2 ). A) 0 B) 1 C) 2 D) 3 E) 4 Solução: Substituindo a reta na equação da elipse, vem: x2 + 4y2 = 1 x2 + 4(a2 x2 + 2abx + b2 ) = 1 (1 + 4a2 ) x2 + 8abx + 4b2 – 1 = 0 Como a reta é tangente, então a interseção é um único ponto. Δ = 0 (único ponto) Daí: (8ab)2 – 4(1 + 4a2 ) ⋅ (4b2 – 1) = 0 64a2 b2 – 16b2 + 4 – 64a2 b2 + 16a2 = 0 8a2 – 8b2 + 2 = 0 8(b2 – a2 ) = 2 Resposta: C 81 Determine o valor de b para o qual a reta y = x + b não intercepta os ramos da hipérbole x2 – y2 = 1. A) 0 B) 1 C) 2 D) 3 E) 4 Solução: Interseção → x2 – ( x + b)2 = 1 → x2 – x2 – 2bx – b2 = 1 → – 2bx =1+ b2 2 1 b x 2b + → = − Veja: para que não exista interseção, basta tomarmos b = 0. Resposta: A 82 Determine o menor inteiro n > o, de modo que n 3 1 i 2 2 ⎛ ⎞ +⎜ ⎟⎜ ⎟ ⎝ ⎠ seja real positivo. A) 6 B) 10 C) 12 D) 16 E) 24 (x da interseção)
  • 30. Professor: Fabrício MaiaMatemática 34 Solução: Temos que: n n3 1 i (cos30º isen30º) cos(n 30º) isen(n 2 2 30º) ⎛ ⎞ + = + = ⋅ +⎜ ⎟⎜ ⎟ ⎝ ⎠ ⋅ Então: n 30º k 360º n 12k ⋅ = ⋅ = Portanto: n = 12 (menor inteiro positivo) Resposta: C 83 Encontre o módulo do complexo 1 , tal que 2 i.=1 A) 1 B) 2 C) 3 D) 2 E) 3 Solução: Temos: 2 2 2 2 i | | |i | | | | 0 1i| | | | | 0 1 | | | | 1 | | 1 = → = → → ⋅ = + → ⋅ = + → → ⋅ = → = 1 1 1 1 1 1 1 1 1 Resposta: A 84 Se A , B e C são números reais, tais que 2 2 1 A Bx C , x(x 2x 2) x x 2x 2 + = + + + + + para todo x, x *∈ 1 , calcule o valor de A + B + C. Solução: 2 2 2 2 2 2 A Bx C 1 , x * x x 2x 2 x(x 2x 2) A(x 2x 2) (Bx C)x 1 , x * x(x 2x 2) x(x 2x 2) 1 A 2A B 0 1 (A B)x (2A C)x 2A 1 2A C 0 B 22A 1 C 1 + + = ∀ ∈ + + + + + + + + = ∀ ∈ + + + + ⎧ =⎪ + =⎧ ⎪⎪ ⎪ + + + + ≡ → + = → = −⎨ ⎨ ⎪ ⎪=⎩ = −⎪ ⎪⎩ 1 1 Portanto: A + B + C = – 1 85 Determine um polinômio P(x) de grau 2 que verifique a identidade P(x + 1) ≡ x2 + 2x + 3. Solução: Supondo P(x) = ax2 = bx + c, temos: P(x + 1) = a(x + 1)2 + b(x + 1) + c = ax2 + (2a + b)x + (a + b + c). Então: P(x + 1) ≡ x2 + 2x + 3 a 1 a 1 2a b 2 b 0 a b c 3 c 2 = =⎧ ⎧ ⎪ ⎪ ⇔ + = ⇔ =⎨ ⎨ ⎪ ⎪+ + = =⎩ ⎩ Logo, P(x) = x2 + 2. 14243 um 14243 zero
  • 31. Professor: Fabrício Maia Matemática 35 86 Que condições devem satisfazer os números a, b e c para que o polinômio ax2 + bx + c seja o quadrado de um polinômio do 1º grau? Solução: Devemos ter ax2 + bx + c ≡ (mx + n)2 , com m ≠ 0; portanto: 2 2 a m b 2mn c n ⎧ = ⎪ =⎨ ⎪ =⎩ Podemos eliminar m e n e obter a relação entre a, b e c e calculando b2 . b2 = (2mn)2 = 4m2 n2 = 4ac Resposta: A condição é b2 = 4ac e a ≠ 0 (pois m ≠ 0) 87 Na figura abaixo indicamos 9 pontos, entre os quais não há 3 colineares, exceto os 4 que marcamos numa mesma reta. Quantos triângulos existem com vértices nestes pontos? Solução: Se não houvessem 3 pontos colineares, o número de triângulos seria C9, 3. Desse número, devemos subtrair as combinações formadas por 3 pontos escolhidos entre os 4 alinhados, isto é, C4, 3, pois essas combinações não correspondem a triângulos. Assim, o número de triângulos que podemos formar é C9, 3 – C4, 3. Temos: 9,3 9! 9x8x7x 6! C 3!6! = = 3x2x1x 6! 4,3 84 4! 4 x 3! C 3!1! = = = 3 4 !x 1 = Logo: C9,3 – C4, 3 = 84 – 4 = 80. 88 Um químico possui 10 tipos de substâncias. De quantos modos possíveis poderá associar 6 dessas substâncias se, entre as 10, duas somente não podem ser juntadas porque produzem mistura explosiva? Solução: Cada mistura de 6 das 10 substâncias corresponde a uma combinação das 10 substâncias tomadas 6 a 6, uma vez que não importa a ordem das substâncias na mistura. Assim, o total de misturas seria C10, 6 se não houvesse problema com nenhuma mistura. Devemos, porém, subtrair desse número as combinações em que entrariam as duas substâncias que, se misturadas, provocam explosão. As combinações em que entram essas duas substâncias são formadas por elas duas e mais quatro substâncias escolhidas entre as outras oito substâncias (excluímos aquelas duas). O número de modos de escolher 4 substâncias em 8 é C8, 4. Concluímos que o número de misturas não explosivas que podem ser produzidas é C10, 6 – C8, 4. Solução: Temos: 10,6 10! 10x9x8x7x 6! C 6!4! = = 6! 8,4 210 x 4x3x2x1 8! 8x7x6x5x 4! C 4!4! = = = 4x3x2x1x 4! 70= Logo: C10, 6 – C8, 4 = 210 – 70 = 140. D H A I G F E CB
  • 32. Professor: Fabrício MaiaMatemática 36 89 Dê a condição sobre o inteiro positivo n para que o desenvolvimento de n 2 1 x x ⎛ ⎞ −⎜ ⎟ ⎝ ⎠ apresente um termo independente de x e não-nulo. Solução: O termo geral do desenvolvimento de n 2 1 x x ⎛ ⎞ −⎜ ⎟ ⎝ ⎠ é k 2 n k 1n T (x ) k x − ⎛ ⎞⎛ ⎞ = − =⎜ ⎟ ⎜ ⎟ ⎝ ⎠ ⎝ ⎠ 2n 2k k kn x ( 1) x k − −⎛ ⎞ − =⎜ ⎟ ⎝ ⎠ k 2n 3kn ( 1) x k −⎛ ⎞ −⎜ ⎟ ⎝ ⎠ Para o termo independente de x devemos ter 2n – 3k = 0, logo k = 2n 3 . Como k dever ser inteiro, concluímos que n deve ser um múltiplo de 3. 90 Calcule a e b de modo que a fração algébrica 2 2 x ax b 2x 1 + + + tenha o mesmo valor numérico para todo x ∈ 1 . Solução: Devemos ter: 2 2 x ax b 2x 1 + + + = k, x∀ ∈1 ; logo: x2 + ax + b ≡ 2kx2 + k 1 2k a 0 b k =⎧ ⎪ =⎨ ⎪ =⎩ A resposta é a = 0 e b = 1 2 . 91 Calcule o valor numérico de x4 + 4x3 y + 6x2 y2 + 4xy3 + y4 , para 4 4 3 1 3 1 x e y 3 3 + − = = . Solução: x4 + 4x3 y + 6x2 y2 + 4xy3 + y4 = (x + y)4 = 4 4 4 2 4 4 4 3 1 3 1 2 3 2 3 48. 33 3 3 ⎛ ⎞ ⎛ ⎞+ − ⋅ + = = =⎜ ⎟ ⎜ ⎟⎜ ⎟ ⎜ ⎟ ⎝ ⎠ ⎝ ⎠ 92 O número 2 é raiz dupla de ax3 + bx + 16. Determine a e b. Solução: Como admite raiz dupla, o grau da equação ax3 + bx + 16 = 0 é maior que 1. Então, a ≠ 0 e concluímos que o grau é 3. Há, portanto, 3 raízes. Supondo que as raízes são 2, 2 e α, com α ≠ 2, temos pelas relações de Girard: 2 2 0 4 4 b a 2 2 2 2 , logo 4 4 , logo a 1 a b b 12 16 4 2 2 a a ⎧ ⎧ ⎪ ⎪+ + α = α = − α = −⎧⎪ ⎪⎪ ⎪ ⎪ ⋅ + α + α = α + = =⎨ ⎨ ⎨ ⎪ ⎪ ⎪ = −⎩− −⎪ ⎪⋅ ⋅ ⎪ ⎪⎩ ⎩ α = α = Portanto: a = 1 e b = – 12 93 Qual é o valor de 12 k k 0 12 9 ? k= ⎛ ⎞ ⎜ ⎟ ⎝ ⎠ ∑ Solução: 12 12 k 12 k k k 0 k 0 12 12 9 1 9 k k − = = ⎛ ⎞ ⎛ ⎞ = ⋅ ⋅⎜ ⎟ ⎜ ⎟ ⎝ ⎠ ⎝ ⎠ ∑ ∑ Notando que 12 k k12 1 9 k −⎛ ⎞ ⋅ ⋅⎜ ⎟ ⎝ ⎠ é o termo geral do binômio (1 + 9)12 , concluímos que: 12 k 12 12 k 0 12 9 (1 9) 10 k= ⎛ ⎞ = + =⎜ ⎟ ⎝ ⎠ ∑ ( o que dá 1 trilhão). Este fator é igual a 1, portanto não altera o valor do termo.
  • 33. Professor: Fabrício Maia Matemática 37 94 Numa urna há 12 etiquetas numeradas, 6 com números positivos e 6 com números negativos. De quantos modos podemos escolher 4 etiquetas diferentes tal que o produto dos números nelas marcados seja positivo? Solução: Teremos o produto positivo em cada caso seguinte: I) Escolhendo 4 etiquetas com números positivos; ou II) Escolhendo 4 etiquetas com números negativos; ou III) Escolhendo 2 etiquetas com números positivos e 2 com números negativos. Números disponíveis: 6 positivos 6 negativos Possibilidades: 4 positivos 0 negativos ou 0 positivos 4 negativos ou 2 positivos 2 negativos Vamos calcular o número de possibilidades de cada caso (lembrando que não importa a ordem das etiquetas). I) O número de modos a escolher 4 números positivos, dispondo de 6 números positivos, é 6,4C . 6,4 6! 6 x 5 x 4! C 4!2! = = 4! 15 x 2 x 1 = II) Como temos também 6 números negativos, o número de modos de escolher 4 deles é 6,4C = 15. III) Dos 6 positivos devemos escolher 2( 6,2C ) e, para cada escolha destes, dos 6 negativos devemos escolher também 2 (C6, 2). O número de possibilidades deste caso é 6,2 6,2C C⋅ . Como 6,2 6! C 15, 2!4! = = temos 15 ⋅ 15 = 225 possibilidades. Então, o total de possibilidades para o produto positivo é 15 + 15 + 225 = 255. 95 Encontre o coeficiente de x5 no desenvolvimento de (1 – x) (1 + x)8 . Solução: Quando multiplicamos (1 – x) pelo polinômio obtido desenvolvendo (1 + x)8 , o termo em x5 resulta da adição de dois produtos: (1 – x) (1 + ... + termo em x4 + termo em x5 + ... + x8 ) Termo em x5 = 1⋅ [termo em x5 de (1 + x)8 ] + [(– x) ⋅ termo em x4 de (1 + x)8 ] O termo geral de (1 + x)8 é T = 8 k k k8 8 1 x x . k k −⎛ ⎞ ⎛ ⎞ ⋅ ⋅ =⎜ ⎟ ⎜ ⎟ ⎝ ⎠ ⎝ ⎠ Para k = 5 temos T = 5 5 5 58! 8 x 7 x 68 x x x 56x . 5 5! 3! 3 x 2 x 1 ⎛ ⎞ = = =⎜ ⎟ ⎝ ⎠ Para k = 4 temos T = 4 4 4 48! 8 x 7 x 6 x 58 x x x 70x . 4 4! 4! 4 x 3 x 2 x 1 ⎛ ⎞ = = =⎜ ⎟ ⎝ ⎠ Então, no produto (1 – x) (1 + x) 8 temos: Termo em x5 = [1 x 56x5 ] + [(– x) ⋅ 70x4 ] = 56x5 – 70x5 = – 14x5 O coeficiente pedido é igual a – 14. ↓ ↓ 6,4C 6,4C ⋅6,2 6,2C C
  • 34. Professor: Fabrício MaiaMatemática 38 96 Se A é uma matriz quadrada de ordem três com detA = 5, então o valor de det(2A) é: A) 6 B) 11 C) 15 D) 30 E) 40 Solução: Sabemos que: det(k.A) = kn ⋅ det(A), onde: n → é a ordem da matriz A Então: det(2A) = 23 ⋅ det(A) = 8 ⋅ 5 = 40. Resposta: E 97 Se a matriz A satisfaz A2 – 2A + I = 0, então A– 1 : A) não existe. B) é igual a I. C) é igual a A. D) é igual a A – 2I. E) é igual a 2I – A. Solução: Sabemos que: A ⋅ A– 1 = A –1 ⋅ A = I Então: A2 – 2A + I = 0 → I = 2A – A2 → → I = 2AI – A2 → I = 2IA – AA → → I = (2I – A) ⋅ A → A–1 = 2I – A Resposta: E 98 Uma loja, realizando uma promoção, oferece um desconto de 20% nos preços dos seus produtos. Para voltar aos preços iniciais, os preços promocionais devem sofrer um acréscimo de A%. Determine o valor de A. A) 10 B) 20 C) 25 D) 30 E) 40 Solução: Preço inicial = P → com desconto = 80 P 100 Para voltar ao preço inicial, temos: Resposta: C 14444244443 novo preço + ⋅ = ⋅ = = = 80 A 80 P P P 100 100 100 A 80 20 P P 100 100 100 A 1 100 4 A 25
  • 35. Professor: Fabrício Maia Matemática 39 99 Sejam p e q números inteiros positivos e consecutivos. Se 1 1 11 , p q 30 + = então p + q é igual a: A) 9 B) 11 C) 13 D) 15 E) 17 Solução: 1 1 11 q p 11 p q 30 pq 30 + + = ⇒ = Como p e q são inteiros positivos consecutivos, então p e q são primos entre si, isto é, m.d.c. (p, q) = 1. Assim, p = 5 e q = 6 ou p = 6 e q = 5. Portanto, p + q = 11 Resposta: B 100 O gráfico da função f(x) = sen x no intervalo [ 2 π ,π[ e: A) crescente B) decrescente C) constante D) nula E) negativa Solução: Esboço do gráfico de sen x no de [0,2π] Resposta: B 101 Um dos ângulos de um triângulo retângulo é α. Se tg α = 2,4 , os lados desse triângulo são proporcionais a: A) 30,40,50 B) 80,150,170 C) 120, 350, 370 D) 50, 120, 130 E) 61, 60, 11 Solução: Se tgα =2,4 → tgα = 24 10 → tgα = 12 5 cateto oposto = 12k e cateto adjacente = 5k → hipotenusa = 13k. Tomando k =10 → lados: 50,120 e 130 Resposta: D 102 A distância do ponto de interseção das retas 2x – 3y + 26 = 0 e 5x + 2y – 49 = 0 à origem é: A) 13 B) 23 C) 15 D) 18 E) 17 Solução: Resolvendo o sistema formado pelas equações acima, encontramos como interseção o ponto (5, 12). Assim, a distância do ponto (5, 12) ao ponto (0, 0) é igual a d= 2 2 (5-0) (12 0) 169 13.+ − = = Resposta: A 103 As promoções do tipo “leve 3 pague 2”, comuns no comércio, acenam com um desconto, sobre cada unidade vendida, de: A) 50 % 3 B) 20% C) 25% D) 30% E) 100 % 3 Solução: Observe: I) Quem leva 3 e paga 2 está comprando 3 e tendo um desconto de 1; II) Se 1 é p% de 3 → 1 = P 100 3 P 100 3 ⋅ → = ou P = 33,33% (aproximadamente) Resposta: E 0 y x decrescente
  • 36. Professor: Fabrício MaiaMatemática 40 104 Sabendo que cos 36º = 1 5 2 + , então cos 72º vale: A) 1 5 2 + B) 5 1 4 − C) 5 1 2 − D) 1 5 2 − E) 1 5 4 − Solução: Sabemos que: cos2x = 2cos2 x – 1 (arco duplo) Tomando x = 36º, encontramos: cos72º = 2cos2 36º – 1 → cos72º = 2 2 1 5 5 1 1 cos72º 4 4 ⎛ ⎞+ − ⋅ − → =⎜ ⎟⎜ ⎟ ⎝ ⎠ Resposta: B 105 Se y = cos2280º, então y é igual a: A) – cos12º B) – cos60º C) – cos30º D) cos12º E) cos60º Solução: Sabemos que: se a + b = 180º, então sen a = sen b e cos a = – cos b Dividindo 2280º por 360º, encontramos: 2280º = 120º + 6.360º Assim: cos 2280º = cos 120º = – cos 60º Resposta: B 106 A área máxima da região limitada por um triângulo retângulo inscrito em um círculo de raio R é: A) 2R2 B) πr2 C) R2 D) 2 R 2 E) 2πr2 Solução: I) A área de um triângulo é igual a base x altura 2 . II) Tome AB como base → base = 2R (diâmetro), pois O é o centro. III) De todas as alturas relativas à hipotenusa AB, a maior é EO = R, onde R é o raio. Logo, o triângulo de área máxima tem área igual a 22R R R . 2 ⋅ = Veja figura: Os ângulos ˆ ˆ ˆ ˆACB,ADB,AEB e AFB são retos. Resposta: C 107 Se p é natural maior que 1, não divisível por 2 e nem por 3, então p2 – 1 é divisível por: A) 18 B) 24 C) 36 D) 9 E) 27 Solução: É fácil ver que: I) Se p não é múltiplo de 2 → p – 1 e p + 1 são pares consecutivos, logo (p – 1) ⋅ (p + 1) é múltiplo de 8. II) Se p não é múltiplo de 3 → p – 1 ou p + 1 será um múltiplo de 3, logo (p – 1) ⋅ (p + 1) é múltiplo de 3. De (I) e (II), concluímos: O produto (p – 1) ⋅ (p + 1) = p2 – 1 é um múltiplo de 24. Resposta: B A B OG H E D C I F
  • 37. Professor: Fabrício Maia Matemática 41 108 Sejam a e b números reais positivos tais que a + 3b = 30. Determine qual é o maior valor possível para ab. Solução: Sabemos que: Se a1, a2, a3, ..., an são números reais positivos, então: Média aritmética = 1 2 3 na a a ... a n + + + + e Média geométrica = n 1 2 3 na a a ...a⋅ ⋅ ⋅ Relação importante entre as duas médias: M A M G⋅ ≥ ⋅ Daí, a 3b 30 a 3b 3ab 15 3ab 225 3ab ab 75 2 2 + ≥ ⋅ → ≥ → ≥ → ≥ → ≤ Portanto, o maior valor para ab é 75. Resposta: 75 109 Seja x um número natural, que ao ser dividido por 9 deixa resto 5, e ao ser dividido por 3 deixa resto 2. Sabendo que a soma dos quocientes é 9, podemos afirmar que x e igual a: A) 28 B) 35 C) 27 D) 33 E) 23 Solução: Temos que: x = 9a + 5, onde a é o quociente da divisão de x por 9. x = 3b + 2, onde b é o quociente da divisao de x por 3. Como a soma dos quocientes é 9, vem: x 5 x 2 9 x 23 9 3 − − + = → = Resposta: E 110 Se 10tgx + 16cosx – 17secx = 0, então senx é igual a: A) 1 1 ou 2 8 B) 1 2 ± C) 1 1 ou 4 12 D) 0 E) 1 1 ou 2 4 − Solução: Temos: 10tgx + 16cosx – 17secx = 0 senx 1 10 16cosx –17 0 cosx cosx → ⋅ + ⋅ = → 10senx + 16cos2 x – 17 = 0 → 10senx + 16⋅ (1 – sen2 x) – 17 = 0 → 16sen2 x – 10senx + 1 = 0 1 1 senx ou senx 2 8 → = = Resposta: A 111 Um atleta, correndo com velocidade constante, completou a maratona em M horas. A fração do percurso que ele correu em 2M minutos, foi: A) 1 2 B) 1 6 C) 1 15 D) 1 30 E) 1 20 Solução: Temos que: M horas = M⋅ 60 minutos Veja: Se M⋅ 60 minutos = P (percurso completo) → 2⋅ M minutos = P 30 (fração do percurso) Resposta: D
  • 38. Professor: Fabrício MaiaMatemática 42 112 Sendo R = 02 – 12 + 22 – 32 + 42 – 52 + ... + 982 – 992 + 1002 , calcule o valor de R . 202 A) 22 B) 23 C) 24 D) 25 E) 26 Solução: Sabemos que: (I) a2 – b2 = (a – b)⋅ (a + b) e (II) Sn = 1 n(a a ) n 2 + ⋅ (soma dos termos de uma P.A.) Veja: R = (22 – 12 ) + (42 – 32 ) + (62 – 52 )+ ... + (1002 – 992 ) R = (2 – 1) ⋅ (2 + 1) + (4 – 3)⋅ (4 + 3) + (6 – 5)⋅ (6 + 5) + ... + (100 – 99)⋅ (100 + 99) R = 3 + 7 + 11 + ... + 199 R = (3 199) 50 R 202.25 25. 2 202 + ⋅ = → = Resposta: D 113 O primeiro termo a de uma progressão aritmética de razão 13 satisfaz 0 ≤ a ≤ 10. Se um dos termos da progressão é 35, determine o valor de a. A) 5 B) 6 C) 7 D) 8 E) 9 Solução: Temos: P.A. (a, a + 13, a + 26, a + 39, ... , 35, ...) Usando a fórmula do termo geral, encontramos: an = a1 + (n – 1)⋅ r → 35 = a + (n – 1)⋅ 13 → 35 = a + 13n – 13 → a = 48 – 13n → 0 ≤ 48 –13n ≤ 10 → – 48 ≤ –13n ≤ –38 → 48 ≥13n ≥ 38 → 2, ... ≤ n ≤ 3, ... → n = 3 → a = 48 – 13⋅ 3 → a = 9. Resposta: E 114 O algarismo das unidades do número N = 1.3.5.7. ... . 1993 é: A) 1 B) 3 C) 5 D) 7 E) 9 Solução: Note que o produto de qualquer número ímpar por 5 sempre termina em 5; logo, com o número N só tem fatores ímpares, seu algarismo das unidades é 5. Resposta: C 115 Se x y 7 3 = e x⋅ y = 189, então: x – y vale com x e y positivos: A) 12 B) 4 C) 9 D) 30 E) 21 Solução: Temos: x y k 7 3 = = → x = 7k e y = 3k Daí: 7k⋅ 3k = 189 → 21k2 = 189 → k2 = 9 → k = 3 Para: k = 3 → x = 21 e y = 9 Portanto: x – y = 12 Resposta: A 116 A planta de um apartamento está confeccionada na escala 1:50. Então a área real, em m2 , de uma sala retangular, cujas medidas na planta são 12cm e 14cm, é: A) 24 B) 26 C) 28 D) 42 E) 54 Solução: Sabemos que escala é a razão entre o comprimento no desenho e o comprimento real, medidos na mesma unidade.
  • 39. Professor: Fabrício Maia Matemática 43 Daí: 1 12 50 x = → x = 600cm → x = 6m 1 14 50 x = → y = 700cm → y = 7m Logo, a área da sala será de 6m x 7m = 42m2 . Resposta: D 117 Prove que todo triângulo ABC vale a relação: ˆ ˆc a cosB b cosA.= ⋅ + ⋅ Solução: mˆ ˆcosA m bcosA b nˆ ˆcosB n acosB a ˆ ˆ ˆ ˆlogo, m n bcosA acosB c acosB bcosA. (OK) = → = = → = + = + → = + Obs: Sendo ˆA ou ˆB obtuso, chegamos com raciocínio análogo ao mesmo resultado. 118 Considerem-se todas as divisões de números inteiros positivos por 17, cujo resto é igual ao quadrado do quociente. A soma dos quocientes dessas divisões é: A) 10 C) 172 E) 12 + 22 + 32 + ... + 172 B) 17 D) 1 + 2 + 3 + ... + 17 Solução: Temos que: n dividido por 17 tem quociente q ≠ 0 e resto r com r = q2 . Veja: (I) Possíveis restos de uma divisão por 17 são: 0, 1, 2, 3 ,... , 16 (II) Como r tem que ser um quadrado perfeito, devemos ter: r = 0 → q = 0 (não sastisfaz) r = 1 → q = 1 r = 4 → q = 2 r = 9 → q = 3 r = 16 → q = 4 Logo, a soma dos quocientes é 10. Resposta: A 119 Determine o valor do produto P = cos36º⋅ cos72º. Solução: Sabemos que sen2x = 2senxcosx → 2sen36º⋅ P = 2sen36º⋅ cos36ºcos72º → 2sen36º⋅ P = sen72ºcos72º→ → 4sen36º⋅ P = 2sen72ºcos72º → 4sen36º⋅ P = sen144º → 4 ⋅ P = 1, pois sen36º = sen144º (suplementares) → P = 1 4 . Resposta: 1 4 A B C ab m nH c
  • 40. Professor: Fabrício MaiaMatemática 44 120 Sejam f(x) = 1 x 1− , x > 1 e g uma função tal que (gof)(x) = x. Determine o valor de g 1 64 ⎛ ⎞ ⎜ ⎟ ⎝ ⎠ . Solução: g(f(x)) = x → g 1 x 1 ⎛ ⎞ ⎜ ⎟−⎝ ⎠ = x Veja: se 1 1 x 1 64 = − → x – 1 = 64 → x = 65 → 1 g 64 ⎛ ⎞ ⎜ ⎟ ⎝ ⎠ = 65 Resposta: 65 121 O triângulo ABC está inscrito em um círculo de raio R. Se cosA = 3 5 , o comprimento do lado BC é igual a: A) 2R 5 B) 3R 5 C) 4R 5 D) 6R 5 E) 8R 5 Solução: Temos que: 3ˆcosA 5 = ( ˆA é agudo) → sen ˆA = 4 5 Lei dos senos a b c BC 2R 2R ˆ ˆ ˆ ˆsenA senB senAsenC = = = → = → BC = 2R⋅ ˆsenA → BC = 2R⋅ 4 5 → BC = 8R 5 Resposta: E 122 Seja f(x) = x x x x e e e e − − − + definida em R. Se g for a função inversa de f, o valor de 7 g 25 e ⎛ ⎞ ⎜ ⎟ ⎝ ⎠ será: A) – 1 B) 0 C) 1 e D) 1 E) 4 3 Solução: Como g é a inversa de f, temos: x x 1 x x 7 7 7 e e g( ) f ( ) 25 25 25 e e − − − − = → = → + 7⋅ ex + 7⋅ e–x = 25⋅ ex – 25⋅ e–x → – 18⋅ ex = – 32⋅ e– x → 7 g( ) 2x x 25 16 4 4 e e e . 9 3 3 → = → = → = Resposta: E 123 A média aritmética dos ângulos internos de um eneágono convexo vale: A) 40º B) 70º C) 120º D) 135º E) 140º Solução: Sabemos que a soma dos ângulos internos de um polígono convexo é dada pela fórmula S = (n – 2)⋅ 180º. Daí, a soma dos ângulos internos de um eneágono convexo é igual a S = (9 – 2)⋅ 180º = 7.180º. Portanto, a média aritmética será igual a 7.180º 7 20º 140º. 9 = ⋅ = Resposta: E
  • 41. Professor: Fabrício Maia Matemática 45 124 Uma solução tem 75% de ácido puro. Quantos gramas de ácido puro devemos adicionar a 48 gramas da solução para que a nova solução contenha 76% de ácido puro? Solução: Veja: Em 48 gramas de solução temos 75 . 48 36 100 = gramas de ácido puro. Adicionando x gramas de ácido puro a solução, teremos: Nova solução = (48 + x) gramas Quantidade de ácido puro = (36 + x) gramas Assim: 36 x 76 48 x 100 + = → + x = 2 gramas Resposta: 2 gramas 125 O gráfico de uma função f é o segmento de reta que une os pontos (– 3, 4) e (3, 0). Se f–1 é a função inversa de f, então o valor de f– 1 (2) é igual a: A) 1 B) 2 3 C) 3 D) 0 E) 3 2 Solução: Temos: f(x) = ax + b, com a ≠ 0. Pontos: (– 3, 4) e (3, 0) Para x = – 3 → a ⋅ (– 3) + b = 4 Para x = 3 → a⋅ 3 + b = 0 Resolvendo, encontramos: b = 2 e a = 2 3 − Daí, a função f é dada por: f(x) = 2 x 2 3 − + Veja: cálculo de f – 1 (2) Para y = 2 → 2 = 2 x 2 3 − + → x = 0 → f – 1 (2) = 0. Resposta: D 126 Um elevador pode levar 20 adultos ou 24 crianças. Se 15 adultos já estão no elevador, quantas crianças podem ainda entrar? A) 5 B) 6 C) 7 D) 8 E) 9 Solução: Se 20 adultos equivalem a 24 crianças → 5 adultos equivalem a 6 crianças Veja: (I) O elevador pode levar 20 adultos (II) Tem 15 adultos no elevador → faltam 5 adultos (equivalem a 6 crianças) . Resposta:B 127 Uma torneira enche um tanque em 4 horas. O ralo do tanque pode esvaziá-lo em 3 horas. Estando o tanque cheio, abrimos, simultaneamente, a torneira e o ralo. Então o tanque: A) nunca se esvazia. B) esvazia-se em 1 hora. C) esvazia-se em 4 horas. D) esvazia-se em 7 horas. E) esvazia-se em 12 horas. Solução: Capacidade do tanque: T (I) Torneira enche T em 4 horas → em 1 hora enche T 4 do tanque (II) Ralo esvazia o tanque T em 3 horas → em 1 hora esvazia T 3 do tanque
  • 42. Professor: Fabrício MaiaMatemática 46 Assim, o tanque em uma hora esvazia 1 12 de sua capacidade. Veja: T T 3T 4T T 4 3 12 12 − − = = − . Portanto, o tanque esvazia-se em 12 horas. Resposta: E 128 Determine o valor de – 63⋅ log8(sen15º ⋅ sen75º). Solução: Fatos que ajudam: (I) Se a + b = 90º, então sen a = cos b e sen b = cos a (II) Sen 2a = 2sen a cos a (III) n m a log b = a m log b n ⋅ Temos que sen15º⋅ sen75º = sen15º⋅ cos15º = 2 sen15º cos15º sen30º 1 2 2 4 ⋅ ⋅ = = Então, a expressão vale: Exp. = – 63⋅ log8 3 2 22 1 2 63 log 2 63 ( ) log 2 42 1 42 4 3 −⎛ ⎞ = − ⋅ = ⋅ − ⋅ = ⋅ =⎜ ⎟ ⎝ ⎠ Resposta: 42 129 Considere um quadrilátero convexo ABCD de área igual a 66cm2 . Determine, em cm2 , a área do quadrilátero cujos vértices são os pontos médios dos lados do quadrilátero ABCD. Solução: Fatos que ajudam: ÁREAS (I) Seja ABC um triângulo qualquer e seja MNP o triângulo que tem vértices nos pontos médios dos lados do triângulo ABC, temos que: Área(MNP) = Área(ABC) 4 . (II) Seja ABCD um quadrilátero qualquer e seja MNQP o quadrilátero que tem vértices nos pontos médios dos lados de ABCD, temos que: Área(MNQP) = Área(ABCD) 2 . Usando o resultado (II) no enunciado da questão, concluímos: Resposta: 33cm2 130 Se x é um número real tal que x2 + 2 1 3 x = , determine o valor de x4 + 4 1 . x Solução: Se 2 2 2 2 4 2 4 4 2 2 2 4 4 4 1 1 1 1 1 1 x 3 x 3 x 2 x 9 x 2 9 x 7 x x x x x x ⎛ ⎞ + = → + = → + ⋅ ⋅ + = → + + = → + =⎜ ⎟ ⎝ ⎠ . Resposta: 7 131 O menor número natural n, diferente de zero, que torna o produto de 3888 por n um cubo perfeito é: A) 6 B) 12 C) 15 D) 18 E) 24 Solução: Fatorando o número 3888, obtemos 3888 = 24 ⋅ 35 Para formar um cubo perfeito devemos multiplicar os dois membros no mínimo por 22 ⋅ 31 , para que as potências dos números 2 e 3 sejam múltiplos de 3. Assim, o menor número que devemos multiplicar por 3888 para obter um cubo perfeito é 12. Resposta: B
  • 43. Professor: Fabrício Maia Matemática 47 132 Quantos números inteiros há entre 602 e 612 que não são quadrados perfeitos? A) 118 B) 119 C) 120 D) 121 E) 122 Solução: Veja: 602 e 612 são quadrados perfeitos consecutivos, então, qualquer inteiro entre eles não é quadrado perfeito. Inteiros que não são quadrados perfeitos: 3601, 3602, 3603, ... ,3720 Quantidade de inteiros que não são quadrados é igual a 3720 – 3601 + 1 = 120. Resposta: C 133 O período da função f (x) = sen4 x + cos4 x vale: A) 2π B) π C) 3π 2 D) π 2 E) 4 π Solução: Sabemos: Se f é periódica → f(x + p) = f(x), para todo x no domínio da função. O menor valor positivo de p, chamamos de período de f. Tomando x = 0, encontramos: f (p) = f (0) → sen4 p + cos4 p = sen4 0 + cos4 0 → sen4 p + cos4 p = 1 → (sen2 p + cos2 p)2 – 2sen2 p⋅ cos2 p = 1 → 1 – 2sen2 pcos2 p =1 → 2sen2 p cos2 p = 0 → senp = 0 ou cosp = 0. * Se senp = 0 → p = 0, π, 2π,... * Se cosp = 0 → p = 3 , ,... 2 π π 2 Agora, devemos verificar se p = 2 π satisfaz a condição f(x + p) = f(x), para todo x no domínio da função. Veja: f(x + p) = f(x + 2 π ) = sen4 (x + 2 π ) + cos4 (x + 2 π ) = (cosx)4 + (–senx)4 = sen4 x + cos4 x = f (x) (OK) Resposta: D Obs: No ciclo trigonométrico, encontramos facilmente: sen(x + 2 π ) = cosx e cos(x + 2 π ) = – senx (verifique!) 134 O conjunto solução da equação x⋅ (log53x + log521) + log5( 3 7 )x = 0 é igual a: A) { } B) {0} C) {1} D) {0, 2} E) {0, – 2} Solução: Temos que: x⋅ (log53x + log521) + log5( 3 7 )x = 0 → x⋅ log5(3x ⋅ 21) + log5( 3 7 )x = 0 → log5(3x ⋅ 21)x + log5( 3 7 )x = 0 → log5[(3x ⋅ 21)x ⋅ ( 3 7 )x ] = 0 → (3x ⋅ 21)x ⋅ ( 3 7 )x = 1 → (3x ⋅ 21⋅ 3 7 )x = 1 → (3x ⋅ 3⋅ 3)x = 1 → → (3x + 2 )x = 1 → 2 x 2x 3 1+ = → x2 + 2x = 0 → x = – 2 ou x = 0. Resposta: E 135 Um número é composto de 2 algarismos, cuja soma é 9. Invertendo a ordem dos algarismos, obtemos um novo número igual a 4 7 do original. Qual é o número? Solução: * Número original: ab = a⋅ 10 + b (forma polinomial) * Invertendo os algarismos obtemos um novo número: ba = b⋅ 10 + a (forma polinomial)
  • 44. Professor: Fabrício MaiaMatemática 48 Equações do problema: a + b = 9 e b⋅ 10 + a = 4 7 ⋅ (a⋅ 10 + b) Segunda equação: 70b + 7a = 40a + 4b → 66b = 33a → a = 2b Substituindo na primeira equação, teremos: a + b = 9 → 2b + b = 9 → 3b = 9 → b = 3 → a = 6 Resposta: 63 136 A distância entre dois lados paralelos de um hexágono regular é igual a 2 3 cm. A medida do lado desse hexágono, em centímetros, é: A) 3 B) 2 C) 2,5 D) 3 E) 4 Solução: Veja: (I) BF = 2 3 , pois BC//FE (II) ae = i 360º 60º a 120º 6 = → = Aplicando a lei dos cossenos no triângulo ABF, teremos: 2 2 2 2 2 (2 3) x x 2 x x cos120º 12 2x x x 2= + − ⋅ ⋅ ⋅ → = + → = Resposta: B 137 Qualquer que seja x, o valor de sen6 x + cos6 x + 3sen2 xcos2 é: A) 0 B) sen2 x C) cos2 x D) 1 E) senx⋅ cosx Solução: Lembre: produto notável → (a + b)3 = a3 + b3 + 3ab(a + b) Temos que: sen2 x + cos2 x = 1→ (sen2 x + cos2 x)3 = 13 → sen6 x + cos6 x + 3sen2 xcos2 x(sen2 x + cos2 x) = 1 → sen6 x + cos6 x + 3sen2 xcos2 x⋅ 1= 1 → sen6 x + cos6 x + 3sen2 xcos2 x = 1. Resposta: D 138 Quantas soluções reais e distintas possui a equação x2 + 9 = 3senx? A) 0 B) 1 C) 2 D) 3 E) infinitas Solução: Observe: (I) x2 + 9 é sempre maior ou igual a 9, para todo x real. (II) 3sen x assume no máximo o valor 3. (III) A igualdade não ocorre para nenhum valor real de x. Logo, a equação não possui solução. Resposta: A 139 O resto da divisão de P(x) = x5 + 4x4 + 2x3 + x2 + x –1 por q(x) = x + 2 é: A) 17 B) 15 C) 0 D) – 15 E) – 17 Solução: Veja: (I) – 2 é raiz de q(x) (II) P(–2) é o resto da divisão de P(x) por q(x). Então, pelo Teorema do resto, encontramos: resto = P(– 2) = – 32 + 64 – 16 + 4 – 2 – 1 = 17 Resposta: A C D E
  • 45. Professor: Fabrício Maia Matemática 49 140 O valor mínimo de cosx + secx, para 0 < x < π 2 é igual a: A) 0 B) 1 C) 2 D) 2 E) 3 Solução: Sabemos que (n – 1)2 ≥ 0, n∀ ∈1 Então: n2 – 2n + 1 ≥ 0 → n2 + 1 ≥ 2n Considerando n positivo, tem-se: 2 n 1 1 2 n 2 n n + ≥ ⇔ + ≥ Portanto, um número positivo adicionado ao seu inverso é sempre maior ou igual a 2. Resposta: D Anotações